Updated Midterm 2 Money and Banking

Réussis tes devoirs et examens dès maintenant avec Quizwiz!

3. When the amount of direct and indirect financing are summed, the result is usually: A. Greater than 100% of GDP B. Equal to GDP C. Less than GDP D. Approximately 50% of GDP

A

31. Which of the following issue exchange-listed option contracts? I. CBOE II. SEC III. OCC IV. NASDAQ A. III only B. IV only C. I and III only D. II and IV only E. I, II, and III only

A

Derivative markets exist to allow for: A. allow for the transfer of risk. B. direct transfers of common stocks for bonds. C. cash receipts from the sale of bonds. D. reduced information asymmetry.

A

A share of common stock represents:

A share of ownership of the company

You start with a portfolio valued at $500. over the next 12 months it loses 40%; the following year is has a gain of 30%. At the end of two years your portfolio is worth:

A. 390 500 x .6 = 300 x 1.3 =390

The short position in a futures contract is the party that will:

A. Deliver a commodity or financial instrument to the buyer at a future date

The fact that common stockholders are residual claimants means the stockholders:

A. have a claim against the revenue that remains after everyone else is paid

The option writer is:

A. the seller of an option

With a 10 percent reserve requirement ratio, a $100 deposit into New Bank means that the maximum amount New Bank could lend is ________. A) $90 B) $100 C) $10 D) $110

Answer: A

Bankers' concerns regarding the optimal mix of excess reserves, secondary reserves, borrowings from the Bank of Canada, and borrowings from other banks to deal with deposit outflows is an example of ________. A) liability management B) liquidity management C) managing interest rate risk D) managing credit risk

Answer: B

Banks develop statistical models to calculate their maximum loss over a given time period. This approach is known as the ________. A) stress-testing approach B) value-at-risk approach C) trading-loss approach D) doomsday approach

Answer: B

Credit risk management tools include ________. A) deductibles B) collateral C) interest rate swaps D) duration analysis

Answer: B

If a bank has ________ rate-sensitive assets than liabilities, then ________ in interest rates will increase bank profits. A) more; a decline B) more; an increase C) fewer; an increase D) fewer; a surge

Answer: B

Each of the Reserve Banks has a president who is:

Appointed by the bank's board of directors but approved by the board of governors.

Derivatives are financial instruments that:

B. when used correctly can actually lower risk

4. Emerging market economies, compared to industrialized economies, have financial markets that: A. Differ in composition and size B. Differ in composition but not in size C. Are the same in composition but differ in size D. Are similar in composition and size

C

43. Assume we have a stock currently worth $100. We also assume the interest rate is zero, and we can buy options for this stock with a strike price of $100. If the stock can rise or fall by $20 with equal probability over the option period, and the option cannot be exercised until the expiration date, what is the time value of the option? a. $20 b. $0 c. $10 d. $100

C

71. Requiring a large net worth on the part of an applicant is one way lenders treat the problem of: A. Free-riders B. Adverse selection C. Moral hazard D. The Lemons market

C

A $1,000 face value bond, with an annual coupon of $40, one year to maturity and a purchase price of $980 has a: A. current yield that equals 4.00%. B. coupon rate that equals 4.08%. C. current yield that equals 4.08% and a yield to maturity that equals 6.12%. D. current yield that equals 4.08% and a yield to maturity that equals 4.0%.

C

Financial intermediaries handle a larger flow of funds than do primary markets primarily because financial intermediaries: A. have a government-provided monopoly. B. have government-regulated prices, so there is little competition. C. can lower transaction costs and increase liquidity for savers. D. do not have to worry about information asymmetry.

C

One thing that is common for all bank loans is that they are: A. securitized. B. liquid. C. part of the banks' assets. D. unsecured.

C

Recession can cause widespread bank crises for all of the following reasons except: A. there is less business investment as banks make fewer loans. B. borrowers' default rates increase. C. bank capital increases. D. the negative effect on banks' balance sheets.

C. Bank capital increases

The long position in a futures contract is the party that will:

C. Benefit from increases in the price of the underlying asset

40. You bought a put with a strike price of $25. The current stock price is $23. What is the current payoff value of this option? A. -$2 B. -$1 C. $0 D. $1 E. $2

E

42. The maximum: A. profit from buying a put is the stock price. B. loss from writing a put is the option premium. C. profit from writing a call is the strike price. D. loss from buying a call is $0. E. profit from writing a put is the option premium.

E

49. Anna bought a $40 April call and a $40 April put on the same underlying stock. This strategy is referred to as which one of the following? A. bull spread B. bear spread C. parity play D. short straddle E. long straddle

E

Financial institutions, acting as financial intermediaries, perform all of the following except:

Increase transaction costs

A bank faces foreign exchange risk when:

It has assets denominated in one currency and liabilities in another.

An example of invisible bank run is:

Lenders refusing to buy asset backed commercial paper from an investment firm (Ch. 14)

The best way for a government to stop the failure of one bank from turning into a bank panic is to:

Make sure solvent institutions can meet the withdrawal demands of depositors. (Ch. 14)

The government's too-big-to-fail policy:

Means that the intermediary is too big or too complex to shut down or sell in an orderly fashion without large and painful spillovers.

Tom borrows 100K from his local bank to purchase inventory for his store for the upcoming holiday season. Tom's neighbor tells him about a get-rich-quick scheme that can take his 100k and triple it in a month. Tom decides to buy into this scheme figuring he can pay the bank and still have plenty left for inventory. This is an example of:

Moral Hazard

Which of the following is not a bank liability?

Reserves.

An arbitrageur is someone who:

Simultaneously buys and sells financial instruments to benefit from temporary price differences.

The principle-agent problem is quite common in large public corporations due to:

The fact that the people making the operational decisions are usually not the owners (Ch. 11)

The principal-agent problem is quite common in large public corporations due to:

The fact that the people making the operational decisions are usually not the owners.

What are the four fundamental characteristics that determine the value of a financial instrument?

The four fundamental characteristics that determine the value of a financial instrument are (1) The size of the payment that is promised; (2) When the promised payment is to be made; (3) the likelihood that the payment will be made; (4) The conditions under which the payment is to be made.

Financial instruments are different from money because: a. They can act as a store of value and money cannot b. They can't be a means of payment but money can c. They can allow for the transfer of risk d. They have greater liquidity

They can allow for the transfer of risk

If a bank has $100,000 of demand deposits, a desired reserve ratio of 20 percent, and it holds $40000 in reserves, then the maximum deposit outflow it can sustain without altering its balance sheet is ________. A) $30000 B) $25000 C) $20000 D) $10000

Answer: B

Which of the following is not an example of a backup line of credit? A) Loan commitments B) Overdraft privileges C) Standby letters of credit D) Mortgages

Answer: D

Inverted market

The case where the futures price is less than the cash price (In other words, the basis is positive)

Economies of scale associated with financial intermediaries means:

The cost per transaction fall as a larger volume of similar transactions are handled. (Ch.11)

Contagion is:

The failure of one bank spreading to other banks through withdrawing of funds.

15. Economies of scale associated with financial intermediaries means: A. The total cost of handling transactions falls as more transactions are handled B. The cost per transaction falls as a larger volume of similar transactions are handled C. The cost per transaction increases as more transactions are handled D. The cost per transaction decreases regardless of the size of the transaction

B

17. Tom buys a futures contract for U.S. Treasury bonds and on the settlement date the interest rate on U.S. Treasury bonds is lower than Tom expected. Tom will have: a. lost money on his long position. b. gained money on his long position. c. lost money on his short position. d. gained money on his short position.

B

21. If market participants believe next year's corn crop is likely to be unusually large: a. the current spot market price of corn is likely to be below the futures price of corn. b. the current spot market price of corn is likely to be above the futures price of corn. c. it would be impossible to find someone to take the short position in a futures contract. d. it will be impossible to find someone to take the long position in a futures contract.

B

33. Financial markets do not function as well as they could due to: A. The fact that banking is highly monopolized B. The cost of obtaining information can be high C. Regulation by governments D. Fluctuations in the inflation rate

B

39. You wrote a $40 call option on a stock that has a market price of $43. Which one of the following statements must be correct if the option expires three months from now? A. Your option currently has zero intrinsic value. B. Your option currently has a negative payoff. C. You have the right to purchase shares at $40 a share. D. Your option payoff will increase if the market price of the stock increases. E. If the market price remains stable, you will make the decision to exercise this option prior to expiration.

B

40. The intrinsic value of an option: a. is the amount the investor believes the option will be worth on the expiration date. b. is the amount the option is worth if it is exercised immediately. c. is equal to price of the underlying asset. d. cannot be determined without knowing the future price of the underlying asset.

B

78. You purchased 5 put option contracts on Mountain Builders stock at an option premium of $0.65. The strike price is $25. What is your break-even stock price? A. $19.90 B. $24.35 C. $25.00 D. $25.75 E. $30.10

B

80. Rosalita purchased a put option with a strike price of $35. She paid a total of $140 for the contract. What is the break-even stock price? A. $31.40 B. $33.60 C. $38.00 D. $41.60 E. $42.80

B

Diversification can eliminate: a. All risk in a portfolio b. Risk only if the investor is risk averse c. The systematic risk in a portfolio d. The idiosyncratic risk in a portfolio

The idiosyncratic risk in a portfolio

A bank's net interest margin is calculated by taking net interest income and: A. dividing it by the bank's capital. B. dividing it by the bank's assets. C. dividing it by the sum of the bank's assets and capital. D. subtracting taxes.

B

A bank's off-balance-sheet activities usually: A. increase both its assets and liabilities while reducing net income. B. increase its net income but do not change its assets or liabilities. C. increases a bank's liabilities but not its assets. D. increases a bank's assets but not its liabilities.

B

A financial intermediary: A. is an agency that guarantees a loan. B. is a third-party that facilitates a transaction between a borrower and a lender. C. would be used in direct finance. D. must be a depository institution.

B

A student receives a five-year loan to pay for a $2,000 used car. The lender and the student agree to an 8% interest rate on a fixed-rate loan. Expected inflation was estimated to equal 2.5%, but unexpectedly decreases to 2%. Which of the following is true? A. The real interest rate decreased. B. The student is made worse off because her real cost of borrowing is higher. C. The lender is made worst off because his real return on the car loan is lower. D. Both the student and the lender benefit.

B

Commissions paid to a stock broker are an example of: A. risk transfer. B. transaction costs. C. information asymmetry. D. liquidity.

B

Financial markets: A. enable buyers and sellers to exchange financial instruments but not risk. B. enable buyers and sellers to exchange risk by buying and selling financial instruments. C. only allow the transfer of risk through derivative securities. D. do not allow for the transfer of risk but do help reduce it.

B

What is the primary distinction between debt/equity markets and derivative markets?

The market for equities (stocks) and debt (mainly bonds) are markets where the actual claims are purchased or sold for immediate cash payment. On the other hand, in derivative markets, parties and counterparties make agreements that are settled at a later date.

To make sure the U.S. President cannot unduly influence the Board of Governors:

The terms of the governor are staggered (Ch. 16)

If a one-year zero-coupon bond has a face value of $100, is purchased for $94, and is held to maturity:

The yield to maturity will be 6.38%.

Central banks often find:

There are tradeoffs that make pursuing all of their goals simultaneously impossible (Ch. 15)

Secondary financial markets: A. are financial markets for all financial instruments rated less than investment grade. B. are financial markets where existing securities are bought and sold. C. eliminate the transaction costs for buyers and sellers. D. are only for stock.

B

Secondary reserves for banks are: A. the same as the bank's net worth. B. mainly the bank's liquid securities. C. vault cash. D. deposits the bank has at the Federal Reserve.

B

The impact of a decrease in expected inflation in the bond market will have a relatively large effect on the prices of bonds prices because the bond demand curve: A. will shift right as will the bond supply curve. B. will shift right but the bond supply curve shifts left. C. and supply curves will shift left. D. will shift left as the bond supply curve shifts right.

B

The largest liability for commercial banks in the U.S. is: A. demand deposits. B. non-transaction deposits. C. borrowing from other U.S. banks. D. borrowing from the Federal Reserve.

B

The most prominent of asset-backed securities is: A. shares of stock in corporations since stockholders own the assets. B. securities backed by home mortgages. C. U.S. Treasury bonds since they are backed by all public assets. D. movie box-office receipts.

B

The pool of information collected by financial markets is usually: A. only available to lenders. B. summarized in the form of a price. C. valuable and not made available until the parties pay for it. D. more than a borrower needs to make a loan.

B

When a loan is amortized, it means the: A. borrower is in default. B. principal and interest are paid off by the borrower over the life of the loan. C. interest is due entirely at the maturity date. D. principal in never repaid, only interest.

B

A well-capitalized financial institution has ________ to lose if it fails and thus is ________ likely to pursue risky activities. A) more; more B) more; less C) less; more D) less; less

B) more; less

You hold an FDIC insured savings account at your neighborhood bank. Your current balance is $275,000. If the bank fails you will receive: A. $275,000. B. $250,000. C. $100,000. D. $125,000.

B. $250,000

comparing an option to a futures contract it would be correct to say:

B. A futures contract carries more risk than the options contract

The Dow Jones Industrial Average is a:

B. Price-weighted index

Interest-rate swaps are:

B. agreements between two parties to exchange periodic interest-rate payments over some future period

A call option is:

B. an option giving the holder the right to buy a given quantity of an asset at a specific price on or before a specified date

considering a put option, an increase in the strike price:

B. causes the intrinsic value of the option to increase if it is above zero

A U.S Treasury bond dealer with a large portfolio who sells a futures contract for U.S Treasury bonds is:

B. ensuring the sales price o the bond through hedging

Tom buys a futures contract for US treasury bonds and on the settlement date the interest rate on US treasury bonds is lower than tom expected. Tom will have:

B. gained money on his long position

the time value of the option should:

B. increase the longer the time to expiration

for a given call option price, which of the following statements is correct?

B. the closer the strike price is to the current price of the underlying asset, the larger is the time value of the option

One of the unique problems that banks face is: A. they hold liquid assets to meet illiquid liabilities. B. they hold illiquid assets to meet liquid liabilities. C. they hold liquid assets to meet liquid liabilities. D. both their assets and their liabilities are illiquid.

B. they hold illiquid assets to meet liquid liabilities

20. Sue buys a futures contract for U.S. Treasury bonds and on the settlement date the interest rate on U.S. Treasury bonds is higher than Sue expected. Sue will have: a. gained money on her short position. b. gained money on her long position. c. lost money on her long position. d. lost money on her short position.

C

23. Louise just purchased 3 call option contracts on GE stock. How many shares of stock can she buy at the strike price based on these contracts? A. 3 B. 30 C. 300 D. 30,000 E. 300,000

C

25. The option holder is: a. the seller of an option. b. another name for the clearinghouse used in futures contracts. c. the buyer of an option. d. always a speculator.

C

26. At what price will a dealer sell the Jun $34 put on General Electric stock? ========= A. $1.64 B. $1.73 C. $1.77 D. $2.52 E. $2.56

C

29. The strike price of an option is: a. the market price at the time the option is written. b. the market price at the time the option is exercised. c. the price at which the option holder has the right to buy or sell. d. always above the market price.

C

3. By definition, a put option grants its owner which one of the following? A. right to buy B. obligation to buy C. right to sell D. obligation to sell E. choice to either buy or sell

C

64. Moody's, Value Line, and Dun and Bradstreet are examples of companies that: A. Provide information free to investors but charge the companies for the ratings provided on the company B. Provide information free to investors but recoup expenses through advertising done by the companies being rated C. Charge investors who subscribe to the services for the information D. Duplicate information that is available to investors at no cost

C

83. A 4-month call has a strike price of $20. The current underlying stock price is $21.45. What is the intrinsic value of this call? A. $0.00 B. $0.48 C. $1.45 D. $3.90 E. $4.35

C

84. One reason lenders may require a large net worth before making a loan is because: A. Then the borrower does not need the funds B. It tells the lender the firm has good employees C. It is one way to treat the problem of moral hazard D. Banking laws require firms have significant net worth before a bank can make a loan

C

A bank's loan loss reserves are: A. the amount of loans that have defaulted in the past twelve months. B. the same as equity capital. C. an amount the bank sets aside to cover potential losses from defaulted loans. D. a liability of the bank since it is a source of funds.

C

A bank's net worth is synonymous with its: A. assets. B. assets + a bank's liabilities. C. capital. D. required reserves.

C

Considering the value of a financial instrument, the bigger the size of the promised payment the: A. less valuable the financial instrument because risk must be greater. B. longer an investor has to wait for the payment. C. more valuable the financial instrument. D. greater the risk.

C

Disability Income Insurance is: A. insurance borrowers can take out in case the company they invest in defaults. B. insurance that makes payments of wages to workers when the company they work for is disabled due to a natural disaster. C. insurance that makes payments to workers when they are unable to work due to an injury. D. only available through the government as part of the Social Security System.

C

Fly-By-Night Inc. issues $100 face value, zero-coupon, one-year bonds. The current return on one-year, zero-coupon U.S. government bonds is 3.5%. If the Fly-By-Night bonds are selling for $92.00, what is the risk premium for these bonds? A. 8.7% B. 1.5% C. 5.2% D. 8.0%

C

If a bank has $100 million in assets and a net worth of $10 million, its debt-to-equity ratio is: A. 10 to 1. B. 5 to 1. C. 9 to 1. D. 0.1 to 1.

C

If a consol is offering an annual coupon of $50 and the annual interest rate is 6%, the price of the consol is: A. $47.17 B. $813.00 C. $833.33 D. $8333.33

C

The holding period return has relevance because: A. most bonds are held by the original purchaser until maturity. B. most bonds are held by the original purchaser until they mature. C. bonds are frequently traded. D. current yields are not that important to bondholders.

C

if a futures contract for US Treasury bonds increases by "12" in the financial page listings, the value of the contract is increased by:

C. 375

14. One argument why farmers in poor countries remain poor is: a. they know very little about farming techniques needed for the crop they are growing. b. they are poor assessors of the risks they face. c. risk taking is a deterrent to growth. d. poor farmers in many countries lack access to commodity futures markets.

D

17. Which of the following has the obligation to purchase stock at the strike price when an option is exercised? A. call holder B. call writer C. put holder D. put writer E. call writer and put holder

D

24. How much option premium per share will you receive if you sell a September $34 put on General Electric stock? ============ A. $1.64 B. $1.68 C. $1.77 D. $2.52 E. $2.56

D

5. The reason financial intermediaries play such an important role in economies has to do with all of the following except: A. Information costs B. High transaction costs C. Complexity of a lot of financial transactions D. The composition of GDP

D

53. Which one of the following correctly defines the range of time values for a put option? A. $0 to +$1 B. -$1 to +$1 C. ≤ $0 D. ≥ $0 E. ¹ $0

D

9. By definition, stock index options would include an option on which one of the following underlying assets? A. gold B. corn C. U.S. dollar D. S&P 500 E. U.S. Treasury bill

D

A bank that cannot meet its loan commitments is experiencing the results of: A. interest rate risk. B. credit risk. C. trading risk. D. liquidity risk.

D

A counterparty to a financial instrument is always the: A. issuer of the financial instrument. B. government agency guaranteeing the value of the instrument. C. person or institution that purchases the financial instrument. D. person or institution that is on the other side of the financial contract.

D

The procedure that estimates the interest-rate sensitivity of a bank's assets and liabilities is called: A. managing credit risk. B. estimating operating risk differential. C. trading risk minimization. D. gap analysis.

D

The return on bonds rises relative to other assets, in the bond market this will result in: A. the price of bonds falling and the yields increasing. B. a rightward shift in the bond supply curve. C. a shift to the left of the bond demand curve. D. an increase in bond prices.

D

17) A loan where the borrower receives money today and repays a single lump sum on a future date is called a(n) ________ loan. A) amortized B) continuous C) balloon D) pure discount E) interest-only

D) pure discount

You have two savings accounts at an FDIC insured bank. You have $225,000 in one account and $40,000 in the other. If the bank fails, you will receive: A. $225,000. B. $40,000. C. $115,000. D. $250,000.

D. $250,000

Voting rights in a corporation are held by the:

D. common stockholders preferred stock has no voting rights

A call option described as at the money would find:

D. the market price of the stock equals the strike price

A call option described as out of the money would find

D. the strike price is above the market price of the stock

29. Which of the following characteristics are correct regarding the new style option quotation system? I. The system is known as OPRA - the Options Price Reporting Authority code II. The system has 3 data elements III. The system has 21 characters IV. The system has 5 characters V. The system is known as the OCC Series Key VI. The root symbol is the underlying stock's ticker symbol A. I, II, III, and VI B. II, III, IV, and V C. I, II, and IV D. II, III, V, and VI E. III, V, and VI

E

If bank with leverage of 8 to 1 increases its assets by adding $1 to capital for every $1 added to assets:

Leverage decreases

In its role as the bankers' bank, a central bank performs each of the following except:

Providing deposit insurance

The theory of efficient markets implies:

Stock prices should be highly unpredictable.

Cash-futures arbitrage

Strategy for earning risk-free profits from an unusual difference between cash and futures prices

Which of the following is (are) not a permanent voting member on the FOMC?

The Secretary of the Treasury (Ch. 16)

In the United States, one problem with central bank independence is:

The United States is a democracy and having an independent central bank is inconsistent with a representative democracy. (Ch. 15)

Commercial banks increased their involvement in mortgages over the years due to:

The ability to securitize mortgages which made them more liquid

A bank's reserves include:

The bank's deposits at the Federal Reserve

A consol is:

a bond that makes periodic interest payments forever.

According to the liquidity premium theory of the term structure, a slightly upward sloping yield curve indicates that short-term interest rates are expected to

remain unchanged in the future

According to the liquidity premium theory of the term structure, a steeply upward sloping yield curve indicates that short-term interest rates are expected to

rise in the future

which of the following is not a feature of common stock? a. stockholders receive regular fixed payments on their shares b. stockholders have limited liability c. stock holders are residual claimants d. stockholders have voting rights

stockholders do NOT A. receive regular fixed payments on their shares

If the U.S. government's borrowing needs increase, all other factors constant the:

supply of bonds will increase

81. You own 100 shares of Deltona stock which is currently worth $43 a share. You just paid an option premium of $0.85 to buy one put contract on this stock with a strike price of $40. What is the maximum loss per share you are avoiding by purchasing the option contract? A. $40.00 B. $40.85 C. $42.15 D. $43.00 E. $43.85

A

Which of the following bank assets would be categorized as secondary reserves? a. U. S Treasury bills b. Cash c. Mortgage loans d. Deposits at the Federal Reserve

US Treasury bills

Once you buy a coupon bond, which of the following can change?

Yield to maturity

Bank reserves include ________. A) deposits at the Bank of Canada and short-term securities B) vault cash and short-term securities C) vault cash and deposits at the Bank of Canada D) deposits at other banks and deposits at the Bank of Canada

Answer: C

Bank's make their profits primarily by issuing ________. A) equity B) negotiable CDs C) loans D) notice deposits

Answer: C

Banks that suffered significant losses in the 1980s made the mistake of ________. A) holding too many liquid assets B) minimizing default risk C) failing to diversify their loan portfolio D) holding only safe securities

Answer: C

If, after a deposit outflow, a bank needs an additional $3 million to meet its desired reserves, the bank can ________. A) reduce deposits by $3 million B) increase loans by $3 million C) sell $3 million of securities D) repay its advances from the Bank of Canada

Answer: C

When you deposit $50 in currency at Old National Bank, ________. A) its assets increase by less than $50 because of reserve requirements B) its reserves increase by less than $50 because of reserve requirements C) its liabilities increase by $50 D) its liabilities decrease by $50

Answer: C

Which of the following is not a source of borrowings for a bank? A) Overnight funds B) Eurodollars C) Time deposits D) Advances

Answer: C

According to the Expectations Theory of the term structure, if interest rates on one year bond are expected to be 2%, 2%, 4%, and 5% over the next four years, what is the yield on a three-year bond today? a. 2.7% b. 4% c. 4.3% d. 8%

2.7%

Which of the following would a bank not hold as insurance against the highest cost of deposit outflow-bank failure? A) Excess reserves B) Secondary reserves C) Bank capital D) Mortgages

Answer: D

Which of the following makes fixed payments indefinitely? A. Amortized loan B. Consol C. Coupon bond D. Zero-coupon bond

B

The process by which simultaneous withdrawals by a particular bank's depositors results in the bank closing is known as a:

Bank Run

Which of the following is not a bank liability? A. Reserves B. Demand deposits C. Non-transaction deposits D. Federal fund borrowings

A

The creation of the Federal Reserve in 1913: A. provided the opportunity for lender of last resort but not the guarantee that it would be used. B. guaranteed the Federal Reserve would always act as lender of last resort. C. eliminated bank panics in the U.S. D. was in response to the Great Depression in the U.S.

A. provided the opportunity for lender of last resort but not the guarantee that it would be used

You hold an FDIC insured savings account at your neighborhood bank jointly with your father. Each of you has contributed equally into the account. The current balance in the account is $300,000. If the bank fails each of you will receive:

$150,000

The price of a stock is currently $750 and the stock will pay a $43 dividend. The interest rate is 7.5%. Based on the dividend-discount model, what is the expected price of this stock for next year?

$763.25.

If a consol is offering an annual coupon of $50 and the annual interest rate is 6%, the price of

$833.33

If the annual interest rate is 5% (.05), the price of a one year Treasury bill would be: (Hint: T-Bills have $100 face value)

$95.24

If the annual interest rate is 5% (.05), the price of a one-year Treasury bill per $100 of face value would be: a. $95.00 b. $97.50 c. $95.24 d. $96.10

$95.24

If the annual interest rate is 5%(.05), the price of a one-year Treasury bill per $100 of face value would be:

$95.24

Bank capital has both benefits and costs for the bank owners. Higher bank capital ________ the likelihood of bankruptcy, but higher bank capital ________ the return on equity for a given return on assets. A) reduces; reduces B) increases; increases C) reduces; increases D) increases; reduces

Answer: A

You have a portfolio valued at $1000. Over the next twelve months it loses 80% of its value. What return does the portfolio need to earn over the following twelve months to restore the portfolio to its original value?

400%.

If 1-year interest rates for the next five years are expected to be 4, 2, 5, 4, and 5 percent, and the 26) 5-year term premium is 1 percent, than the 5-year bond rate will be

5 percent

19. Selling a call option on stock which you own is referred to as which one of the following strategies? A. covered call B. naked call C. protective put D. underlying put E. straddle

A

53. Used car dealers that provide warranties on the cars they sell are treating the: A. Lemons problem B. Monopoly problem C. Problem of people preferring foreign cars D. Adverse selection problem of buyers preferring new versus used cars

A

72. Requiring a home buyer to have a large down payment reduces the risk to a mortgage lender because: A. If the price of the home falls the buyer is still likely to stay B. The buyer is less likely to sell the house C. It means the buyer likely underpaid when she bought the house D. It means there is more information available on the buyer

A

As general business conditions deteriorate, all other factors constant: A. the demand for bonds will decrease. B. the supply of bonds will increase. C. bond prices will decrease. D. bond yields will increase.

A

Newly issued U.S. Treasury Securities are sold in: A. the primary financial market. B. only to the Federal Reserve who then resells them. C. the secondary market since bonds cannot be sold in the primary market. D. secondary markets but only using registered bond dealers.

A

In considering the holding period return, the longer the term of the bond the: A. less important is the capital gain and the more important in the current yield. B. less important is the coupon rate and the more important is the current yield. C. less important is the capital gain. D. more important is the capital gain.

D

48. A firm that has a well-earned reputation for providing high quality: A. Has found a way to treat the free-rider problem B. Has found a way to treat the moral hazard problem C. Has found a way to treat the problem of adverse selection D. Will not survive in a market if low quality is provided at a lower price

C

When expected inflation decreases for any given nominal interest rate, all of the following occur except the: A. real interest rate decreases. B. bond supply curve shifts to the left. C. cost of borrowing increases and the desire to borrow decreases. D. price of bonds increases.

C

When expected inflation increases, for any given nominal interest rate the: A. real cost of repayment for bond issuers increases. B. real return for bondholders increases. C. real cost of repayment for bond issuers decreases. D. bond demand curve shifts right.

C

Which of the following financial instruments is used mainly to transfer risk? A. Asset-backed securities B. Bonds C. Options D. Stocks

C

a key use of interest-rate swaps is to:

C. provide a hedge against interest-rate risk

a wheat farmer who must purchase his inputs now but will sell his wheat at a market price at a future dateL

C. would hedge by taking the short position in a wheat futures contract

The price (P) of a consol offering an annual coupon payment (C) is best expressed by:

C/i

The interest rate decisions made by the FOMC:

Cannot be overridden by anyone outside of the Federal Reserve

Whenever central bankers face more than one goal, the policy framework requires:

Central bankers to make their priorities clear (Ch. 15)

Compared to an independent central bank, elected officials are likely to:

Choose monetary policies that are overly accommodative

A bank's net interest margin is calculated by taking net interest income and:

Dividing it by the bank's assets

22. Consider both a European put and call that expire in June and have a strike price of $30. The no-arbitrage relationship between this put and call is referred to as which one of the following? A. intrinsic equilibrium B. Euro-match C. bull-call spread D. butterfly spread E. put-call parity

E

The regulatory system that has evolved in the United States whereby banks are regulated at the state level, the national level, or both, is known as a A) bilateral regulatory system. B) tiered regulatory system. C) two-tiered regulatory system. D) dual banking system.

dual banking system.

61. The problem of adverse selection created the opportunity for: A. Lenders to profit significantly at the expense of borrowers B. Significant deregulation of financial markets C. A new market in the trading of information D. Stock prices for many years to be much lower than what they should have been

C

66. Requiring that borrowers put up collateral to obtain a loan is a tool designed to treat: A. The Lemons Problem B. The problem of adverse selection C. The problem of moral hazard D. The free-rider problem

C

If the quantity of bonds supplied exceeds the quantity of bonds demanded, bond prices would:

fall and yields would rise.

Brokerage commissions: A. are set by government regulators so they cannot vary across firms for the same services. B. can vary but typically don't because firms tend to set them at the same levels. C. can differ reflecting the different services being offered. D. are always a percentage of the amount of the trade.

C

Consider a one-year corporate bond that has a 20% probability of default. The payoff on the bond is $2,000 if the corporation does not default. The interest rate is 10%. If buyers of this bond are risk-neutral, this bond will sell for: A. $400 B. $909.09 C. $1,454.54 D. $1,600

C

The information concerning the issuer of a financial instrument: A. needs to be complete and closely monitored by the buyers of the instrument for change. B. is somewhat non-standardized to minimize the cost of the instrument. C. is usually standardized to the essential information required by the buyers. D. is closely monitored by the buyers of these instruments for change.

C

Well-run financial markets: a. Keep transactions costs high to benefit brokers b. Prevent the widespread pooling of information c. Ensure that resources are allocated efficiently d. Are usually the result of little or no government regulation

Ensure that resources are allocated efficiently

Full hedge

A futures position that is equal; fully long and fully short

Interest-rate risk would not matter to which of the following bondholders?

A holder of a U.S. government bond that plans on holding it until it matures.

There is a futures contract for the purchase of 1,000 bushels of corn at $3.00 per bushel. At the end of the day when the market price of corn falls to $2.50:

A. the buyer (long position) needs to transfer $500 to the seller (short position)

considering a call option, if the price of the underlying asset decreases:

A. the intrinsic value of the option decreases if it is above zero

we have a stock selling for $90. there's a put option for this stock with a strike price of $85 and an option price of $1.20:

A. the intrinsic value of this option is 0 and the time value is 1.20

Under the European System of Central Banks, the Governing Council is similar in structure to the ________ of the Federal Reserve System. A) Board of Governors B) Federal Open Market Committee C) Federal Reserve Banks D) Federal Advisory Council

Federal Open Market Committee

In terms of economic growth, the central bank would like to:

Keep the economy close to its potential or sustainable rate of growth (Ch. 15)

3. Forward contracts are: a. an agreement between more than two parties. b. contracts usually involving the exchange of a commodity or financial instrument. c. always standardized. d. easily resold.

B

30. With a call option, the option holder: a. has the right to sell the asset. b. has the right to buy the asset. c. can buy or sell, it is their option. d. can buy the asset but only after the date specified.

B

5. With a futures contract: a. payment is made when the contract is created. b. no payment is made until the settlement date. c. the short position agrees to purchase the underlying asset. d. the risk is eliminated for both parties.

B

51. Interest-rate swaps are: a. exchanges of equity securities for debt securities. b. agreements between two parties to exchange periodic interest-rate payments over some future period. c. agreements involving swapping of option contracts. d. agreements that allow both parties to convert floating interest rates to fixed interest rates.

B

63. The price for private information is likely higher than it should be and the number of subscribers is lower than users due to the problem of: A. Adverse selection B. Free-riders C. The government regulations regarding information D. Moral hazard

B

There's a call option written for 100 shares of GM stock for $85.00 a share, prior to the third Friday of October 2013: The option writer:

Is required to post margin.

85. The market for bonds is initially described by the supply of bonds - S0, and the demand for bonds - D0, with the equilibrium price and quantity being P0 and Q0. An increase in the nation's wealth, all else constant, would cause the A. Bond supply curve to shift to S1. B. Bond demand curve to shift to D1. C. Bond supply curve to shift to S2. D. Bond demand curve to shift to D2.

B

A bank's assets tend to be long-term while its liabilities are short-term. Therefore, when interest rates rise, the value of the bank's assets: A. increases by more than the value of its liabilities. B. will decrease by more than the value of its liabilities. C. increases and the value of its liabilities decreases. D. decreases and the value of its liabilities increases.

B

As general business conditions deteriorate, all other factors constant:

the demand for bonds will decrease.

According to the segmented markets theory of the term structure

the interest rate for each maturity bond is determined by supply and demand for that maturity bond.

If a bank has customer deposits of $150 million, $15 million in reserves and the amount of excess reserves equals 0 (zero): A. the required reserve rate is 15 percent. B. the required reserve rate is 10 percent. C. the required reserve rate is 1 percent. D. the bank's net interest margin is zero (0).

B

Interest-rate risk results from:

a mismatch between an individual's investment horizon and a bond's maturity.

When the yield curve is flat or downward-sloping, it suggest that the economy is more likely to enter

a recession

Suppose that the expected return on bonds falls relative to other assets. In the bond market this will result in:

a shift to the left of the bond demand curve.

The clearing corporation's main role in the futures market is to:

B. Act as the counterparts to both sides of the transaction, thereby guaranteeing a payment

For every $100 in assets, a bank has $30 in interest-rate sensitive assets, and the other $70 in non-interest-rate sensitive assets. The same bank has $60 for every $100 in liabilities in interest-rate sensitive liabilities, the other $40 are in liabilities that are not interest-rate sensitive. If the interest rate on assets decreases from 6 to 5 percent, and the interest rate on liabilities decreases from 4 to 3, percent the impact on the bank's profits per $100 of assets will be: a. A reduction of $0.30 b. An increase of $0.30 c. A reduction of $3.00 d. Zero since the interest rates on assets and liabilities fell by the same amount

b. An increase of $0.30 (RSA-RSL) * change in i (30-60)*-.01=.30

Which of the following statements is most correct? A. Financial regulators do everything possible to encourage competition in banking. B. Financial regulators work to prevent monopolies but also work to prevent strong competition in banking. C. Financial regulators discourage competition in banking. D. Financial regulators prefer banks to have monopoly power in their geographic markets.

B. Financial regulators work to prevent monopolies but also work to prevent strong competition in banking

When healthy banks fail due to widespread bank panics, those who are likely to be hurt are: A. government regulators. B. households and small businesses. C. the FDIC. D. the Federal Reserve.

B. Households and small businesses

With a futures contract:

B. No payment is made until the settlement date

Savings banks and savings and loans are regulated by a combination of agencies which includes the: A. Federal Reserve System. B. Office of the Comptroller of the Currency. C. Securities and Exchange Commission. D. Internal Revenue Service.

B. Office of the Comptroller of the Currency

When the price of a bond equals the face value the:

current yield is equal to the coupon rate.

The segmented markets theory can explain

why yield curves usually tend to slope upward.

When the price of a bond is below the face value, the yield to maturity:

will be above the coupon rate.

Most economies agree that a well-designed central bank would:

Be independent of political pressure

The Federal Reserve's Fedwire system is used to mainly provide:

An inexpensive and reliable way for financial institutions to transfer funds to one another (Ch. 15)

Traders working for banks are subject to the ________. A) principal-agent problem B) free-rider problem C) double-jeopardy problem D) exchange-risk problem

Answer: A

Which of the following are primary concerns of the bank manager? A) Maintaining sufficient reserves to minimize the cost to the bank of deposit outflows B) Extending loans to borrowers who will pay low interest rates, but who are poor credit risks C) Acquiring funds at a relatively high cost, so that profitable lending opportunities can be realized D) Maintaining high levels of capital and thus maximizing the returns to the owners

Answer: A

If the quantity of bonds demanded exceeds the quantity of bonds supplied, bond prices:

would rise and yields would fall

If a one-year zero-coupon bond has a face value of $100, is purchased for $94, and is held to maturity the:

yield to maturity will be 6.38%

Because of their ________ liquidity, ________ government securities are called secondary reserves. A) low; short-term B) low; long-term C) high; short-term D) high; long-term

Answer: C

When a lender refuses to make a loan, although borrowers are willing to pay the stated interest rate or even a higher rate, the bank is said to engage in ________. A) coercive bargaining B) strategic holding out C) credit rationing D) collusive behavior

Answer: C

A pure discount bond is also known as a:

zero-coupon bond.

Which of the following would not be a way to increase the return on equity? A) Buy back bank stock B) Pay higher dividends C) Acquire new funds by selling negotiable CDs and increase assets with them D) Sell more bank stock

Answer: D

The central bank which is generally regarded as the most independent in the world because its charter cannot be changed by legislation is the A) Bank of England. B) Bank of Canada. C) European Central Bank. D) Bank of Japan.

C) European Central Bank.

The first test of the Federal Reserve as lender of last resort occurred with the: A. attack on Pearl Harbor by the Japanese. B. widespread failures of Savings and Loans in the 1980's. C. introduction of flexible exchange rates in the U.S. in 1971. D. stock market crash in 1929.

D. Stock market crash in 1929.

When studying world stock indexes, we observe that:

D. The indexes are comparable but only in percentage terms

One reason customers do not care about the quality of their bank's assets is: A. most people cannot distinguish an asset from a liability. B. the quality of a bank's assets changes almost daily. C. they assume the bank only has high quality assets. D. with deposit insurance, there isn't any real reason to care; their deposits are protected even if the bank fails.

D. With deposit insurance, there isn't any real reason to care; their deposits are protected even if the bank fails.

The federal government is concerned about the health of the banking system for many reasons, the most important of which may be: A. banks are where government bonds are traded. B. a significant number of people are employed in the banking industry. C. many people earn the majority of their income from interest on bank deposits. D. banks are of great importance in enabling the economy to operate efficiently.

D. banks are of great importance in enabling the economy to operate efficiently.

the principal in an interest rate swap is:

D. is not borrowed, lent, or exchanges. it just serves as the basis for the calculation of cash flows

With a put option, the option holder:

Has the right to sell the asset.

The U.S. Treasury offers several ways to purchase U.S. government bonds. There are the traditional coupon bonds and Treasury Inflation-Indexed Securities. How do these bonds differ from their traditional counterparts?

Inflation-indexed securities protect the borrower against inflation risk. If inflation is higher than expected, this will reduce bond prices for the traditional government bonds, but not the inflation-indexed bonds.

Explain why non-transactions accounts have become a more important source of funds for the bank than transaction accounts over the past thirty years?

Non-transactions accounts include savings and time deposits, and certificates of deposit. Over the past quarter of a century financial innovation and technology have caused people to economize on their deposits in transaction accounts (checkable deposits) since most of these accounts paid little to no interest. The development of money market accounts and the technology that has allowed people to transfer funds out of savings accounts when their checking account balances run low has caused the balances in the savings accounts to increase significantly.

The payoff method used by the FDIC to address the insolvency of a bank is when the FDIC:

Pays off the depositors up to the current $250,000 limit, so it is possible that some depositors will suffer losses (Ch. 14)

Bank capital has both benefits and costs for the bank owners. Higher bank capital _____ the likelihood of bankruptcy, but higher bank capital _____ the return of equity for a given return on assets.

Reduce; Reduce

The government's too-big-to-fail policy:

Reduces the risk faced by depositors with accounts exceeding $250,000 (Ch. 14)

Why would a bank usually want to minimize the amount of excess reserves it has on hand?

Reserves are assets held in the form of vault cash and deposits at the Federal Reserve in a non-interest bearing account. As a result, the opportunity cost of holding reserves is usually high. Regulations require banks to hold a certain amount of reserves, called required reserves, but any amounts held above this level are excess reserves and the bank is forgoing the return that could have been earned on these funds.

The return on bonds rises relative to other assets, in the bond market this will result in:

an increase in bond prices.

A key assumption in the segmented markets theory is that bonds of different maturities

are not substitutes at all.

. If a bank has $150 million in assets and a net worth of $20 million, its asset-to-equity ratio is: a. 6.5 to 1 b. 7.5 to 1 c. 0.13 to 1 d. 0.15 to 1

b. 7.5 to 1

The spread between the interest rates on bonds with default risk and default-free bonds is called the

risk premium

If the risk on foreign government bonds increases relative to U.S. government bonds, the price of U.S. government bonds should:

increase as the demand for these bonds increases.

Other things being equal, an increase in the default risk of corporate bonds shifts the demand curve for corporate bonds to the ________ and the demand curve for Treasury bonds to the ________.

left; right

a price of a futures contract for US Treasury Bonds listed as "111-15" is measured in:

A. 32nds

the intrinsic value of the option

B. is the amount the option is worth if it is exercised immediately

The interest rate that the FOMC currently chooses to control is:

The Federal Funds Rate

In the U.S. the authority to issue currency is held by:

The Federal Reserve

Monetary policy in the United States is under the control of:

The Federal Reserve

As general business conditions deteriorate, all other factors constant:

the bond supply curve will shift left.

Consider the bonds below. Which is subject to the greatest interest-rate risk?

A consol

What are the four characteristics of a financial instrument?

1) A financial instrument is a written legal obligation; (2) A financial instrument transfers something of value to another party; (3) A financial instrument specifies some future date for this transfer to occur; and (4) A financial instrument specifies certain conditions under which payment will be made.

A bank run involves:

A large number of depositors withdrawing their funds during a short time span (Ch. 14)

Considering interest-rate swaps, the swap rate is:

A. the benchmark rate plus a premium

If financial intermediaries did not have the ability to pool the resources of small savers:

Borrowers needing large amounts of money would find it more costly to obtain the funds.

Which of the following makes fixed payments indefinitely?

Consol

Which of the following best expresses the equation for holding period return?

Current yield + capital gain

What type of contract is exposed to counter party risk? (Forward or future?)

Forward contracts

What type of contract is sold on an exchange? (Forward or future?)

Futures contracts

The Dow Jones Industrial Average:

Gives greater weight to shares with higher prices.

If the risk on foreign government bonds increases relative to U.S. government bonds, the price of U.S. government bonds should:

Increase as the demand for these bonds increases.

. The risk premium for an investment: a. Is negative for U.S. Treasury Securities b. Is a fixed amount added to the risk-free return, regardless of the level of risk c. Increases with risk d. Is zero (0) for risk-averse investors

Increases with risk

Standardization of derivative contracts:

Leads to greater liquidity and lower risk.

A bank that cannot meet its loan commitments is experiencing the results of:

Liquidity risk (Ch. 12)

Credit default swaps contributed to the financial crisis of 2007-2009 by:

Making it easier for sellers of insurance to assume and conceal risk.

Financial intermediaries pool the resources of many small savers so that they can:

Obtain the funds necessary to make loans to borrowers seeking large amounts

That fact that banks often make loans to other banks means:

One bank's failure can be contagious and spread to other banks (Ch. 14)

66. You purchased one SPX call option with a strike of 1,500. You wrote one SPX call option with the same maturity date and a strike of 1,450. At maturity, what is your payoff if the S&P 500 is at 1,475? A. -$2,500 B. -$250 C. $25 D. $250 E. $2,500

S

Considering the balance sheet for all commercial banks in the U.S., the largest category of liabilities is:

Savings deposits and time deposits

Considering the balance sheet for all commercial banks in the U.S., the largest category of liabilities is:

Savings deposits and time deposits (Ch. 12)

The credit risk a bank faces is the risk resulting specifically from:

Some of the bank's loans not being repaid.

Carrying Charge Market

The case where the futures price is greater than the cash price (In other words, the basis is negative)

Underyling asset

The commodity or financial instrument on which the futures contract is based

What is the equation that reflects a bank's balance sheet?

The equation reflecting a bank's balance sheet is: Total Bank Assets = Total Bank Liabilities + Bank Capital

One lesson learned from the bank panics of the early 1930s is:

The mere existence of a lender of last resort will not keep the financial system from collapsing.

If a public corporation goes bankrupt and does not have enough assets to pay off all creditors:

The stockholders cannot lose more than their investment.

The fact that common stockholders are residual claimant's means:

The stockholders have a claim against the revenue that remains after everyone else is paid.

Checkable deposits have decreased since the 1970s mainly because:

These deposit accounts offer little or no interest so depositors find them to be expensive

Often a bank will require a loan officer to make personal visits on customers with loans outstanding. This is encouraged because:

This is an effective monitoring technique and should reduce moral hazard

One reason customers do not care about the quality of their bank's assets is:

With deposit insurance, there isn't any real reason to care; their deposits are protected even if the bank fails

One reason customers do not care about the quality of their bank's assets is:

With deposit insurance, there isn't any real reason to care; their deposits are protected even if the bank fails.

One characteristic that distinguishes holding period return from the coupon rate, the current yield, and the yield to maturity is:

all of the other returns can be calculated at the time the bond is purchased, but holding

An increase in the nation's wealth, all other factors constant, would cause the:

bond demand curve to shift right.

. In the Governing Council, the decision of what policy to implement is made by A) majority vote of the Executive Board members. B) majority vote of the heads of the National Banks. C) consensus. D) majority vote of all members of the Governing Council.

consensus.

A 10-year Treasury note as a face value of $1,000, price of $1,200, and a 7.5% coupon rate. Based on this information, we know the:

coupon payment on this bond is equal to $75

When expected inflation increases, for any given nominal interest rate the:

cost of borrowing decreases and the desire to borrow increases.

If interest rates are expected to fall, bond prices will:

increase due to the demand for bonds increasing.

Suppose that the return on assets other than bonds falls. In the bond market this will result in a(n):

increase in the price of bonds

The relationship between the price and the interest rate for a zero coupon bond is best described as:

inverse.

When the price of a bond is above face value the yield to maturity:

is below the coupon rate.

A decrease in the liquidity of corporate bonds, other things being equal, shifts the demand curve for corporate bonds to the ________ and the demand curve for Treasury bonds shifts to the ________.

left; right

The larger the bond dealer's spread the:

less liquid is the market for that bond.

A bond with default risk will always have a ________ risk premium and an increase in its default risk will ________ the risk premium.

positive; raise

In calculating the current yield for a bond the:

present value of the capital gain/loss is ignored.

When expected inflation increases, for any given nominal interest rate the:

real cost of repayment for bond issuers decreases.

An increase in the riskiness of corporate bonds will ________ the price of corporate bonds and ________ the price of Treasury bonds, everything else held constant.

reduce; increase

In actual practice, short-term interest rates and long-term interest rates usually move together; this is the major shortcoming of the

segmented markets theory.

The difference in the prices of a zero-coupon bond and a coupon bond with the same face value and maturity date is simply:

the present value of the coupon payments.

The bid price for a bond quote is:

the price at which the bond dealer is willing to purchase the bond.

Suppose that general business conditions improve, and at the same time, wealth increases. Based on this information, we know that:

the quantity of bonds increases.

As bond prices increase:

the quantity of bonds supplied increases

with a call option that is described as in the money:

C. the market price of the stock is above the strike price

considering a put option; if the price of the underlying asset increases:

C. the value of the option decreases

Options are popular because of all of the following except:

C. they present a tool to limit losses but also limit gains

A pension fund manager who plans on purchasing bonds in the future:

C. will take the long position in a futures contract

The government's role of lender of last resort:

Was extended to nonbank intermediaries through the Fed's emergency lending authority in the financial crisis of 2007-2009.

If an investment will return $1,500 half of the time and $700 half of the time, the expected value of the investment is: a. $1,250 b. $1,050 c. $1,100 d. $2,200

$1,100

Which of the following stock price index is a price-weighted index? a. Dow Jones Industrial Average b. Standard & Poor's 500 Index c. Nasdaq d. Wilshire 5000

A. Dow Jones Industrial Average

15. Futures markets and derivatives contribute to economic growth by: a. decreasing speculation. b. increasing the risk-taking capacity of the economy. c. deterring the transfer of risk. d. forcing people to accept the risk their decisions create.

B

Consider a one-year corporate bond that has a 20% probability of default. The payoff on the bond is $2,000 if the corporation does not default. The interest rate is 10%. If buyers of this bond are risk-neutral, this bond will sell for:

$1,454.54

Which of the following best expresses the formula for determining the price of a U.S. Treasury bill that matures n periods from now per $100 of face value when the interest rate is i?

$100/(1 + i)n

What is the future value of $1000 after six months earning 12% annually? a. $1050.00 b. $1060.00 c. $1120.00 d. $1058.30

$1058.30

A $500 investment has the following payoff frequency: half of the time it will pay $350 and the other half of the time it will pay $900. Its value at risk respectively are: a. $150 b. $275 c. $350 d. $500

$150

If a bank has $100,000 of checkable deposits, a required reserve ratio of 20%, and it holds $40,000 in reserves, then the maximum deposit outflow it can sustain without altering its balance sheet is:

$25,000

An investment will pay $2,000 half of the time and $1,400 half of the time. The standard deviation for this investment is: a. $90,000 b. $300 c. $1,700 d. $30

$300

If the annual interest rate is 5%(.05), the price of a six-month Treasury bill would be:

$97.59

If the annual interest rate is 5% (.05), the price of a three-month Treasury bill would be:

$98.79

If the annual interest rate is 5%(.05), the price of a three-month Treasury bill would be:

$98.79

If the annual interest rate is 5%, the price of a three-month Treasury bill would be: (Hint: T-Bills have $100 face value)

$98.79

How many members belong to the Board of Directors for each of the Reserve Banks of the Fed?

9 (Ch. 16)

Consider a two-year, 8% coupon bond with a $1,500 face value that is originally purchased for $1,490. Calculate the holding period return of this bond if it is sold after one year at a price of $1,505.

9.06%

Number of treasury notes needed to hedge a bond portfolio (Calculation)

= ((Duration of bond portfolio * Value of bond portfolio) / (Duration of futures contract * Value of single futures contract))

Number of futures contracts needed to hedge a stock portfolio fully (Calculation)

= ((Value of stock portfolio / Value of stock index futures contract) * (Beta stock portfolio / Beta futures contract))

Basis (Calculation)

= Cash price - Futures Price

Duration of futures contract (Calculation)

= Duration of underlying instrument + Time remaining until contract maturity

Spot futures parity (calculation) in multiple periods

= Spot price*(1+r - cash flows if applicable)^t

Spot futures parity (calculation) in one period

= Spot price*(1+r)^t - cash flows if applicable

10. There is a futures contract for the purchase of 1,000 bushels of corn at $3.00 per bushel. At the end of the day when the market price of corn falls to $2.50: a. the buyer (long position) needs to transfer $500 to the seller (short position). b. the seller (long position) needs to transfer $500 to the buyer (short position). c. nothing happens since marked to market adjustments only occur if the market price rises above the contract price. d. nothing happened since no funds are transferred until the settlement date.

A

11. Which one of the following terms is defined as an option that would have a positive payoff if exercised now? A. in-the-money option B. out-of-the-money option C. straddle D. crossed option E. cash-settled

A

12. Financial intermediaries, through their ability to lower transaction costs: A. Allow people to specialize in what they do at the lowest opportunity cost B. Decrease the efficiency of an economy C. Allow for people to be more self-sufficient D. Make collecting and processing information easier

A

13. The fact that a financial intermediary can hire a lawyer to write one contract that works for many customers is an example of: A. Economies of scale B. The law of diminishing marginal returns C. The law of increasing opportunity cost D. The law of demand

A

18. Sue sells a futures contract for U.S. Treasury bonds and on the settlement date the interest rate on U.S. Treasury bonds is lower than Sue expected. Sue will have: a. lost money on her short position. b. gained money on her long position. c. gained money on her short position. d. lost money on her long position.

A

18. The reduction in transaction costs provided by financial intermediaries benefit: A. Small borrowers and small savers B. Large borrowers but not small savers C. Society in the net, but small savers bear much of the cost D. Small borrowers but not small savers

A

20. Kris implemented an option trading strategy consisting of two call options. This strategy is known as which one of the following? A. spread B. straddle C. split D. combination E. counteraction

A

24. An individual who neither uses nor produces a commodity but sells a futures contract for the asset is: a. speculating that the price of the commodity is going to fall. b. speculating that the price of the commodity is going to increase. c. hedging trying to transfer risk. d. using arbitrage to earn profits without taking a risk.

A

24. When a bank takes savings from many small savers and lends it to many borrowers, the bank: A. Decreases the risk to savers through diversification B. Increases the risk to borrowers through high transaction costs C. Decreases the risk to savers through economies of scale D. Decreases the return to savers and increases the cost to borrowers

A

26. The option writer is: a. the seller of an option. b. the buyer of an option. c. the underlying asset of the option. d. the individual who obtains the rights.

A

30. The change in the option symbol quotation system was driven by which of the following? I. Advances in technology II. Increase in the number and type of option products III. Difficulty in applying the old system to NASDAQ stocks IV. Difficulty in applying the system to complicated option products A. I, II, III, and IV B. II, III, and IV C. I, II, and IV D. II and III E. I, II and III

A

32. Asymmetric information poses two important obstacles to the smooth flow of funds from savers to investors. They are: A. Adverse selection, which arises before the transaction occurs, and moral hazard, which occurs after the transaction B. Moral hazard, which arises before the transaction occurs, and adverse selection, which occurs after the transaction C. Adverse selection and moral hazard, both of which occur after the transaction D. Adverse selection and moral hazard, both of which occur before the transaction

A

34. The usual situation in banking regarding asymmetric information is: A. Borrowers know more than lenders B. Lenders know more than borrowers C. Borrowers and lenders have the same information D. Lenders and borrowers have perfect information

A

34. Which one of the following options is in-the-money? A. call with a $45 strike and an underlying stock price of $42 B. put with a $35 strike and an underlying stock price of $36 C. call with a $15 strike and an underlying stock price of $15 D. put with a $45 strike and an underlying stock price of $42 E. call with a $30 strike and an underlying stock price of $29

A

37. Mom's Bakery goes out of business due to decreasing sales resulting from the dramatic increase in people on low carbohydrate diets. The decrease in business also results in Mom's defaulting on the loan they have with the bank. This is an example of: A. Lack of perfect information in financial markets B. Asymmetric information in financial markets C. Moral hazard in financial markets D. Symmetric information in financial markets

A

39. Which of the following is not true of adverse selection? A. It exists because information is perfect B. It describes the problem a lender faces in identifying loan applicants as good or bad risk borrowers C. It arises because borrowers have more information than lenders regarding their creditworthiness D. It arises if lenders try to charge an average price to all applicants

A

41. As an option approaches its expiration date, the value of the option approaches: a. the intrinsic value. b. the price of the underlying asset. c. zero. d. infinity.

A

41. In a financial market where information is symmetric: A. The information known by both parties in a transaction is the same B. All information is known by both parties in a transaction C. The ability to obtain information is available to only one party in a transaction D. Information is not free

A

41. The maximum option payoff from: A. writing a put is $0. B. buying a put is $0. C. writing a call is an unlimited profit. D. buying a call is the strike price. E. writing a call is the stock price.

A

43. Which of the following is a problem of adverse selection? A. The lender has a problem of distinguishing good risk from bad risk borrowers B. The lender has a problem determining that the proceeds from a loan are being used as the borrower stated C. A person takes up the hobby of bungee jumping after purchasing health insurance D. Individuals use more medical services as a result of their purchase of a health insurance plan

A

45. Assume we have a stock currently worth $50. We also assume the interest rate is zero, and we can buy options for this stock with a strike price of $50. If the stock can rise or fall by $10 with equal probability over the option period, and the option cannot be exercised until the expiration date, what is the time value of the option? a. $5 b. $10 c. $50 d. $40

A

47. One lesson that Akerlof's Lemons model provides is: A. That for high quality providers to survive they must provide a way that customers can distinguish high quality from low quality B. Low quality will not survive in a market C. People always prefer high quality to low quality goods D. Moral hazard is unavoidable

A

47. Which one of the following is a bull call spread? A. buying a $20 call and selling a $25 call on the same stock B. selling a $20 call and buying a $25 call on the same stock C. buying a $20 call and selling a $15 call on the same stock D. selling a $20 call and buying a $25 put E. buying a $20 call and selling a $25 put

A

48. Considering a call option, if the price of the underlying asset decreases: a. the intrinsic value of the option decreases if it is above zero. b. the intrinsic value of the option increases if it is above zero. c. the strike price decreases. d. the value of the option increases.

A

49. The interest rates charged on most credit cards is: A. High due to the problem of adverse selection B. High because Visa and MasterCard have a virtual monopoly on this business C. High due to diseconomies of scale that exist in this market D. Lower than they should be given the problem of adverse selection

A

50. We have a stock selling for $90.00. There is a put option for this stock with a strike price of $85 and an option price of $1.20: a. the intrinsic value of this option is $0.00 and the time value of the option is $1.20. b. the intrinsic value of this option is $90.00 and the time value of the option is $1.20. c. the intrinsic value of this option is -$5.00 and the time value of the option is $1.20. d. you cannot determine the intrinsic value or time value of the option since the strike price is less than the underlying asset price.

A

51. The publication, Consumer's Reports, is one tool designed to treat: A. Adverse selection B. Moral hazard C. The free-rider problem D. Symmetric information

A

52. In the bond market, the assigning of a risk premium is a tool designed to address the problem of: A. Adverse selection B. Information asymmetry C. The free-rider D. Moral hazard

A

54. Considering interest-rate swaps, the swap rate is: a. the benchmark rate plus a premium. b. the rate being offered on U.S. Treasury securities of similar maturities. c. another name for the swap spread. d. a measure of overall risk in the economy.

A

6. The key difference between a forward and a futures contract is: a. a forward contract is customized where a futures contract is not. b. a forward contract is bought and sold on organized exchanges. c. only the forward contracts have settlement dates. d. the amount of time involved.

A

6. Which one of the following is defined as an option that can only be exercised at expiration? A. European style option B. in-the-money option C. out-of-the-money option D. American style option E. derivative option

A

65. The scandals involving Enron, World Com, Global Crossing and other large firms: A. Are examples of asymmetric information and have led, at least temporarily, to a less well functioning stock market B. Is what should have been expected on the part of investors, that is why there is a risk premium C. Have resulted in a cry for less government regulation of public corporations D. Demonstrate that the government should be responsible for collecting and distributing financial information on firms

A

72. Gerold purchased 3 put option contracts at an option premium of $0.95 and a strike price of $40. At expiration, the stock price was $42.25 per share. What is his percentage return? A. -100 percent B. 0 percent C. 15.79 percent D. 21.62 percent E. 31.58 percent

A

79. You own 4 put option contracts on ALZ stock. The contracts have a $17.50 strike price and you paid an option premium of $0.40. What is the break-even stock price? A. $17.10 B. $17.30 C. $17.50 D. $17.70 E. $17.90

A

80. The fact that many companies employ supervisors to oversee the actions of workers is a way to treat: A. Moral hazard B. Adverse selection C. The law of diminishing returns D. The free-rider problem

A

82. A bank usually treats the moral hazard problem by using all of the following, except: A. Not making loans B. Requiring collateral C. Requiring down payments D. Restrictive covenants

A

89. A European 3-month call has a strike price of $35. The stock price is currently $34.30. What is the lower price bound on this call? A. $0.00 B. $0.30 C. $0.70 D. $1.00 E. $1.30

A

9. If financial intermediaries did not have the ability to pool the resources of small savers: A. Borrowers needing large amounts of money would find it more costly to obtain the funds B. The economy would grow faster C. People would likely save more D. The risk associated with lending would decrease

A

90. Large companies seeking to raise funds often will use a well-known investment bank because: A. The investment bank's reputation identifies the company as being credit worthy B. They are required to by government regulation C. The investment bank is paying the company for the publicity and goodwill it will generate D. This minimizes moral hazard

A

91. A 4-month, $25 call option on Teller stock has an option premium of $0.25. The 4-month, $25 put option has an option premium of $0.80. The risk-free rate is 3 percent. The options are European-style. What is the price of Teller stock? A. $24.20 B. $24.53 C. $24.62 D. $25.97 E. $26.08

A

A $1,000 face value bond purchased for $965.00, with an annual coupon of $60, and 20 years to maturity has a: A. current yield equal to 6.22%. B. current yield equal to 6.00%. C. coupon rate equal to 6.22%. D. yield to maturity and current yield equal to 6.00%.

A

A bank faces foreign exchange risk when: A. it has assets denominated in one currency and liabilities in another. B. it lends to foreign borrowers because they are less likely to repay a U.S. bank. C. foreign governments restrict dollar-denominated payments. D. it has branches in other countries.

A

A bank that does not want to hold a lot of excess reserves but wants to manage liquidity risk is likely to: A. hold a lot in highly liquid securities. B. make sure that most of its assets are in small business loans. C. have a high ratio of loans to securities. D. limit withdrawals by customers.

A

A bank's Return on Equity (ROE) is calculated by: A. dividing the bank's net profit after taxes by the bank's capital. B. dividing the banks liabilities by the bank's capital. C. taking the bank's assets plus the net profit after taxes and dividing this sum by the bank's capital. D. dividing the bank's net profit after taxes by the sum of the bank's assets and its liabilities.

A

A bank's reserves include: A. vault cash. B. U.S. Treasury Securities. C. the bank's loan portfolio. D. U.S. Treasury bills and vault cash.

A

A category of assets for banks is cash items in the process of collection. This is: A. uncollected funds the bank is due to receive from the clearing of checks. B. currency the bank is due from the Treasury. C. late fees the bank is owed from loan payments that were not made on time. D. payments from the FDIC insurance fund due the bank.

A

A decrease in expected inflation for any given nominal interest rate will cause: A. bond prices to increase and interest rates to decrease. B. bond prices to decrease and interest rates to increase. C. the bond demand curve to shift to the left. D. the bond supply curve to shift to the left.

A

A decrease in the nation's wealth, all other factors constant, would cause: A. the bond demand curve to shift left. B. bond prices to rise. C. interest rates to decrease. D. the bond supply curve to shift left.

A

A futures contract is an example of: A. a derivative instrument. B. an instrument used solely by financial institutions. C. a high-risk security that will only have value if certain events occur. D. a contract that is traded but is not a financial instrument.

A

A rumor starts that says a bank has suffered significant losses and may not be able to honor its promises to depositors. This causes most of the depositors to line up in front of the bank the next morning wanting to withdraw their deposits. This is an example of: A. liquidity risk. B. operational risk. C. interest rate risk. D. credit risk.

A

An increase in expected inflation for any given nominal interest rate will cause: A. the real return to bondholders to decrease. B. a movement down the bond demand curve, but no change in the bond demand curve. C. the bond demand curve to shift right. D. the price of bonds to increase.

A

An insurance company is an example of a financial institution that: A. transfers risk. B. acts as a broker. C. serves as a depository institution. D. sells derivative securities.

A

As bond prices increase: A. the quantity of bonds supplied increases. B. the quantity of bonds supplied decreases. C. the quantity of bonds demanded increases. D. yields increases.

A

As general business conditions deteriorate, all other factors constant: A. the bond supply curve will shift left. B. there will be a movement down the existing bond supply curve. C. the bond demand curve shifts left. D. the price of bonds will decrease.

A

Asymmetric information in financial markets is a potential problem usually resulting from: A. borrowers having more information than the lenders. B. lenders having more information than borrowers. C. the fact that people are basically dishonest. D. the uncertainty about Federal Reserve monetary policy.

A

Capital is the cushion banks have against: A. sudden drops in the value of their assets. B. an unexpected decrease in liabilities. C. liquidity risk. D. moral hazard.

A

Commercial banks differ from credit unions in the following way: A. credit unions focus on consumer loans while commercial banks primarily make loans to businesses. B. credit unions make loans and accept deposits while commercial banks just make loans. C. commercial banks cannot make auto loans to individuals, just to businesses while credit unions can do both. D. credit unions do not have to hold reserves while commercial banks do.

A

Commercial banks increased their involvement in mortgages over the years due to: A. the ability to securitize mortgages which made them more liquid. B. the demands of regulators. C. the increase in commercial loans demanded due to the development of the commercial paper market. D. the reduced risk of borrowers' defaulting on mortgage loans.

A

Consider a zero-coupon bond with a $1,100 payment in one year. Suppose the interest rate decreases from 10% to 8%. The price of this bond: A. increases from $1,000 to $1,018. B. increases from $1,000 to $1,375. C. decreases from $110 to $88. D. decreases from $1,210 to $1,188.

A

Considering a bank's balance sheet, which of the following statements is false? A. Total Bank Assets + Total Bank Liabilities = Total Bank Capital B. Total Bank Assets = Total Bank Liabilities + Total Bank Capital C. Total Bank Liabilities = Total Bank Assets - Total Bank Capital D. Total Bank Capital = Total Bank Assets - Total Bank Liabilities

A

Considering the balance sheet for all commercial banks in the U.S., the net worth of banks is: A. about 12% of total liabilities. B. about 5 times total assets. C. about the same as total assets. D. about 4 times total liabilities.

A

Considering the value of a financial instrument, the more likely it is the payment will be made: A. the more valuable the financial instrument. B. the less valuable is the instrument because risk is lower. C. the less valuable is the financial instrument because it is highly liquid. D. the greater the uncertainty; therefore the less valuable is the financial instrument.

A

Default risk is the risk associated with: A. the bond issuer not being able to make the promised payments. B. the illiquidity associated with small issues. C. the effect on bond prices caused by changes in market rates of interest. D. changes in the expected inflation rate.

A

Everything else equal, if the ratio of bank assets to bank capital decreases, the bank's return on equity should: A. decrease. B. remain constant. C. increase. D. cannot be determined from the information provided.

A

Financial instruments and money share which of the following characteristics? A. Both can function as a means of payment and a store of value. B. Both can function as a store of value and allow for trading of risk. C. Both can function by acting as a means of payment and allow for trading of risk. D. Both can function as a store of value even though they do not allow for trading of risk.

A

Financial instruments are used to channel funds from: A. savers to borrowers in financial markets and via financial institutions. B. savers to borrowers in financial markets but not through financial institutions. C. borrowers to savers in financial markets but not through financial institutions. D. borrowers to savers through financial institutions, but not in financial markets.

A

Financial instruments used primarily as stores of value would not include: A. a car insurance policy. B. a U.S. Treasury bond. C. shares of General Motors stock. D. a home mortgage.

A

Financial intermediaries include each of the following, except: A. the New York Stock Exchange. B. credit unions. C. savings banks. D. commercial banks.

A

For every $100 in assets, a bank has $30 in interest-rate sensitive assets, and the other $70 in non-interest-rate sensitive assets. The same bank has $60 for every $100 in liabilities in interest-rate sensitive liabilities, the other $40 are in liabilities that are not interest-rate sensitive. If the interest rate on assets decreases from 6 to 5 percent, and the interest rate on liabilities decreases from 4 to 3 percent, the impact on the bank's profits per $100 of assets will be: A. a reduction of $0.30. B. an increase of $0.30. C. a reduction of $3.00. D. zero since the interest rates on assets and liabilities fell by the same amount.

A

If a bank's return on equity remains constant, but the ratio of bank assets to bank capital decreases: A. the bank's return on assets must have increased. B. the bank's return on assets must have decreased. C. the bank's assets and capital must have increased by the same percent. D. the bank must be unprofitable.

A

If the annual interest rate is 5%(.05), the price of a three-month Treasury bill would be: A. $98.79 B. $95.00 C. $98.75 D. $97.59

A

If the quantity of bonds demanded exceeds the quantity of bonds supplied, bond prices: A. would rise and yields would fall. B. would fall and yields would increase. C. will rise and yields will remain constant. D. will rise and yields would increase.

A

In calculating the current yield for a bond the: A. coupon payment and purchase price is all that is needed. B. present value of the capital gain/loss is ignored. C. present value of the final payment is the only important consideration. D. present value of the coupon payments is the only important consideration.

A

Juan purchases automobile insurance; the insurance contract is a: A. financial instrument. B. form of money. C. transfer of risk from the insurance company to Juan. D. financial intermediary.

A

Money Center Banks differ from community banks in all of the following ways except: A. they are usually much smaller. B. they obtain their funds primarily through borrowing and not by deposits. C. they are a much smaller percentage of the total number of banks. D. they are actively engaged in the money market.

A

One characteristic that distinguishes holding period return from the coupon rate, the current yield, and the yield to maturity is: A. all of the other returns can be calculated at the time the bond is purchased, but holding period return cannot. B. holding period return will always be the highest return. C. holding period return will usually be less than the other returns. D. only the holding period return includes the capital gain/loss.

A

One of the lessons from the 2007-2009 financial crisis regarding the management of risk by financial institution is that: A. many banks lacked real-time information that would allow them to assess their various risk exposures at the bank-wide level. B. some banks, especially large ones, overestimated the trading risk associated with mortgage backed securities. C. banks were holding too much capital as a protection against market risk. D. many of the usual mechanisms for managing liquidity risk actually worked pretty well.

A

Regulators require a bank to hold some of its assets as reserves mainly to address: A. liquidity risk. B. trading risk. C. credit risk. D. operational risk.

A

Roles served by financial markets include the following, except: A. eliminating risk. B. providing liquidity. C. pooling and communicating information. D. sharing of risk.

A

Suppose a particular bank is very large in terms of assets, and makes consumer and residential loans as well as commercial and industrial loan. The bank is probably a: A. regional or super-regional bank. B. money center bank. C. community bank. D. savings bank.

A

The bond dealer's spread is: A. the asking price less the bid price. B. the difference between the current yield and the yield to maturity. C. the bid price less the asking price. D. usually negative; the dealer makes a profit holding the bonds.

A

The bond demand curve slopes downward because: A. at lower prices the reward for holding the bond increases. B. as bond prices fall so do yields. C. as bond prices fall bonds are less attractive. D. as bond prices rise yields increase.

A

The fact that a bank's assets tend to be long-term while its liabilities are short-term creates: A. interest-rate risk. B. credit risk. C. lower risk for the bank, this is why they follow this strategy. D. trading risk.

A

The larger the bond dealer's spread the: A. less liquid is the market for that bond. B. greater is the coupon rate for that bond. C. more liquid is the market for that bond. D. less risk there is for the dealer to hold that bond.

A

The market for bonds is initially described by the supply of bonds - S0, and the demand for bonds - D0, with the equilibrium price and quantity being P0 and Q0. If the federal government were to offer larger tax breaks on the purchase of new equipment for businesses, all other factors constant, we would expect to see: A. Bond supply curve to shift to S1 B. Bond demand curve to shift to D1 C. Bond supply curve to shift to S2 D. Bond demand curve to shift to D2

A

The weighted average difference between the interest received on assets and the interest rate paid for liabilities for a bank is the bank's: A. interest rate spread. B. net interest margin. C. net interest income. D. return on equity.

A

When the price of a bond is above face value the yield to maturity: A. is below the coupon rate. B. will be above the coupon rate. C. will equal the current yield. D. will equal the coupon rate.

A

Which of the following bank assets would be categorized as secondary reserves? A. U.S. Treasury bills B. Cash C. Mortgage loans D. Deposits at the Federal Reserve

A

Which of the following is not considered to be a shadow bank? A. Credit unions B. Brokerages C. Insurers D. Money-market mutual funds

A

Which of the following statements is most correct? A. All banks are financial intermediaries, but not all financial intermediaries are banks. B. Financial intermediaries must be public corporations. C. All financial intermediaries are insurance companies. D. Financial intermediaries are government agencies.

A

5. Describe the condition that would have a call option in the money. Now describe the condition that has a put option out of the money.

A call option will be in the money when the strike price is below the spot or current market price. The option holder has the right to call the asset away from the option writer at a price below what the asset could be sold for on the spot market. A put option is out of the money when the strike price is below the spot or market price. Here the option holder has the right to put (sell) the asset to the option writer at a predetermined (strike) price. If the strike price is below the market price the option holder would be better off selling the asset on the spot market versus selling it to the option writer.

2. Explain why a forward contract may actually carry more risk than a futures contract.

A forward contract is a private agreement between two parties that is customized for the two parties. As a result, the high degree of customization makes them very difficult if not impossible to resell. Futures contracts on the other hand are highly standardized. The high level of standardization allows them to be bought and sold on organized exchanges, which increases their liquidity and reduces risk. In addition, forward contracts are private agreements and thus carry greater default risk. Futures contracts are usually settled through clearing corporations where procedures such as mark to market greatly reduce default risk.

One reason that financial regulations restrict the assets that banks can own is to: A. combat the moral hazard that government safety nets provide. B. limit the growth rate of banks. C. prevent banks from being too profitable. D. keep banks from spending lavishly on perks for executives.

A. Combat the moral hazard that government safety nets provide

Under the purchase-and-assumption method of dealing with a failed bank, the FDIC: A. finds another bank to take over the insolvent bank. B. takes over the day to day management of the bank. C. sells the failed bank to the Federal Reserve. D. sells off the profitable loans of the failed bank in an open auction.

A. Finds another bank to take over the insolvent bank

Sue sells a futures contract for US Treasury bonds and on the settlement date the interest rate on US treasury bonds is lower than Sue expected. Sue will have:

A. Lost money on her short position

The best way for a government to stop the failure of one bank from turning into a bank panic is to: A. make sure solvent institutions can meet the withdrawal demands of depositors. B. declare a bank holiday until solvent banks can acquire adequate liquidity. C. limit the withdrawals of depositors. D. provide zero-interest rate loans to all banks regardless of net worth.

A. Make sure solvent institutions can meet the withdrawal demands of depositors

If each company that made up the DJIA increased the number of their shares outstanding by 10%, but the share prices did not change, the value of the index would:

A. Not change

A long-standing goal of financial regulators has been to: A. prevent banks from growing too big and powerful. B. minimize the competition that banks face. C. encourage banks to grow as large as possible. D. discourage small rural banks.

A. Prevent banks from growing too big and powerful

Credit Unions are regulated by a combination of agencies which includes: A. state authorities. B. The Federal Reserve. C. The Federal Deposit Insurance Corporation. D. The Office of the Comptroller of the Currency.

A. State authorities

When the Federal Reserve was unable to stem the bank panics of the 1930s, Congress responded by: A. taking over the lender of last resort function and assigning this function to the U.S. Treasury. B. ordering the printing of tens of billions of dollars of additional currency. C. creating the FDIC and offering deposit insurance. D. declaring a bank holiday and closing banks for 30 days.

A. Taking over the lender of last resort functions and assigning this function to the U.S. treasury

Savings banks and savings and loans are regulated by a combination of agencies which includes all of the following except: A. The Federal Reserve System. B. The Comptroller of the Currency. C. The Federal Deposit Insurance Corporation. D. state authorities.

A. The Federal Reserve System

If a bank's return on equity remains constant, but the ratio of bank assets to bank capital decreases: A. The bank's return on assets must have increased b. The bank's return on assets must have decreased c. The bank's assets and capital must have increased by the same percent d. The bank must be unprofitable

A. The bank's return on assets must have increased

Contagion is: A. the failure of one bank spreading to other banks through depositors withdrawing of funds. B. the phenomenon that if one bank loan defaults it will cause other bank loans to default. C. the rapid contraction of investment spending that occurs when interest rates are increased by the Federal Reserve. D. the rapid inflation that results from the printing of money.

A. The failure of one bank spreading to other banks through depositors withdrawing of funds.

Considering the S&P 500 index, if each company's stock price increased by 10%:

A. The weights in the index would remain the same

The existence of a lender of last resort creates moral hazard for bank managers because: A. they have an incentive to take too much risk in their operations. B. officials are likely to undervalue the bank's portfolio of assets. C. they are less likely to apply for a direct loan from the central bank. D. banks seek loans from the central bank only after exploring other options.

A. They have incentive to take too much risk in their operations

The reasons for the government to get involved in the financial system include each of the following, except: A. to protect the bank's monopoly position. B. to protect investors. C. to ensure the stability of the financial system. D. to protect bank customers from monopolistic exploitation.

A. To protect the bank's monopoly position

A moral hazard situation arises in the lender of last resort function because: A. a central bank finds it difficult to distinguish illiquid from insolvent banks. B. a central bank usually will only make a loan to a bank after it becomes insolvent. C. a central bank usually undervalues the assets of a bank in a crisis. D. the central bank is the first place a bank facing a crisis will turn.

A. a central bank finds it difficult to distinguish illiquid from insolvent banks

The key difference between a forward and futures contract is:

A. a forward contract is customized where a futures contract is not

The Nasdaq Composite Index is:

A. a value-weighted index

Governments employ three strategies to contain the risks created by government safety nets. These include each of the following, except: A. government supervision. B. an excise tax on bank profits. C. government regulation. D. formal bank examination.

A. an excise tax on bank profits

There's a call option written for 100 shares of GM stock for $85.00 a share, prior to the third Friday of October 2017: the option writer:

A. has the requirement to sell 100 shares of GM for $85 a share on or before the third Friday of October 2017 if the option holder wants to exercise the option

An investor who purchases a call option is:

A. highly leveraged for a gain but is limited in losses

The reason that a run on a single bank can turn into a bank panic that threatens the entire financial system is: A. information asymmetries. B. moral hazard. C. the lack of regulation. D. the increased reliance on web-based funds transfers.

A. information asymmetries

The process of marking to market:

A. is done by the clearing corporation to reduce risk in futures contracts

The moral hazard problem caused by government safety nets: A. is greater for larger banks. B. is greater for smaller banks. C. is pretty constant across banks of all sizes. D. only exists for banks with high leverage ratios.

A. is greater for larger banks

If the government did not offer the too-big-to-fail safety net: A. large banks would be more disciplined by the potential loss of large corporate accounts. B. the moral hazard problem of insuring large banks would increase. C. the moral hazard problem of insuring large banks would not be affected. D. the FDIC deposit insurance limits would have to be raised.

A. large banks would be more disciplined by the potential loss of large corporate accounts

as an option approaches its expiration date, the value of the option approaches:

A. the intrinsic value

A put option described as out of the money would find:

A. the strike price is below the market price of the stock

Long hedge

Adding a long futures position to a short position in the underlying asset

Short hedge

Adding a short futures position to a long position in the underlying asset

Asymmetric information poses two important obstacles to the smooth flow of funds from savers to investors. They are:

Adverse selection, which arises before the transaction occurs, and moral hazard, which occurs after the transaction.

Providing stock options to corporate managers was an idea designed to:

Align managers' interest with the stockholders' interest

Providing stock options to corporate managers was an idea designed to:

Align managers' interest with the stockholders' interest.

The primary difference among various kinds of depository institutions is in the composition of their loan portfolios. Agree or disagree? Explain.

All depository institutions have deposits as their sources of funds and use those funds to make loans. There are three basic types of depository institutions: commercial banks, savings institutions, and credit unions. Commercial banks make loans primarily to businesses; saving and loans primarily provide mortgages to individuals; and credit unions specialize in consumer loans. However, the balance sheets of these depository institutions have changed over time and they now compete with each other to a greater degree than in the past. For example, on the deposits side, savings institutions offer types of checking accounts. In terms of the loan portfolios, commercial banks now serve individuals with consumer and residential loans, and savings and loan institutions today engage in a much broader range of financial activities than in the past. Therefore it is safe to say that while the depository institutions do differ from each other in terms of the loan portfolios, the differences will mostly be matters of the proportions of different types of loans each holds.

A T-account represents ________. A) a simplified balance sheet B) asset transformation C) T-bills D) term deposits

Answer: A

A bank is insolvent when ________. A) its liabilities exceed its assets B) its assets exceed its liabilities C) its capital exceeds its liabilities D) its assets increase in value

Answer: A

Bank capital is equal to ________ minus ________. A) total assets; total liabilities B) total liabilities; total assets C) total assets; total reserves D) total liabilities; total borrowings

Answer: A

Banks face the problem of ________ in loan markets because bad credit risks are the ones most likely to seek bank loans. A) adverse selection B) moral hazard C) moral suasion D) intentional fraud

Answer: A

Bruce the Bank Manager can reduce interest rate risk by ________ the duration of the bank's assets to increase their rate sensitivity or, alternatively, ________ the duration of the bank's liabilities. A) shortening; lengthening B) shortening; shortening C) lengthening; lengthening D) lengthening; shortening

Answer: A

Conditions that likely contributed to a credit crunch in 2008 include ________. A) capital shortfalls caused in part by falling real estate prices B) regulated hikes in bank capital requirements C) falling interest rates that raised interest rate risk, causing banks to choose to hold more capital D) increases in reserve requirements

Answer: A

Examples of off-balance-sheet activities include ________. A) loan sales B) extending loans to depositors C) borrowing from other banks D) selling negotiable CDs

Answer: A

Holding all else constant, when a bank receives the funds for a deposited cheque, ________. A) cash items in the process of collection fall by the amount of the cheque B) bank assets increase by the amount of the cheque C) bank liabilities decrease by the amount of the cheque D) bank reserves increase by the amount of desired reserves

Answer: A

If a bank has $10 million of demand deposits, a desired reserve ratio of 10 percent, and it holds $2 million in reserves, then it will not have enough reserves to support a deposit outflow of ________. A) $1.2 million B) $1.1 million C) $1 million D) $900,000

Answer: A

If a bank has $200,000 of demand deposits, a desired reserve ratio of 20 percent, and it holds $80000 in reserves, then the maximum deposit outflow it can sustain without altering its balance sheet is ________. A) $50000 B) $40000 C) $30000 D) $25000

Answer: A

If a bank has excess reserves greater than the amount of a deposit outflow, the outflow will result in equal reductions in ________. A) deposits and reserves B) deposits and loans C) capital and reserves D) capital and loans

Answer: A

If a bank's liabilities are more sensitive to interest rate movements than are its assets, then ________. A) an increase in interest rates will reduce bank profits B) a decrease in interest rates will reduce bank profits C) interest rates changes will not impact bank profits D) an increase in interest rates will increase bank profits

Answer: A

In one sense ________ appears surprising since it means that the bank is not ________ its portfolio of loans and thus is exposing itself to more risk. A) specialization in lending; diversifying B) specialization in lending; rationing C) credit rationing; diversifying D) screening; rationing

Answer: A

Long-term customer relationships ________ the cost of information collection and make it easier to ________ credit risks. A) reduce; screen B) increase; screen C) reduce; increase D) increase; increase

Answer: A

Modern liability management has resulted in ________. A) increased sales of certificates of deposits to raise funds B) increase importance of deposits as a source of funds C) reduced borrowing by banks in the overnight loan market D) failure by banks to coordinate management of assets and liabilities

Answer: A

Most of a bank's operating income results from ________. A) interest on assets B) service charges on deposit accounts C) off-balance-sheet activities D) fees from standby lines of credit

Answer: A

Net profit after taxes per dollar of assets is a basic measure of bank profitability called ________. A) return on assets B) return on capital C) return on equity D) return on investment

Answer: A

One of the problems in conducting a duration gap analysis is that the duration gap is calculated assuming that interest rates for all maturities are the same. That means that the yield curve is ________. A) flat B) slightly upward sloping C) steeply upward sloping D) downward sloping

Answer: A

One way for banks to reduce the principal-agent problems associated with trading activities is to ________. A) set limits on the total amount of a traders' transactions B) make sure that the person conducting the trades is also the person responsible for recording the transactions C) encourage traders to take on more risk if the potential rewards are higher D) reduce the regulations on the traders so that they have more flexibility in conducting trades

Answer: A

Property promised to the lender as compensation if the borrower defaults is called ________. A) collateral B) deductibles C) restrictive covenants D) contingencies

Answer: A

Secondary reserves are so called because ________. A) they can be converted into cash with low transactions costs B) they are not easily converted into cash, and are, therefore, of secondary importance to banking firms C) 50 percent of these assets count toward meeting desired reserves D) they rank second to bank vault cash in importance of bank holdings

Answer: A

To reduce moral hazard problems, banks include restrictive covenants in loan contracts. In order for these restrictive covenants to be effective, banks must also ________. A) monitor and enforce them B) be willing to rewrite the contract if the borrower cannot comply with the restrictions C) trust the borrower to do the right thing D) be prepared to extend the deadline when the borrower needs more time to comply

Answer: A

When $1 million is deposited at a bank, the desired reserve ratio is 20 percent, and the bank chooses not to make any loans but to hold excess reserves instead, then, in the bank's final balance sheet, ________. A) the assets at the bank increase by $1 million B) the liabilities of the bank decrease by $1 million C) reserves increase by $200,000 D) liabilities increase by $200,000

Answer: A

When a $10 cheque written on the First National Bank is deposited in an account at CIBC, then ________. A) the reserves of the First National Bank decrease by $10 B) the reserves of the First National Bank increase by $10 C) the reserves of CIBC decrease by $10 D) the assets of CIBC decrease by $10

Answer: A

When a bank suspects that a $1 million loan might prove to be bad debt that will have to be written off in the future the bank ________. A) can set aside $1 million of its earnings in its loan loss reserves account B) reduces its reported earnings by $1, even though it has not yet actually lost the $1 million C) reduces its assets immediately by $1 million, even though it has not yet lost the $1 million D) reduces its reserves by $1 million, so that they can use those funds later

Answer: A

When a new depositor opens a chequing account at the First National Bank, the bank's assets ________ and its liabilities ________. A) increase; increase B) increase; decrease C) decrease; increase D) decrease; decrease

Answer: A

When banks calculate the losses the institution would incur if an unusual combination of bad events happened, the bank is using the ________ approach. A) stress-test B) value-at-risk C) trading-loss D) maximum value

Answer: A

Which of the following are bank assets? A) The building owned by the bank B) A discount loan C) A negotiable CD D) A customer's chequing account

Answer: A

Which of the following are reported as liabilities on a bank's balance sheet? A) Advances B) Reserves C) Securities D) Loans

Answer: A

Which of the following has not resulted from more active liability management on the part of banks? A) Increased bank holdings of cash items B) Aggressive targeting of goals for asset growth by banks C) Increased use of negotiable CDs to raise funds D) An increased proportion of bank assets held in loans

Answer: A

Which of the following statements is true? A) Chequable deposits are payable on demand. B) Chequable deposits do not include notice deposits. C) Chequable deposits are the primary source of bank funds. D) Chequable deposits are chequable deposits that pay interest.

Answer: A

Which of the following statements most accurately describes the task of bank asset management? A) Banks seek the highest returns possible subject to minimizing risk and making adequate provisions for liquidity. B) Banks seek to have the highest liquidity possible subject to earning a positive rate of return on their operations. C) Banks seek to prevent bank failure at all cost; since a failed bank earns no profit, liquidity needs supersede the desire for profits. D) Banks seek to acquire funds in the least costly way.

Answer: A

A bank will want to hold more excess reserves (everything else equal) when ________. A) it expects to have deposit inflows in the near future B) brokerage commissions on selling bonds increase C) the cost of selling loans falls D) the discount rate decreases

Answer: B

A bank with insufficient reserves can increase its reserves by ________. A) lending overnight funds B) calling in loans C) buying short-term securities D) buying provincial bonds

Answer: B

All else the same, if a bank's liabilities are more sensitive to interest rate fluctuations than are its assets, then ________ in interest rates will ________ bank profits. A) an increase; increase B) an increase; reduce C) a decline; reduce D) a decline; not affect

Answer: B

Asset transformation can be described as ________. A) borrowing long and lending short B) borrowing short and lending long C) borrowing and lending only for the short term D) borrowing and lending for the long term

Answer: B

Bank capital is listed on the ________ side of the bank's balance sheet because it represents a ________ of funds. A) liability; use B) liability; source C) asset; use D) asset; source

Answer: B

Bank loans from the Bank of Canada are called ________ and represent a ________ of funds. A) advances; use B) advances; source C) overnight funds; use D) overnight funds; source

Answer: B

Banks hold excess and secondary reserves to ________. A) reduce the interest-rate risk problem B) provide for deposit outflows C) satisfy margin requirements D) achieve higher earnings than they can with loans

Answer: B

Because of an expected rise in interest rates in the future, a banker will likely ________. A) make long-term rather than short-term loans B) buy short-term rather than long-term bonds C) buy long-term rather than short-term bonds D) make either short or long-term loans; expectations of future interest rates are irrelevant

Answer: B

Duration analysis involves comparing the average duration of the bank's ________ to the average duration of its ________. A) securities portfolio; non-deposit liabilities B) assets; liabilities C) loan portfolio; deposit liabilities D) assets; deposit liabilities

Answer: B

For a given return on assets, the lower is bank capital, ________. A) the lower is the return for the owners of the bank B) the higher is the return for the owners of the bank C) the lower is the credit risk for the owners of the bank D) the lower the possibility of bank failure

Answer: B

If a bank has ________ rate-sensitive assets than liabilities, a ________ in interest rates will reduce bank profits, while a ________ in interest rates will raise bank profits. A) more; rise; decline B) more; decline; rise C) fewer; decline; decline D) fewer; rise; rise

Answer: B

If borrowers with the most risky investment projects seek bank loans in higher proportion to those borrowers with the safest investment projects, banks are said to face the problem of ________. A) adverse credit risk B) adverse selection C) moral hazard D) lemon lenders

Answer: B

If interest rates increase from 3% to 4%, a $100,000 10 year bond with a duration of 8 years would ________ in price by approximately ________. A) increase; 7.8% B) decrease; 7.8% C) increase; 9.7% D) decrease; 9.7%

Answer: B

In general, banks make profits by selling ________ liabilities and buying ________ assets. A) long-term; shorter-term B) short-term; longer-term C) illiquid; liquid D) risky; risk-free

Answer: B

In order to reduce the ________ problem in loan markets, bankers collect information from prospective borrowers to screen out the bad credit risks from the good ones. A) moral hazard B) adverse selection C) moral suasion D) adverse lending

Answer: B

Measuring the sensitivity of bank profits to changes in interest rates by multiplying the gap for several maturity subintervals times the change in the interest rate is called ________. A) basic gap analysis B) the maturity bucket approach to gap analysis C) the segmented maturity approach to gap analysis D) the segmented maturity approach to interest-exposure analysis

Answer: B

Measuring the sensitivity of bank profits to changes in interest rates by multiplying the gap times the change in the interest rate is called ________. A) basic duration analysis B) basic gap analysis C) interest-exposure analysis D) gap-exposure analysis

Answer: B

Provisions in loan contracts that prohibit borrowers from engaging in specified risky activities are called ________. A) proscription bonds B) restrictive covenants C) due-on-sale clauses D) liens

Answer: B

Risk that is related to the uncertainty about interest rate movements is called ________. A) default risk B) interest-rate risk C) the problem of moral hazard D) security risk

Answer: B

The fraction of chequable deposits that banks choose to hold are ________. A) excess reserves B) desired reserves C) vault cash D) total reserves

Answer: B

The principal-agent problem that exists for bank trading activities can be reduced through ________. A) creation of internal controls that combine trading activities with bookkeeping B) creation of internal controls that separate trading activities from bookkeeping C) elimination of regulation of banking D) elimination of internal controls

Answer: B

The quantity defined as interest income minus interest expenses divided by assets is a measure of bank performance known as ________. A) operating income B) net interest margin C) return on assets D) return on equity

Answer: B

Through correspondent banking, large banks provide services to small banks, including ________. A) loan guarantees B) foreign exchange transactions C) issuing stock D) debt reduction

Answer: B

Unanticipated moral hazard contingencies can be reduced by ________. A) screening B) long-term customer relationships C) specialization in lending D) credit rationing

Answer: B

When $1 million is deposited at a bank, the desired reserve ratio is 20 percent, and the bank chooses not to hold any excess reserves but makes loans instead, then, in the bank's final balance sheet, ________. A) the assets at the bank increase by $800,000 B) the liabilities of the bank increase by $1,000,000 C) the liabilities of the bank increase by $800,000 D) reserves increase by $160,000

Answer: B

When you deposit a $50 bill in the New National Bank, ________. A) its liabilities decrease by $50 B) its assets increase by $50 C) its reserves decrease by $50 D) its cash items in the process of collection increase by $50

Answer: B

Which of the following are reported as liabilities on a bank's balance sheet? A) Reserves B) Demand and notice deposits C) Loans D) Deposits with other banks

Answer: B

Which of the following bank assets is the most liquid? A) Consumer loans B) Reserves C) Cash items in process of collection D) Government securities

Answer: B

Which of the following statements is false? A) Chequable deposits are usually the lowest cost source of bank funds. B) Demand deposits are the primary source of bank funds. C) Chequable deposits are payable on demand. D) Chequable deposits include notice deposits.

Answer: B

A bank's commitment to provide a firm with loans up to pre-specified limit at an interest rate that is tied to a market interest rate is called ________. A) an adjustable gap loan B) an adjustable portfolio loan C) loan commitment D) pre-credit loan line

Answer: C

As the costs associated with deposit outflows ________, the banks willingness to hold excess reserves will ________. A) decrease; increase B) increase; decrease C) increase; increase D) decrease; not be affected

Answer: C

Assume a bank has $200 million of assets with a duration of 2.5, and $190 million of liabilities with a duration of 1.05. The duration gap for this bank is ________. A) 0.5 year B) 1 year C) 1.5 years D) 2 years

Answer: C

Banks earn profits by selling ________ with attractive combinations of liquidity, risk, and return, and using the proceeds to buy ________ with a different set of characteristics. A) loans; deposits B) securities; deposits C) liabilities; assets D) assets; liabilities

Answer: C

Because ________ are less liquid for the depositor than ________, they earn higher interest rates. A) money market deposit accounts; time deposits B) chequable deposits; savings account C) savings account; chequable deposits D) savings account; time deposits

Answer: C

For banks, ________. A) return on assets exceeds return on equity B) return on assets equals return on equity C) return on equity exceeds return on assets D) return on equity is another name for net interest margin

Answer: C

Holding large amounts of bank capital helps prevent bank failures because ________. A) it means that the bank has a higher income B) it makes loans easier to sell C) it can be used to absorb the losses resulting from bad loans D) it makes it easier to call in loans

Answer: C

If a bank needs to raise the amount of capital relative to assets, a bank manager might choose to ________. A) buy back bank stock B) pay higher dividends C) sell bank stock D) sell securities the bank owns and put the funds into the reserve account

Answer: C

In general, banks would prefer to acquire funds quickly by ________ rather than ________. A) reducing loans; selling securities B) reducing loans; borrowing from the Bank of Canada C) borrowing from the Bank of Canada; reducing loans D) "calling in" loans; selling securities

Answer: C

In the absence of regulation, banks would probably hold ________. A) too much capital, reducing the efficiency of the payments system B) too much capital, reducing the profitability of banks C) too little capital D) too much capital, making it more difficult to obtain loans

Answer: C

Large-denomination CDs are ________, so that like a bond they can be resold in a ________ market before they mature. A) nonnegotiable; secondary B) nonnegotiable; primary C) negotiable; secondary D) negotiable; primary

Answer: C

Net profit after taxes per dollar of equity capital is a basic measure of bank profitability called ________. A) return on assets B) return on capital C) return on equity D) return on investment

Answer: C

Off-balance sheet activities involving guarantees of securities and back-up credit lines ________. A) have no impact on the risk a bank faces B) does not change the risk a bank faces C) increase the risk a bank faces D) slightly reduce the risk a bank faces

Answer: C

The amount of assets per dollar of equity capital is called the ________. A) asset ratio B) equity ratio C) equity multiplier D) asset multiplier

Answer: C

The most important category of assets on a bank's balance sheet is ________. A) advances B) securities C) loans D) cash items in the process of collection

Answer: C

The share of chequable deposits in total bank liabilities has ________. A) expanded moderately over time B) expanded dramatically over time C) shrunk over time D) remained virtually unchanged since 1960

Answer: C

When a $10 cheque written on the First National Bank is deposited in an account at CIBC, then ________. A) the liabilities of the First National Bank increase by $10 B) the reserves of the First National Bank increase by $ 10 C) the liabilities of CIBC increase by $10 D) the assets of CIBC fall by $10

Answer: C

When banks involved in trading activities attempt to outguess markets, they are ________. A) forecasting B) diversifying C) speculating D) engaging in riskless arbitrage

Answer: C

When you deposit $50 in your account at First National Bank and a $100 cheque you have written on this account is cashed at Chemical Bank, then ________. A) the assets of First National rise by $50 B) the assets of Chemical Bank rise by $50 C) the reserves at First National fall by $50 D) the liabilities at Chemical Bank rise by $50

Answer: C

Which of the following statements are true? A) A bank's assets are its sources of funds. B) A bank's liabilities are its uses of funds. C) A bank's balance sheet shows that total assets equal total liabilities plus capital. D) A bank's balance sheet indicates whether or not the bank is profitable.

Answer: C

________ may antagonize customers and thus can be a very costly way of acquiring funds to meet an unexpected deposit outflow. A) Selling securities B) Selling loans C) Calling in loans D) Selling negotiable CDs

Answer: C

A reason why rogue traders have bankrupt their banks is due to ________. A) the separation of trading activities from the bookkeepers B) stringent supervision of trading activities by bank management C) accounting errors D) a failure to maintain proper internal controls

Answer: D

Assume a bank has $200 million of assets with a duration of 2.5, and $190 million of liabilities with a duration of 1.05. If interest rates increase from 5 percent to 6 percent, the net worth as a percentage of assets would ________ by approximately ________. A) increase; 1.8% B) decrease; 1.8% C) increase; 1.4% D) decrease; 1.4%

Answer: D

Assume a bank has $200 million of assets with a duration of 2.5, and $190 million of liabilities with a duration of 1.05. What is its duration gap? A) 0.95 years B) 1.15 years C) 1.35 years D) 1.50 years

Answer: D

Because ________ are less liquid for the depositor than ________, they earn higher interest rates. A) savings account; time deposits B) money market deposit accounts; time deposits C) money market deposit accounts; savings account D) time deposits; savings account

Answer: D

Because borrowers, once they have a loan, are more likely to invest in high-risk investment projects, banks face the ________. A) adverse selection problem B) lemon problem C) adverse credit risk problem D) moral hazard problem

Answer: D

From the standpoint of ________, specialization in lending is surprising but makes perfect sense when one considers the ________ problem. A) moral hazard; diversification B) diversification; moral hazard C) adverse selection; diversification D) diversification; adverse selection

Answer: D

If a banker expects interest rates to fall in the future, her best strategy for the present is ________. A) to increase the duration of the bank's liabilities B) to buy short-term bonds C) to sell long-term certificates of deposit D) to increase the duration of the bank's assets

Answer: D

If interest rates increase from 3% to 4%, a $100,000 25 year bond with a duration of 21 years would ________ in price by approximately ________. A) increase; 24.3% B) decrease; 24.3% C) increase; 20.4% D) decrease; 20.4%

Answer: D

If interest rates increase from 9 percent to 10 percent, a bank with a duration gap of 2 years would experience a decrease in its net worth of ________. A) 0.9 percent of its assets B) 0.9 percent of its liabilities C) 1.8 percent of its liabilities D) 1.8 percent of its assets

Answer: D

If the First National Bank has a gap equal to a negative $30 million, then a 5 percentage point increase in interest rates will cause profits to ________. A) increase by $15 million B) increase by $1.5 million C) decline by $15 million D) decline by $1.5 million

Answer: D

Of the following, which would be the first choice for a bank facing a reserve deficiency? A) Call in loans B) Borrow from the Bank of Canada C) Sell securities D) Borrow from other banks

Answer: D

The difference of rate-sensitive liabilities and rate-sensitive assets is known as the ________. A) duration B) interest-sensitivity index C) rate-risk index D) gap

Answer: D

The goals of bank asset management include ________. A) maximizing risk B) minimizing liquidity C) lending at high interest rates regardless of risk D) purchasing securities with high returns and low risk

Answer: D

When Jane Brown writes a $100 cheque to her nephew (who lives in another province), Ms. Brown's bank ________ assets of $100 and ________ liabilities of $100. A) gains; gains B) gains; loses C) loses; gains D) loses; loses

Answer: D

When banks offer borrowers smaller loans than they have requested, banks are said to ________. A) shave credit B) rediscount the loan C) raze credit D) ration credit

Answer: D

Which of the following are not reported as assets on a bank's balance sheet? A) Cash items in the process of collection B) Loans C) Securities D) Demand deposits

Answer: D

Which of the following statements is false? A) A bank's assets are its uses of funds. B) A bank issues liabilities to acquire funds. C) The bank's assets provide the bank with income. D) Bank capital is recorded as an asset on the bank balance sheet.

Answer: D

Explain the relationship between return on assets and return on equity. What incentives does this relationship give a bank manager? Is this the desired outcome preferred by regulators? Discuss.

Answer: For a given return on assets, the greater the amount of capital, the lower is the return on equity. Bank managers who seek to increase the return on equity must increase the asset base, purchase riskier assets, or reduce the amount of capital by paying dividends or buying back stock. Regulators (and depositors) prefer higher capital for bank safety. Managers typically prefer lower equity than regulators, resulting in regulatory bank capital requirements.

How can specializing in lending help to reduce the adverse selection problem in lending?

Answer: Reducing the adverse selection problem requires the banks to acquire information to screen bad credit risks from good credit risks. It is easier for banks to obtain information about local businesses. Also if the bank lends to firms in a few specific industries they will become more knowledgeable about those industries and a better judge of creditworthiness in those industries.

What are the main items in a bank's asset side of the balance sheet? Discuss them briefly.

Answer: The students must answer that the main items in a bank's asset side of the balance sheet are: - Cash reserves: vault cash, settlement balances - Deposits at other banks: interbank deposits - Cash items in process of collection: items in transit or bank float - Securities: government of Canada, provincial and municipal, and other securities - Loans: commercial, industrial, real estate - Fixed and other assets: physical capital, etc.

What is a loan sale and how does it work?

Answer: The students must explain that the loan sale is an off-balance-sheet activity that has grown in importance in recent years and it generates income for banks. A loan sale is also called a secondary loan participation and involves a contract that sells all or part of the cash stream from a specific loan and thereby it removes it from the bank's balance sheet. Banks earn profit by selling the loans for an amount slightly higher than the original loan amount. The high interest rate for these loans makes them attractive and institutions are willing to buy them at the higher price which means that they earn a slightly lower interest rate than the original loan usually on the order of 0.15 percentage points.

The existence of deposit insurance can increase the likelihood that depositors will need deposit protection, as banks with deposit insurance:

Are likely to take on greater risks than they otherwise would

The trading losses that some banks incurred could be thought to be from trading risk, but in many cases the real cause of the losses could be attributed to moral hazard. Why was this the case?

As the chapter points out, in the case of trading, often times the trader (the bank employee) is in a situation where, if the trades turn out profitable, the trader reaps a large portion of the gains; but if the trade generates losses, the bank loses. This is a classic moral hazard problem where the trader has a strong incentive to take significant amounts of risk. This was certainly the case with Nick Leeson and Barings Bank.

Explain how the introduction of asset-backed securities has allowed investors to take advantage of higher returns from loans that most investors could never make on their own.

Asset-backed securities are instruments that allow the holder to share in the returns or payments arising from specific assets such as home mortgages or car loans. Investors purchase shares in the revenue that come from the underlying assets. While most investors would not or could not take the risk of making a home mortgage directly, they can purchase these securities and enjoy the higher return offered by home mortgages with less risk than would be the case if they made a home mortgage directly.

11. Financial intermediaries, through their ability to lower transaction costs: A. Allow for people to be more self-sufficient B. Increase the amount of trading that occurs in an economy C. Take people away from their comparative advantage D. Reduce the number of financial transactions that occur

B

15. Which one of the following refers to selling an option contract? A. calling B. writing C. exercising D. striking E. spotting

B

16. Which of the following has the obligation to sell a stock at the strike price when an option is exercised? A. call holder B. call writer C. put holder D. put writer E. call holder and put writer

B

19. Tom buys a futures contract for U.S. Treasury bonds and on the settlement date the interest rate on U.S. Treasury bonds is higher than Tom expected. Tom will have: a. gained money on his short position. b. lost money on his long position. c. gained money on his long position. d. lost money on his short position.

B

23. Lines of credit provided by financial intermediaries: A. Decrease liquidity for customers but increase income for the intermediary B. Are pre-approved loans that can increase liquidity and lowering transaction costs C. Are costly for intermediaries to provide so are only available to large commercial customers D. Require deposits in the intermediary that equal or exceed the amount of the line of credit

B

23. The user of a commodity who is trying to insure against the price of the commodity rising would: a. take the short position in a futures contract. b. take the long position in a futures contract. c. be better off speculating on price movements and earning higher profits. d. want to hedge by selling a futures contract.

B

25. What is the current price per underlying share if you wish to buy a June $32.50 call option on General Electric stock? =========== A. $0.68 B. $0.70 C. $0.73 D. $1.60 E. $1.62

B

27. Mutual funds are attractive because: A. They provide high returns from purchasing the financial securities of a few select companies B. They provide the investor with greater diversification at a lower cost than what most investors could obtain individually C. They have inside information that is not available to other investors D. They usually have inside information because they run most of the companies they invest in

B

28. A call option is: a. any option written more than sixty days into the future. b. an option giving the holder the right to buy a given quantity of an asset at a specific price on or before a specified date. c. an option giving the seller the right to sell a given quantity of an asset at a specific price on or before a specified date. d. an option where all rights are granted to the seller of the option.

B

33. A put option that is described as in the money would find: a. the market price of the stock above the strike price. b. the strike price is above the market price of the stock. c. the market and strike prices are the same. d. the option has been exercised.

B

34. The main difference between European and American options is: a. holders of European options have more options than holders of American options. b. American option holders have more options than European option holders. c. European option holders can exercise the option prior to expiration. d. European options cannot be resold.

B

37. The seller of a put option is transferring the risk: a. of a price decrease of the stock to the buyer of the option. b. of a price increase of the stock to the buyer of the option. c. this statement is incorrect since options do not transfer risk. d. this statement is incorrect since only sellers of call options are transferring risk.

B

38. Often times we see companies offering money back guarantees to customers if they are not satisfied. These guarantees are a way to treat the problem of: A. Buyers having more information about the product than the seller B. The seller having more information about the product than the buyer C. Symmetric information D. Adverse selection

B

44. Which of the following is a problem of moral hazard? A. A lender cannot distinguish good risk from bad risk borrowers B. An individual who purchases auto insurance begins to leave his or her keys in the car while running into a store C. Life insurance companies offer an average premium to smokers and non-smokers so they do not have to have two different premiums D. An auto insurance company charges higher premiums to younger drivers than what they charge to older drivers

B

46. Which one of the following applies to a naked call? A. unlimited potential profits B. unlimited potential losses C. sale of a put on a stock you do not own D. sale of a call on a stock you currently own E. purchase of a call on a stock you do not own

B

55. Considering interest-rate swaps, the swap spread is: a. another name for the swap rate. b. the difference between the benchmark rate and the swap rate. c. the benchmark rate plus the swap rate. d. a measure of the time value of the swap.

B

55. Which of the following statements is true? A. Adverse selection is a problem of monopoly and moral hazard is a problem of information asymmetry B. Adverse selection and moral hazard are problems stemming from asymmetric information C. Adverse selection is a problem that occurs after a transaction D. Moral hazard is a problem that occurs before a transaction

B

57. One reason lenders usually require a lot of information from loan applicants is to avoid: A. The problems of moral hazard B. The problem of adverse selection C. Being harmed by symmetric information D. Charges of discrimination in lending

B

57. What is the total amount you will receive if you sell 10 June $27.50 puts on Texas Instruments? ============== A. unknown B. $70 C. $100 D. $4,050 E. $4,300

B

6. Which of the following is not a role of a financial institution acting as a financial intermediary? A. Pooling the resources of small savers B. Formulating oversight regulations C. Providing ways to diversify risk D. Supplying liquidity

B

60. The First Bank of Podunk has recently suffered some extraordinary losses on its loan portfolio due to the closing of the largest employer in town. As a result, the bank's management decides to raise the interest rate to new loan applicants. This move is likely to: A. Increase the profitability of the bank B. Cause even greater losses C. Significantly increase both loan applicants and profits D. Treat the problem of adverse selection that contributed to the losses the bank is experiencing

B

62. Recent history has shown that the government regulations requiring the disclosure of information from public corporations have: A. All but eliminated the problems from asymmetric information B. Reduced but not eliminated the problems from asymmetric information C. Just about eliminated the market for information services D. Resulted in symmetric information

B

69. An unsecured loan is: A. A loan where the applicant does not have any net worth B. A loan where the applicant does not post any collateral C. Another name for a mortgage loan D. Usually a low-risk loan

B

69. You purchased 6 call options with a $40 strike price at a total cost of $150. On the expiration date, the underlying stock was priced at $39.20. What is the percentage return on your investment? A. -420 percent B. -100 percent C. 68.75 percent D. 2.02 percent E. 220 percent

B

7. The clearing corporation's main role in the futures market is to: a. set the market price of the contract. b. act as the counter party to both sides of the transaction, thereby guaranteeing payment. c. provide the underlying assets so the contracts can be created. d. all of the above.

B

70. A home mortgage is a good example of: A. An unsecured loan B. A secured loan C. A high risk loan D. The problem of adverse selection

B

73. Which of the following statements is not true? A. Home mortgage loans are secured loans B. Credit card loans are secured C. Most automobile loans are secured loans D. Secured loans usually carry less risk than unsecured loans

B

74. Which of the following statements is true? A. Unsecured loans generally involve very high interest rates as a result of the free-rider problem B. Unsecured loans generally involve very high interest rates as a result of adverse selection C. Unsecured loans are no longer made; all loans now must have some form of collateral D. Unsecured loans are only made to individuals with very high net worth because it is the only way to limit the risk

B

8. Financial intermediaries pool the resources of many small savers so that they can: A. Charge fees to these small savers and earn substantial income B. Obtain the funds necessary to make loans to borrowers seeking large amounts C. Lower their transaction costs of obtaining funds D. Avoid paying any interest to obtain funds to lend

B

8. Marking to market is a process that: a. involves a transfer of risk. b. ensures that the buyers and sellers receive what the contract promises. c. always requires the sellers of contracts to transfer funds to the buyers of contracts. d. buyers and sellers can request for an additional fee when the contract is created.

B

83. Moral hazard problems arise because: A. Lenders cannot distinguish good from bad risks B. Borrowers have incentives to act in ways that do not reflect the lender's interest C. Firms hire incompetent employees D. Lenders charge interest rates that are too low

B

85. Providing stock options to corporate managers was an idea designed to: A. Hide increases in pay of corporate executives from stockholders B. Align managers' interest with the stockholders' interest C. Treat adverse selection D. Treat the free-rider problem

B

86. A 6-month call has a strike price of $30. The underlying stock is priced at $31.80 and the option premium on the call is $2.40. What is the per share time value of the call? A. $0.00 B. $0.60 C. $1.40 D. $2.80 E. $3.60

B

87. A 3-month put has a strike price of $47.50 and an option premium of $1.40. The underlying stock is selling for $46.70 per share. What is the time value of the put? A. $0.00 B. $0.60 C. $0.70 D. $1.20 E. $1.40

B

88. One reason that financial intermediaries exist is that they: A. Are required by government regulation B. Have developed low-cost methods to obtain information C. Are the only way to obtain information D. Earn high returns from lending their own funds

B

89. The screening process a bank follows for a loan applicant: A. Uses information that anyone can obtain, the bank can usually obtain it cheaper B. Includes information that can be available to other firms, as well as proprietary information that only the bank would have C. Is based on just public information D. Uses only confidential information

B

9. There is a futures contract for the purchase of 100 bushels of wheat at $2.50 per bushel. At the end of the day when the market price of wheat increases to $3.00 per bushel: a. the buyer (long position) needs to transfer $50 to the seller (short position). b. the seller (short position) needs to transfer $50 to the buyer (long position). c. nothing happens since with a futures contract all payments are made at the settlement date. d. nothing happens since marked to market adjustments only take place when the market price falls below the contract price.

B

92. In looking at the figure showing the sources of business finance, the largest source in the United States is: A. Financial intermediaries B. Internal funding C. Financial markets D. Foreign sources

B

A $1,000 face value bond purchased for $965.00, with an annual coupon of $60, and 20 years to maturity has a: A. current yield and coupon rate equal to 6.22% and a coupon rate above this. B. current yield equal to 6.22% and a coupon rate below this. C. coupon rate equal to 6.00% and a current yield below this. D. yield to maturity and current yield equal to 6.00%.

B

A bank that makes most of its long-term loans at adjustable interest rates is: A. reducing both interest rate and credit risk. B. increasing credit risk and reducing interest rate risk. C. reducing credit risk and increasing interest-rate risk. D. increasing both interest-rate and credit risk.

B

A bank that meets deposit withdrawal by borrowing additional funds will alter: A. the asset side of their balance sheet. B. the liabilities side of the balance sheet. C. the amount of bank capital. D. the asset and liabilities side of the balance sheet.

B

A bank's Return on Assets (ROA) is calculated by dividing: A. the bank's assets by its net worth. B. the bank's net profits after taxes by its assets. C. the bank's net worth by its assets. D. the bank's assets less its net profit after taxes by its net worth.

B

A non-transaction deposit would include each of the following, except: A. a savings account. B. a checking account. C. a passbook savings account. D. a certificate of deposit.

B

A primary financial market is: A. located only in New York, London, and Tokyo but can handle transactions anywhere in the world. B. one where the borrower obtains funds directly from the lender for newly issued securities. C. a market where U.S. Treasury bonds are traded. D. one that can only deal in the highest investment grade securities.

B

A share of Ford Motor Company stock is an example of: A. a non-standardized financial instrument. B. a standardized financial instrument. C. a non-standardized financial instrument since their prices can differ over time. D. a financial instrument without risk.

B

A zero-coupon bond refers to a bond which: A. does not pay any coupon payments because the issuer is in default. B. promises a single future payment. C. pays coupons only once a year. D. pays coupons only if the bond price is above face value.

B

As general business conditions improve, all other factors constant the: A. price of bonds will increase. B. yield on bonds will increase. C. bond demand curve shifts right. D. bond supply curve shifts left.

B

Consider the bonds below. Which is subject to the greatest interest-rate risk? A. A 30-year fixed-rate mortgage (fixed payment loan) B. A consol C. A Treasury bill D. A 20-year corporate bond

B

Considering U.S. commercial banks, loans account for: A. about one-third of total assets. B. one-half of total assets. C. two-thirds of liabilities. D. three-quarters of total assets.

B

Considering a bank's balance sheet, which of the following statements is true? A. Total Bank Assets = Total Bank Capital - Total Bank Liabilities B. Total Bank Assets = Total Bank Liabilities + Total Bank Capital C. Total Bank Assets + Total Bank Capital = Total Bank Liabilities D. Total Bank Assets + Total Bank Liabilities = Total Bank Capital

B

Considering the balance sheet for all commercial banks in the U.S., the largest category of liabilities is: A. borrowing from other banks in the U.S. B. saving's deposits and time deposits. C. checkable deposits. D. borrowings from non-banks in the U.S.

B

Considering the balance sheet for all commercial banks in the U.S., the net worth of banks is: A. about 5 times the total assets. B. about 1/11 of total assets. C. just about the same as total assets. D. about the same as total liabilities.

B

Considering the value of a financial instrument, the circumstances under which the payment is to be made influence the value because: A. we like uncertain payoffs because this adds to the return. B. payments that are made when we need them the most are more valuable. C. the sooner the payment is to be made the better. D. we know when certain events are going to occur and that is when we want the payment.

B

Countries that lack well-defined property laws and legal structures: A. have large secondary financial markets because the primary markets do not exist. B. will not develop as fast economically as counties with clear property rights and a formal legal system. C. will have much lower transaction costs associated with any level of lending. D. will not have any financial markets at all.

B

Derivatives would include all of the following except: A. options. B. U.S. Treasury securities. C. swaps. D. futures.

B

Financial institutions: A. raise the level of transaction costs relating to borrowing/lending. B. can lower the information asymmetry involved with borrowing/lending. C. decrease the liquidity to savers. D. are required for all financial transactions.

B

Financial instruments used primarily as stores of value include each of the following, except: A. bonds. B. futures contracts. C. stocks. D. home mortgages.

B

Financial markets enable the transfer of risk by: A. requiring that risk-averse investors have access to U.S. Treasury bond markets. B. allowing individuals and firms less willing to bear risk to transfer risk to other individuals and firms more willing to bear risk. C. making sure that higher default risk is offset by greater liquidity. D. enabling even unsophisticated investors to purchase highly complex financial instruments.

B

For every $100 in assets, a bank has $40 in interest-rate sensitive assets, and the other $60 in non-interest-rate sensitive assets. The same bank has $50 for every $100 in liabilities in interest-rate sensitive liabilities, the other $50 are in liabilities that are not interest-rate sensitive. If the interest rate on assets increases from 5 to 6 percent, and the interest rate on liabilities increases from 3 to 4 percent, the impact on the bank's profits per $100 of assets will be: A. an increase of $0.10. B. a decrease of $0.10. C. a reduction of $1.00. D. zero since the interest rates on assets and liabilities increased by the same amount.

B

If a bank has $150 million in assets and a net worth of $20 million, its asset-to-equity ratio is: A. 6.5 to 1. B. 7.5 to 1. C. 0.13 to 1. D. 0.15 to 1.

B

If a bank has more interest-rate sensitive liabilities than interest-rate sensitive assets, an increase in the interest rate will cause profits to: A. increase. B. decrease. C. remain constant. D. be negative, meaning there will not be profits, only losses.

B

If a bank's return on equity remains constant, but the ratio of bank assets to bank capital increases: A. the bank's return on assets must have increased. B. the bank's return on assets must have decreased. C. the bank's assets and capital must have increased by the same percentage. D. the bank must be unprofitable.

B

If bank with leverage of 8 to 1 increases its assets by adding $1 to capital for every $1 added to assets: A. leverage increases. B. leverage decreases. C. leverage stays constant. D. the answer cannot be determined from the information in the question.

B

If interest rates are expected to rise, the bond prices will: A. not change until interest rates actually change. B. fall, due to the demand for bonds decreasing. C. rise, as people seek capital gains. D. move in the same direction as the expected change in interest rates.

B

If the U.S. government's borrowing needs increase, all other factors constant the: A. price of bonds will increase. B. supply of bonds will increase. C. demand for bonds will decrease. D. supply of bonds and the demand for bonds will both increase.

B

If the U.S. government's borrowing needs increase, in the bond market this would be seen as the: A. bond demand curve shifting right. B. bond supply curve shifting right. C. bond demand curve shifting left. D. bond supply curve shifting left.

B

If the annual interest rate is 5%(.05), the price of a six-month Treasury bill would be: A. $97.50 B. $97.59 C. $95.25 D. $95.00

B

If the quantity of bonds supplied exceeds the quantity of bonds demanded, bond prices would: A. rise and yields would fall. B. fall and yields would rise. C. rise but yields will remain constant. D. fall and yields would fall.

B

In calculating the current yield for a bond the: A. coupon payment is ignored. B. present value of the capital gain/loss is ignored. C. present value of the final payment is the only important consideration. D. present value of the coupon payments is the only important consideration.

B

Interest-rate risk results from: A. bond prices being fixed over the life of the bond. B. a mismatch between an individual's investment horizon and a bond's maturity. C. the fact that most people hold bonds until they mature. D. inflation being uncertain.

B

Loans made between borrowers and lenders are: A. liabilities to the lenders and assets to the borrowers since the borrower obtains the funds. B. assets to the lenders and liabilities of the borrowers since the promises are made to the lenders. C. not part of either parties' assets or liabilities until the loans are repaid. D. liabilities to both the lenders and the borrowers.

B

Many financial instruments are standardized because: A. it is believed that most parties to a contract do not read them anyway. B. complexity is costly, the more complex a contract, the more it costs to create. C. the standardization of contracts makes them harder to understand. D. it is required by the government.

B

Mary purchases a U.S. Treasury bond; the bond is a(n): A. asset of the U.S. government as well as an asset for Mary. B. liability of the U.S. government and an asset for Mary. C. asset for Mary but not a liability of the U.S. Government. D. asset for the government but a liability for Mary.

B

More detailed financial instruments tend to be: A. less costly because all possible contingencies are covered. B. more costly because it will cost more to create. C. more desirable than less detailed ones, no matter what the price. D. less costly because they can be standardized more easily.

B

Most individuals borrow: A. directly without the use of a financial intermediary. B. using a financial intermediary because it lowers the cost of borrowing. C. using a financial intermediary, but would save money if they financed directly. D. without using financial intermediaries, preferring credit cards.

B

One way for a bank to deal with credit risk is to: A. charge all borrowers from the same industry an average rate for that industry. B. add a mark-up for a specific borrower based on the borrower's credit history to the cost of funds. C. avoid making loans to borrowers from a broad spectrum and to specialize geographically and in specific industries. D. increase the number of loans made in any year.

B

Repurchase agreements are usually used by banks that: A. have a need for long-term financing. B. need cash for a very short period of time. C. have negative net worth. D. cannot obtain financing from any other source.

B

Savings and loan institutions: A. are owned by the depositors. B. originally were formed primarily to make home mortgages. C. today offer a much smaller array of services than when originally formed. D. are owned by depositors who also have a common bond.

B

Sue has a checking account at the First National Bank; her checking account is a(n): A. asset to the bank and a liability to Sue. B. asset to Sue and a liability to the bank. C. asset to Sue but actually a liability to the Federal Reserve. D. liability to Sue until she spends the funds.

B

Suppose there is a decrease in the price at which a bondholder sells her bond. In this case, the holding period return will: A. increase, since yields and prices are inversely related. B. decrease, since this lowers the capital gain. C. be negative. D. equal the coupon rate.

B

The New York Stock Exchange (NYSE) originated as: A. a decentralized electronic market made up of dealers all over the world. B. an example of a centralized exchange. C. a financial market where nearly 100 million shares of stock are traded every business day. D. the only centralized stock exchange in the world.

B

The bid price for a bond quote is: A. the price at which the bond dealer is willing to sell the bond. B. the price at which the bond dealer is willing to purchase the bond. C. fixed over the life of a bond. D. determined solely by the time left to maturity.

B

The high volume of shares of stock that are traded on a normal day on stock markets reflects the: A. high transaction costs associated with these financial markets. B. low transaction costs and high liquidity associated with these markets. C. low transaction costs and low liquidity associated with these markets. D. high transactions costs and low liquidity associated with these markets.

B

The most common form of zero-coupon bonds found in the United States is: A. AAA rated corporate bonds. B. U.S. Treasury bills. C. 30-year U.S. Treasury bonds. D. municipal bonds.

B

The primary difference in certificates of deposit (CDs) that are equal to or less than $100,000 and those over $100,000 (other than the amount) is: A. a bank does not have to include CDs equal to or less than $100,000 in its liabilities. B. CDs greater than $100,000 are negotiable and therefore can be bought and sold. C. CDs equal to or less than $100,000 are issued for only six months or less. D. CDs greater than $100,000 are issued for only six months or less.

B

The primary use of derivative contracts is: A. for IRA and other pension plans since they only have value well into the future. B. to shift risk among investors. C. for investors seeking a greater return by taking greater risk. D. to add to the profits an investor obtains through information asymmetry.

B

The process of financial intermediation: A. creates a net cost to an economy. B. increases the economy's ability to produce. C. is always used when a borrower needs to obtain funds. D. is used primarily in underdeveloped countries.

B

The value of a financial instrument rises as: A. the size of the payment promised decreases. B. the promised payment is made sooner rather than later. C. it is less likely the payment will be made. D. the payments are made when the prospective investor needs them least.

B

When expected inflation increases, for any given nominal interest rate the: A. bond demand curve shifts right. B. bond supply curve shifts right. C. price of bonds increases. D. yield on bonds will increase.

B

When the current yield and the coupon rate are equal, the bond is: A. purchased at a discount. B. purchased at a price that equals the face value. C. a zero-coupon bond. D. purchased at a price that exceeds its face value.

B

When the price of a bond is below the face value, the yield to maturity: A. is below the coupon rate. B. will be above the coupon rate. C. will equal the current yield. D. will equal the coupon rate.

B

Which of the following is a bank liability? A. Mortgage loans B. Demand deposits C. Reserves D. U.S. Treasury securities

B

Which of the following is not true of over-the-counter markets? A. Traders are linked by computer. B. Dealers buy and sell only for their customers. C. Trading does not take place in one physical location. D. Traders are willing to buy and sell stocks and bonds at posted prices.

B

Which of the following statements is most correct? A. Financial intermediaries are banks. B. A bank is a financial intermediary. C. Financial intermediaries are insurance companies. D. Financial intermediaries are essential to direct finance.

B

Which of the following statements is most correct? A. When a risk is difficult to predict, financial instruments are created to transfer these risks. B. Financial instruments are created to transfer risks that are relatively easy to predict. C. Financial instruments require certainty of an event to be able to transfer risk. D. Financial instruments eliminate the risk from uncertainty, they do not transfer it.

B

Which of the following would not be an example of a secondary financial market transaction? A. You call a broker and purchase 100 shares of McDonalds Corp. stock. B. You go to the bank and purchase a $5000 certificate of deposit. C. You call a broker and purchase a U.S. Treasury bond. D. You call a broker and purchase a bond issued by General Motors.

B

5) Which one of the following statements related to annuities and perpetuities is correct? A) An ordinary annuity is worth more than an annuity due given equal annual cash flows for 10 years at 7 percent interest, compounded annually. B) A perpetuity comprised of $100 monthly payments is worth more than an annuity of $100 monthly payments provided the discount rates are equal. C) Most loans are a form of a perpetuity. D) The present value of a perpetuity cannot be computed but the future value can. E) Perpetuities are finite but annuities are not.

B) A perpetuity comprised of $100 monthly payments is worth more than an annuity of $100 monthly payments provided the discount rates are equal.

24) You need $25,000 today and have decided to take out a loan at 7 percent interest for five years. Which one of the following loans would be the least expensive for you? Assume all loans require monthly payments and that interest is compounded on a monthly basis. A) Interest-only loan B) Amortized loan with equal principal payments C) Amortized loan with equal loan payments D) Discount loan E) Balloon loan where 50 percent of the principal is repaid as a balloon payment

B) Amortized loan with equal principal payments

14) Which one of the following statements related to loan interest rates is correct? A) The annual percentage rate considers the compounding of interest. B) When comparing loans you should compare the effective annual rates. C) Lenders are most apt to quote the effective annual rate. D) Regardless of the compounding period, the effective annual rate will always be higher than the annual percentage rate. E) The more frequent the compounding period, the lower the effective annual rate given a fixed annual percentage rate.

B) When comparing loans you should compare the effective annual rates.

13) Your credit card charges you .85 percent interest per month. This rate when multiplied by 12 is called the ________ rate. A) effective annual B) annual percentage C) periodic interest D) compound interest E) episodic interest

B) annual percentage

23) An amortized loan: A) requires the principal amount to be repaid in even increments over the life of the loan. B) may have equal or increasing amounts applied to the principal from each loan payment. C) requires that all interest be repaid on a monthly basis while the principal is repaid at the end of the loan term. D) requires that all payments be equal in amount and include both principal and interest. E) repays both the principal and the interest in one lump sum at the end of the loan term.

B) may have equal or increasing amounts applied to the principal from each loan payment.

The original Basel Accord was: A. the basic set of guidelines the Federal Reserve applies in regulating domestic banks. B. a set of guidelines for basic capital requirements for internationally active banks. C. an agreement between state and federal regulators to try to have one standard set of guidelines for all banks. D. a set of guidelines applied only to international banks operating with U.S. boundaries.

B. A set of guidelines for basic capital requirements for internationally active bankc

As a result of government provided deposit insurance, the ratio of assets to capital for commercial banks since the 1920s has: A. just about doubled. B. almost tripled. C. not changed. D. decreased.

B. Almost tripled

the main difference between European and American options is:

B. American option holders have more options than European options holders

One reason a bank's officer may be reluctant to write off a past-due loan is that it will: A. increase the bank's liabilities. B. decrease the bank's assets and capital. C. increase the bank's liabilities and assets, requiring more capital to be held. D. make the bank's accounts less transparent.

B. Decrease the bank's assets and capital

The government provides deposit insurance; this insurance protects: A. large corporate deposit accounts, but only the amounts that exceed the $250,000 deductible. B. depositors for up to $250,000 should a bank fail. C. the deposits of banks in their Federal Reserve accounts. D. the deposits that people have, but only for federally chartered banks.

B. Depositors for up to $250,000 should a bank fail

The purpose of the government's safety net for banks is to do each of the following, except: A. protect the integrity of the financial system. B. eliminate all risk that investors face. C. stop bank panics. D. improve the efficiency of the economy.

B. Eliminate all risk that investors face

Which of the following is not a pillar of the latest Basel Accord? A. A revised set of minimum capital requirements B. It includes liquidity requirements in addition to capital requirements C. It supplements capital requirements based on risk-weighted assets with restrictions on leverage D. Uniform international laws for bank regulation

B. It includes liquidity requirements in addition to capital requirements

The government's too-big-to-fail policy applies to: A. certain highly populated states where a bank run impacts a large percent of the total population. B. large banks whose failure would start a widespread panic in the financial system. C. large corporate payroll accounts held by some banks where many people would lose their income. D. banks that have branches in more than two states.

B. Large banks whose failure would start a widespread panic in the financial system

The acronym CAMELS, which is the criteria used by supervisors to evaluate the health of banks, includes the following, except: A. asset quality. B. losses. C. management. D. earnings.

B. Losses

Which of the following statements is most correct? a. managers, directors, and stockholders almost always share the same interest b. managers' and directors' interests often conflict with stockholder's interest c. managers and stockholders have the same interests, but this usually conflicts with the interests of directors d. directors and stockholders have the same interests, but this usually conflicts with the interests of managers

B. Managers' and directors' interests often conflict with the stockholders' interest

The supervision of banks includes: A. requiring bank officers to attend classes on an annual basis. B. on-site examinations of the bank. C. extensive background checks of all bank officers. D. requiring banks to file monthly reports on their revenues, expenses and profits.

B. On-site examinations of the bank

Considering the methods available to the FDIC for dealing with a failed bank, the depositors of the failed bank should: A. be indifferent between the two since it really does not matter to them which method is used. B. prefer the purchase and assumption method since the deposits over $250,000 will also be protected. C. prefer the payoff method because they will have access to their funds earlier. D. prefer the payoff method since a lot less paperwork is involved for the depositor.

B. Prefer the purchase and assumption method since the deposits over $250,000 will also be protected

During the financial crisis of 2007-2009 in the United States it was revealed that the function of a lender of last resort had not kept pace with the evolving financial system because: A. financial intermediaries had grown sufficiently large so as not to need a lender of last resort. B. shadow banks lacked access to the financial resources available through the lender of last resort. C. banks were sufficiently linked to one another that the need for a lender of last resort had diminished. D. banks had become sufficiently diversified so as to be able to provide for their own liquidity.

B. Shadow banks lacked access to the financial resources available through the lender of last resort

In the United Kingdom, regulation of the financial system is concentrated in two agencies. They are: A. The Federal Deposit Insurance Conglomerate and the Bank of England. B. The Financial Conduct Authority and the Bank of England. C. The Financial Conduct Authority and English Banking Authority. D. The Bank of England and the U.K. Treasury.

B. The Financial Conduct Authority and the Bank of England.

If market participants believe next year's corn crop is likely to be unusually large:

B. The current spot market price of corn is likely to be above the futures price of corn

Which of the following statements is most correct? A. the higher the deposit insurance limit the lower the risk of moral hazard. B. the higher the deposit insurance limit the greater the risk of moral hazard. C. deposit insurance limits do not impact moral hazard, they impact adverse selection. D. increasing the deposit insurance limits above $100,000 would increase coverage for over 50 percent of all depositors.

B. The higher the deposit insurance limit the lower the risk of moral hazard

One lesson learned from the bank panics of the early 1930's is: A. the lender of last resort function almost guarantees that bank panics are a thing of the past. B. the mere existence of a lender of last resort will not keep the financial system from collapsing. C. only the U.S. Treasury can be a true lender of last resort. D. the financial system will collapse without a lender of last resort.

B. The mere existence of a lender of last resort will not keep the financial system from collapsing

Bank mergers require government approval because banking officials want to make sure that: A. the merger will create a larger bank. B. the merger will not create a monopoly. C. the merged bank will be more profitable. D. the merger will not result in regulatory competition.

B. The merger will not create a monopoly

Financial regulators set capital requirements for banks. One characteristic about these requirements is: A. every bank will have to hold the same level. B. the riskier the asset holdings of a bank, the more capital it will be required to have. C. the more branches a bank has, the more capital it must have. D. the amount of capital required is inversely related to the amount of assets the bank owns.

B. The riskier the asset holdings of a bank, the more capital it will be required to have

There is a futures contract for the purchase of 100 bushels of wheat at $2.50 per bushel. At the end of the day when the market price of wheat increases to $3.00 per bushel:

B. The seller (short position) needs to transfer $50 to the buyer (long position)

Regulators and supervisors of banks are challenged by all of the following, except: A. globalization of financial services. B. the use of new financial instruments that shift risk without shifting ownership. C. technological innovation. D. reinforcement by Congress of functional and geographic barriers in banking.

B. The use of new financial instruments that shirt risk without shirting ownership

Banks are required to disclose certain information. This disclosure is done for all of the following reasons except: A. to enable regulators to more easily assess the financial condition of banks. B. to allow financial market participants to penalize banks that carry additional risk. C. to allow customers to more easily compare prices for services offered by banks. D. create uniform prices for standard bank services.

B. To allow financial market participants to penalize banks that carry addition risk

The most broadly based stock index in use is the:

B. Wilshire 5000

Forward contracts are:

B. contracts usually involving the exchange of a commodity or financial instrument

The dividends that stockholders receive are:

B. distributions from profits

Marking to market is a process that:

B. ensures that the buyers and sellers receive what the contract promises

with a call option, the option holder:

B. has the right to buy the asset

speculators differ from hedgers in the sense that:

B. hedgers seek to transfer risk

Futures markets and derivatives contribute to economic growth by:

B. increasing the risk-taking capacity of the economy

During a bank crisis: A. officials at the Federal Reserve find it easy to sort out solvent from insolvent banks. B. it is important for regulators to be able to distinguish insolvent from illiquid banks. C. it is easy to determine the market prices of bank's assets. D. a bank will go to the central bank for a loan before going to other banks.

B. it is important for regulators to be able to distinguish insolvent from illiquid banks

Tom buys a futures contract for U.S Treasury bonds and on the settlement date the interest rate on the bonds is higher than Tom expected. Tom will have:

B. lost money on his long position

An economic rationale for government protection of small investors is that: A. large investors can better afford losses. B. many small investors cannot adequately judge the soundness of their bank. C. there is inadequate competition to ensure a bank is operating efficiently. D. banks are often run by unethical managers who will often exploit small investors.

B. many small investors cannot adequately judge the soundness of their bank

the seller of a put option is transferring the risk:

B. of a price increase of the stock to the buyer of the option

the primary risk in swaps is that:

B. one of the parties will default

Two characteristics that make owning stock attractive are:

B. share prices are relatively inexpensive and are transferable

An individual who neither uses nor produces a commodity but buys a futures contract for the asset is:

B. speculating that the price of the commodity is going to increase

The user of a commodity who is trying to insure against the price of the commodity rising would:

B. take the long position in a futures contract

You have savings accounts at two separately FDIC insured banks. At one of the banks your account has a balance of $150,000. At the other bank the account balance is $60,000. If both banks fail, you will receive:

$210,000

considering interest-rate swaps, the swap spread is:

B. the difference between the benchmark rate and the swap rate

If your stockbroker gives you bad advice and you lose your investment: A. the government will reimburse you similar to reimbursing depositors if a bank fails. B. the government will not reimburse you for the loss; you are not protected from bad advice by your stockbroker. C. these losses would be covered under FDIC insurance. D. your investment would only be covered if the stockbroker was employed by a bank.

B. the government will not reimburse you for the loss; you are not protected from bad advice by your stockbroker

Asymmetric information in financial markets is a potential problem usually resulting from: a. Borrowers having more information than the lenders, and not disclosing this information b. Lenders having more information than borrowers and not disclosing this information c. The fact that people are basically dishonest d. The uncertainty about Federal Reserve monetary policy

. Borrowers having more information than the lenders, and not disclosing this information

What are the five features of a futures contract?

1.) Identify of the underlying commodity or financial instrument 2.) The futures contract size 3.) The futures maturity date (AKA: expiration date) 4.) The delivery or settlement procedure 5.) The futures price

Three essential things to know about futures trading accounts:

1.) Margin is required 2.) Futures accounts are maked to market daily 3.) A futures position can be closed out any time by a reverse trade

The number of voting members on the Federal Open Market Committee (FOMC) is:

12

Fly-By-Night Inc. issues $100 face value, zero-coupon, one-year bonds. The current return on one-year, zero-coupon U.S. government bonds is 3.5%. If the Fly-By-Night bonds are selling for $92.00, what is the risk premium for these bonds?

5.2%

A saver knows that if she put $95 in the bank today she will receive $100 from the bank one year from now, including the interest she will earn. What is the interest rate she is earning? a. 5.10% b. 6.00% c. 5.52% d. 5.26%

5.26%

Most individuals save at banks rather than lend directly because:

Banks can reduce the cost of information asymmetry

The government's role of lender of last resort is directed to:

Banks that experience sudden deposit outflows (Ch. 14)

The stability of the financial system is enhanced by the ability of the central banks to:

Be a lender of last resort

The attendees at the FOMC meetings receive information prior to the meetings that is contained in books with colorful names. The information that is released to the public prior to the meetings is from the:

Beige book only (Ch. 16)

The long position in a futures contract is the party that will:

Benefit from increases in the price of the underlying asset.

Moral hazard problems arise because:

Borrowers have incentives to act in ways that do not reflect lender's interest (Ch. 11)

The usual situation in banking regarding asymmetric information is:

Borrowers know more than lenders

The usual situation in banking regarding asymmetric information is:

Borrowers know more than lenders.

If financial intermediaries did not have the ability to pool the resources of small savers:

Borrowers needing large amounts of money would find it more costly to obtain the funds. (Ch. 11)

Bank panics often begun as a result of:

Both rumors and real economic events

Which of the following statements is TRUE?

A liquid asset is one that can be quickly and cheaply converted into cash.

Which of the following is an example of a financial market? a. A local coffeehouse where people regularly buy and sell financial instruments b. A bank that only accepts deposits and issues loans c. An electronic network used for buying and selling textbooks d. A central bank used for raising taxes and borrowing on behalf of the government

A local coffeehouse where people regularly buy and sell financial instruments

Short position

A market position where the holder benefits from price decreases and loses from price increases

Long position

A market position where the holder benefits from price increases and loses from price decreases

Speculator

A person or firm that takes the risk of loss for the chance for profit

The theory of efficient markets means:

A professional fund manager should really not expect to beat the market average consistently.

The financial system is inherently more unstable than most other industries due to the fact that:

A single firm failing in banking can bring down the entire system; this isn't true in most other industries

1. In a derivative transaction: a. the dollar amount of the transaction increases as the contract date approaches. b. the risk is less than if actually purchasing the underlying asset. c. what one person gains is what the other person loses. d. there is always a futures contract.

C

14. Which one of the following is equal to the option premium minus the intrinsic value? A. parity value B. payoff value C. time value D. strike value E. profit

C

18. You currently own 300 shares of Microsoft stock. If you purchase options on this stock to protect against future declines in the price of the stock you are implementing which one of the following? A. covered call B. naked call C. protective put D. bear spread E. straddle

C

16) Which one of the following compounding periods will yield the lowest effective annual rate given a stated future value at Year 5 and an annual percentage rate of 10 percent? A) Annual B) Semi-annual C) Monthly D) Daily E) Continuous

A) Annual

19) Amortized loans must have which one of these characteristics over its life? A) Either equal or unequal principal payments B) One lump-sum principal payment C) Increasing payments D) Equal interest payments E) Declining periodic payments

A) Either equal or unequal principal payments

A bank's loan loss reserves are:

An amount the bank sets aside to cover potential losses from defaulted loans.

19. Automated teller machines provided by financial intermediaries are an example of: A. High transactions costs associated with financial intermediaries B. Diseconomies of scale C. The ability of financial intermediaries to provide liquidity D. The ability of financial intermediaries to earn profits by raising transaction costs above the norm

C

1. Explain how an interest rate futures contract differs from an outright purchase of a bond.

An investor who purchases a bond does so with the thought that the price of a bond is going to rise. There really is no way to profit from a price decline when you actually purchase the bond. On the other hand, an investor can profit from price declines in bonds by using an interest rate futures contract. With such a contract, two individuals agree that they will make payments to the counterparty based on interest rate movements over some specified time period. Another key difference is that with the futures contract (derivative) one person's loss is the counter party's gain. The amount on the table never changes, it just moves between the counterparties. Yet another difference is that a futures contract is equivalent to a low-cost, leveraged exposure to fluctuations in the bond price.

20. The function of providing liquidity by financial intermediaries: A. Includes depositors withdrawing funds but not borrowers B. Only considers people who borrow on a short-term basis, but not depositors C. Affects people who need to borrow and depositors who withdraw their funds D. Only affects customers with savings accounts

C

All of the following are operating expenses for a bank except ________. A) service charges on deposit accounts B) salaries and employee benefits C) rent on buildings D) servicing costs of equipment such as computers

Answer: A

21. Since one function of financial intermediaries is to provide liquidity: A. They must keep all of their funds in short-term securities B. They keep almost all of their funds in cash C. They must know approximately how much liquidity their customers will need each day and have these funds available D. Regulations require financial intermediaries to keep 50% of their assets in cash

C

13. Speculators differ from hedgers in the sense that: a. speculators do not like risk. b. hedgers seek to transfer risk. c. speculators seek to transfer risk. d. speculators are hedgers, there isn't any difference.

B

We saw in Chapter 12 that initially savings and loans were created to make home mortgages, and their main source of funds was deposits from savers. In the late 1970's and into the 1980's, the U.S. experienced rising interest rates that had depositors looking for higher returns. Congress quickly removed the interest rate ceilings that savings and loans could offer. Explain the initial impact this had on the interest rate spread and the net interest margin for the savings and loans.

Before the dramatic increase in interest rates, savings and loans were paying a regulated rate on their liabilities (deposits) and earning a higher rate on their assets (mortgages). They had a positive interest rate spread which also gave them a positive net interest margin. Since they were quite confident concerning the cost of funds, they could set mortgage rates that would ensure an adequate spread and margin. Once interest rates began to increase and the rate ceiling on deposits was removed, the interest rate offered to depositors (cost of funds) increased, but because most of the assets of savings and loans were in long-term mortgages, the interest earned on assets was fairly interest rate insensitive. This quickly put many savings and loans into negative spread and margin positions and contributed to the crisis that was to hit this industry.

The objectives set for the Fed by Congress are:

By design, quite vague, allowing the Fed to really set its own goals (Ch. 16)

What are the securities that U.S. banks own and why are they often referred to as secondary reserves?

By regulation U.S. banks are not allowed to own stocks, so their securities are limited to bonds. And, for regulatory reasons, banks choose to hold U.S. government and agency bonds. Most of these bonds are highly liquid and in the case of need, can be sold quickly to back up the cash positions of the bank. As a result they are often referred to as secondary reserves.

1. Financial intermediation is: A. Far less important than direct finance through stock and bond markets B. Only a little more important than direct finance in the United States C. Much more important than direct finance through stock and bond markets D. The same thing as finance through stock and bond markets

C

25. If a bank has 1,000 depositors, each of whom deposits $1,000 in the bank, and the bank makes 100 loans of $10,000 each, then each depositor has contributed: A. $100 to each loan B. $1 to each loan C. $10 to each loan D. $1000 to each loan

C

27. The right to buy a given quantity of an underlying asset at a predetermined price on or before a specific date is called a(n): a. put option. b. option writer. c. call option. d. arbitrage contract.

C

28. A bank has 10,000 depositors, each of whom deposits $100 in the bank. If the bank makes 1000 loans for $1,000 each then each depositor has contributed: A. $1 to each loan B. $100 to each loan C. $0.10 to each loan D. $10 to each loan

C

28. Which of the following characteristics are correct regarding the old style option quotation system? I. The system is known as OPRA - the Options Price Reporting Authority code II. The system has 3 data elements III. The system has 21 characters IV. The system has 5 characters V. The system is known as the OCC Series Key VI. The root symbol is the underlying stock's ticker symbol A. I, II, III, and VI B. II, III, IV, and V C. I, II, and IV D. II, III, V, and VI E. III, V, and VI

C

30. Most individuals save at banks rather than lend directly because: A. The bank creates information asymmetry B. Moral hazard exists only when individuals make loans directly to borrowers, it does not occur when banks issue loans C. Banks through economies of scale can reduce the cost of information asymmetry D. Information asymmetry is a problem for individuals but not for banks

C

31. With a put option, the option holder: a. has the right to buy the asset. b. can buy or sell the asset, it is their option. c. has the right to sell the asset. d. can buy the asset but only on the date specified.

C

32. What is the maximum percentage loss you can incur if you buy a put option? A. 0% B. 10% C. 100% D. 1,000% E. unlimited percentage

C

32. With a call option that is described as in the money: a. the market price of the stock is below the strike price. b. the market price of the stock equals the strike price. c. the market price of the stock is above the strike price. d. the option has been exercised.

C

35. If information in a financial market is symmetric, this means: A. Borrowers and lenders have perfect information B. Borrowers would have more information than lenders C. Borrowers and lenders have the same information D. Lenders have more information than borrowers

C

35. One key difference between options contracts and futures contracts is: a. in a futures contract, one part has more rights than the other. b. with an options contract both parties have equal rights. c. in an options contract, the rights belong to one party. d. in a futures contract all rights are held by just one party.

C

36. Mom's Pizzeria goes out of business due to a dramatic decrease in sales from a local newspaper article highlighting the fact that Mom's Pizzeria has been purchasing expired meat from a distributor at cut rate prices for years. The decrease in business also results in Mom's defaulting on the loan they have with the bank. This is an example of: A. Symmetric information in the financial markets B. Perfect information in the financial markets C. Asymmetric information in the financial markets D. Perfect information in the pizza market

C

37. Which one of the following statements is true? A. A call with a strike price of $25 and a stock price of $23 has positive intrinsic value. B. A European style option is more valuable than an American style option. C. An American style out-of-the-money call option can have a positive value. D. A $40 put option has more intrinsic value than a $50 put option on the same underlying asset. E. The time value of an option is equal to the intrinsic value minus the option premium.

C

Financial markets: a. Enable buyers and sellers to exchange financial instruments but not risk b. Enable buyers and sellers to exchange risk by buying and selling financial instruments c. Only allow the transfer of risk through derivative securities d. Do not allow for the transfer of risk but do help reduce it

Enable buyers and sellers to exchange risk by buying and selling financial instruments

39. The two parts that make up an option's price are: a. extrinsic value and the time value of the option. b. the commission and the time value of the option. c. the intrinsic value and the time value of the option. d. the price of the underlying asset and the time value of the option.

C

4. The long position in a futures contract is the party that will: a. benefit from decreases in the price of the underlying asset. b. agree to make delivery of a commodity or financial instrument at a future date. c. benefit from increases in the price of the underlying asset. d. accept the greater share of the risk.

C

4. Which one of the following is defined as the price at which an option will be exercised? A. straddle B. spread C. strike D. market E. underlying

C

42. Two problems that arise from asymmetric information are: A. Adverse selection and diseconomies of scale B. Moral hazard and the free-rider problem C. Moral hazard and adverse selection D. The free-rider problem and adverse selection

C

43. Which one of the following is the primary purpose of a protective put? A. profit from an expected future increase in the underlying stock's value B. guarantee a higher return than is possible from just owning the underlying security C. offset the risk associated with a decrease in the value of the underlying asset D. receipt of the option premium E. increase in potential rate of return due to increase in risk

C

45. One of the conclusions from Akerlof's paper titled "The Market for Lemons" was: A. High quality goods will drive low quality goods out of the market B. Lacking the ability to distinguish high from low quality, the quality the market will end up offering will be the average quality C. Lacking the ability to distinguish high from low quality, low quality may drive high quality out of the market D. High quality is always demanded by consumers over low quality

C

45. You wrote a covered call with a strike price of $45 and an option premium of $1.10. Assume the stock price is $44 a share currently and that it falls to $42 a share and remains at that price until the option expires. As a result, you will: A. lose an amount equal to the option premium. B. lose the option premium but get to keep the stock. C. keep both your stock and the option premium. D. keep the option premium but lose your shares of stock. E. lose both your stock and the option premium.

C

46. As the volatility of the stock price increases, the time value of the option: a. decreases. b. is zero. c. increases. d. doesn't change.

C

48. Which one of the following is a bear call spread? A. buying a $20 call and selling a $25 call on the same stock B. selling a $20 call and buying a $20 call on the same stock C. buying a $20 call and selling a $15 call on the same stock D. selling a $20 call and buying a $25 put E. buying a $20 call and selling a $25 put

C

49. If we have a stock selling for $95.00 and a call option for this stock has a strike price of $82.00 and an option price of $13.60: a. the intrinsic value of the option is $0.60 and the time value of the option is $13.00. b. the intrinsic value is $82.00 and the time value of the option is $13.60. c. the intrinsic value of the option is $13.00 and the time value of the option is $0.60. d. the intrinsic value is $0 since the option is out of the money.

C

50. A short straddle: A. involves exercising two or more options simultaneously. B. is the purchase of both a put and a call on the same underlying asset. C. obtains its maximum profit when the underlying stock price is equal to the strike price. D. involves writing a call on shares of stock you currently own. E. is a highly bullish strategy.

C

50. If the market prices the shares of stock of two companies, one of high quality and the other of lower quality, are the same average price and potential buyers cannot distinguish the prospects of the companies: A. The shares of the low quality firm will disappear from the market B. The shares of both companies will trade on the market C. The shares of the high quality firm will disappear from the market D. This is an example of moral hazard and the shares of both companies will cease to trade

C

52. A key use of interest-rate swaps is to: a. eliminate risk for both parties involved in the transaction. b. earn the fees for constructing the swaps. c. provide a hedge against interest-rate risk. d. manage government revenues.

C

56. Which of the following statements is true? A. Adverse selection is a problem that occurs after a transaction B. Moral hazard is a problem that occurs before a transaction C. Adverse selection is a problem stemming from asymmetric information D. Both adverse selection and moral hazard occur before a transaction

C

58. One reason the government requires public corporations to disclose so much information is to: A. Minimize the monopoly profits some corporations earn B. Give small corporations a better chance of competing against large corporations C. Address the potential harm from asymmetric information D. Discourage risk-taking by investors

C

59. A lender who wants to avoid the problem of adverse selection could: A. Charge a very high interest rate and assume all loan applicants are high risk B. Charge the same average interest rate to all borrowers C. Charge a low interest rate and make the applicant prove they warrant the low rate by providing information D. Only lend by issuing credit cards

C

60. Katie purchased 6 call options on Atlas Co. stock with a strike price of $40.00. On the expiration date, the stock was priced at $38.95 a share. What is the total payoff on the option contracts? A. -$220 B. -$55 C. $0 D. $2.20 E. $55

C

62. Jennifer purchased 4 put option contracts on Winslow Mfg. stock. The option premium was $0.25 and the strike price was $17.50. On the expiration date, the stock was selling for $17.75 a share. What is the total payoff on the option contracts? A. -$100 B. -$50 C. $0 D. $50 E. $150

C

64. Tim purchased 5 put option contracts on Western Fields stock. The strike price was $35 and the option premium was $0.55. At expiration, the stock was selling for $35.75. What is the payoff on the option contracts? A. -$60 B. -$30 C. $0 D. $30 E. $60

C

68. Private mortgage insurance is usually required in situations where: A. The lender feels the buyers have overpaid for the house B. The buyers have no down payment C. The buyers have a down payment less than 20 percent of the purchase price D. In the lenders' view the buyers do not have adequate monthly income to handle the mortgage payment

C

68. You own one SPX put option with a strike of 1,300. What is the payoff at maturity for this option contract if the S&P 500 index is 1,322? A. -$3,600 B. -$36 C. $0 D. $36 E. $3,600

C

7. A list of available option contracts and their prices for a particular security listed in order of strike price and maturity date is referred to as which one of the following? A. value chain B. intrinsic list C. option chain D. strike list E. exercise price display

C

7. Financial institutions, acting as financial intermediaries, perform all of the following, except: A. Provide ways to diversify risk B. Pooling resources of small savers C. Increase transactions costs D. Provide safekeeping and accounting services

C

71. Courtney purchased 5 call options with a $47.50 strike price and a call premium of $1.10. On the expiration date, the underlying stock was priced at $50.60 per share. What is her percentage return on this investment? A. -100 percent B. 70.45 percent C. 181.82 percent D. 267.38 percent E. 909.10 percent

C

73. Considering a put option; if the price of the underlying asset increases: a. the value of the put option also increases. b. the intrinsic value of the option increases. c. the value of the option decreases. d. the time value of the option decreases.

C

75. Deflation compounds information problems because it: A. Increases a company's net worth B. Tends to understate a company's assets and overstate their liabilities C. Reduces the dollar value of assets while the dollar value of liabilities stays constant D. Always harms lenders

C

76. A borrower who obtains funds from a lender to purchase additional inventory but uses the funds to finance a trip to Las Vegas for a weekend of gambling at the opening of a new casino is an example of: A. The problem of adverse selection B. The free-rider C. The moral hazard problem D. Lax government regulation

C

78. The principal-agent problem is: A. A form of adverse selection B. When stockholders are not acting in the best interest of managers C. A form of moral hazard D. Due to managers not being able to monitor stockholder behavior

C

84. A 6-month put has a strike price of $40. The underlying stock's price is $38.25. What is the intrinsic value of this put? A. $0.00 B. $0.90 C. $1.75 D. $2.30 E. $3.60

C

85. A 6-month put has a strike price of $32.50. The underlying stock's price is $31.10. What is intrinsic value of this put? A. $0.00 B. $0.70 C. $1.40 D. $2.10 E. $2.80

C

86. The moral hazard that can result from debt financing is mainly due to the: A. Borrower not working as hard once he or she obtains the loan B. Borrower wanting to refinance the loan C. Borrower taking greater risk in hopes of obtaining a larger return D. Economy turning sour and the borrower defaulting

C

90. A stock is valued at $25.75 a share. A European 6-month call option has a strike price of $25 and an option premium of $1.50. The market rate is 9.5 percent and the risk-free rate is 2.5 percent. What is the price of a European 6-month put option with a $25 strike price? A. $0.00 B. $0.09 C. $0.44 D. $1.48 E. $1.61

C

91. Often a bank will require a loan officer to make personal visits on customers with loans outstanding. This is encouraged because: A. The bank worries about another bank trying to steal their customers B. The bank wants to make sure the business is busy C. This is an effective monitoring technique and should reduce moral hazard D. The bank has excess funds available and hopes to make another loan to the business

C

94. A call option with 6 months to expiration currently sells for $2.05. A put option with the same expiration sells for $0.60. The options are European style. The risk-free rate is 3.0 percent and the strike price of both options is $50. What is the current stock price? A. $47.89 B. $49.19 C. $50.72 D. $51.29 E. $52.08

C

A 10-year Treasury note as a face value of $1,000, price of $1,200, and a 7.5% coupon rate. Based on this information, we know the: A. present value is greater than its price. B. current yield is equal to 8.33%. C. coupon payment on this bond is equal to $75. D. coupon payment on this bond is equal to $90.

C

A 30-year Treasury bond as a face value of $1,000, price of $1,200 with a $50 coupon payment. Assume the price of this bond decreases to $1,100 over the next year. The one-year holding period return is equal to: A. -9.17%. B. -8.33%. C. -4.17%. D. -3.79%.

C

A bank is a financial intermediary. Which of the following statements is most accurate? A. The bank's depositors are the ultimate lenders and the bank is the ultimate borrower. B. People seeking loans from the bank are the ultimate spenders while the bank is the ultimate lender. C. The bank's depositors are the ultimate lenders, while those seeking loans from the bank are the ultimate spenders. D. Those seeking loans from the bank are the ultimate spenders; the bank's stockholders are the ultimate lenders.

C

A bank's reserves include: A. U.S. Treasury bills. B. currency in the bank but not currency in the ATM machines. C. the bank's deposits at the Federal Reserve. D. U.S. Treasury bills and currency in the bank.

C

A borrower has information that is not available to a prospective lender; this is an example of: A. a wise borrower and an unwise lender. B. a transfer of risk. C. information asymmetry. D. liquidity risk.

C

A consol is: A. another name for a zero-coupon bond. B. a bond with a maturity date exceeding 10 years. C. a bond that makes periodic interest payments forever. D. a form of a bond that is issued quite often by the U.S. Treasury.

C

A derivative instrument: A. comes into existence after the underlying instrument is in default. B. is a low-risk financial instrument used by highly risk-averse savers. C. gets its value and payoff from the performance of the underlying instrument. D. should be purchased prior to purchasing the underlying security.

C

A financial instrument would include: A. only a written obligation and a transfer of value. B. only a written obligation and a specified date. C. a written obligation, a transfer of value, a future date, and certain conditions. D. a written obligation, a transfer of value, a specific date for payment, uncertain conditions.

C

A repurchase agreement is: A. an asset that represents the value of all collateral repossessed by the bank and held for sale. B. a long-term collateralized loan. C. an agreement where the parties agree to reverse the transaction on a specific day. D. only made between two or more banks.

C

A share of Microsoft stock would best be described as which of the following? A. A derivative instrument B. A means of payment C. An underlying instrument D. A debt instrument

C

Agencies exist which rate bonds based on characteristics of the borrower Such bond rating agencies are an example of a financial market response designed to: A. increase information asymmetry. B. decrease the real return to bondholders. C. provide a lower cost solution to the high cost of information. D. transfer risk from the buyer to the rating agency.

C

The largest of the regional Federal Reserve Banks is located in:

New York City

All of the following are depository institutions, except: A. commercial banks. B. credit unions. C. insurance companies. D. savings banks.

C

An increase in expected inflation for any given nominal interest rate will cause the: A. bond supply curve to shift to the left. B. bond demand curve to shift to the right. C. price of bonds to decrease. D. price of bonds to increase.

C

An increase in the nation's wealth, all other factors constant, would cause: A. bond prices to fall and yields to increase. B. bond prices and yields to increase. C. bond prices to rise and yields to decrease. D. the bond supply curve to shift right.

C

As general business conditions improve, we would witness the following in the bond market: A. the bond demand curve shifting left. B. the bond supply curve shifting left. C. bond prices decreasing. D. bond prices increasing.

C

Banks do not hold a lot of their assets in the form of cash mainly because of: A. regulation. B. the fear of being robbed. C. the opportunity cost of holding cash; cash does not earn interest. D. it can encourage employee theft.

C

Bond prices and yields: A. move together in the same direction. B. do not change if the coupon is fixed. C. move together inversely. D. are independent of each other.

C

Checkable deposits have decreased since the 1970's mainly because: A. regulators allowed higher rates to be paid on these accounts and banks found them to be highly unprofitable. B. people prefer to use credit cards rather than writing checks. C. these deposit accounts offer little or no interest so depositors find them to be expensive. D. as banks added fees to these accounts people increased their holdings of currency.

C

Consider the price paid for debt issued by the State of California. Which of the following would lead to a decrease in the value of State of California bonds? A. The State of California bonds are in small dollar amounts. B. The State of California bonds have a shorter maturity. C. The State of California experiences a fiscal crisis that makes it less likely it will be able to honor its interest payments. D. The State of California pays back its previous bonds ahead of schedule.

C

Considering the value of a financial instrument, the sooner the promised payment is made: A. the less valuable is the promise to make it since time is valuable. B. the greater the risk, therefore the promise has greater value. C. the more valuable is the promise to make it. D. the less relevant is the likelihood that the payment will be made.

C

Equity markets are markets: A. of U.S. Treasury bonds. B. for AAA rated bonds. C. for stocks. D. for either stocks or bonds.

C

Everything else equal, if the ratio of bank assets to bank capital increases, the bank's return on equity should: A. remain constant. B. decrease. C. increase. D. cannot be determined from the information provided.

C

Financial instruments are different from money because they: A. can act as a store of value and money cannot. B. can't be a means of payment but money can. C. can allow for the transfer of risk. D. have greater liquidity.

C

Financial instruments used primarily as stores of value do not include: A. asset backed securities. B. U.S. Treasury bonds. C. a car insurance policy. D. a bank loan.

C

If Bank A sells some its loans to Bank B for cash, everything else equal: A. Bank A's assets decrease and Bank B's assets increase. B. Bank A becomes less liquid while Bank B becomes more liquid. C. Banks A's total assets do not change, but Bank A is more liquid. D. Bank A's liabilities decrease by the amount of the loans that are sold.

C

If a bank has deposits of $250 million, reserves that total $30 million and has a required reserve rate of 10 percent: A. the bank is short of required reserves. B. the bank has excess reserves of $27.5 million. C. the bank has excess reserves of $5 million. D. the bank has excess reserves of $3 million.

C

Considering the methods available to the FDIC for dealing with a failed bank, the depositors of the failed bank should:

Prefer the purchase and assumption method since the deposits over $250,000 will also be protected

Cash Price (AKA: Spot price)

Price of a commodity or financial instrument quoted for current delivery

The primary objective of most central banks in industrialized economies is:

Price stability (Ch. 15)

If a bank sells off all of its assets and pays all of its liabilities, the amount remaining would be its: A. net profit. B. reserves. C. net worth. D. excess reserves.

C

If a bond's purchase price equals the face value the: A. coupon rate equals the current yield, which is less than the yield to maturity. B. current yield equals the yield to maturity, which exceeds the coupon rate. C. coupon rate equals the yield to maturity, which equals the current yield. D. coupon rate does not equal the current yield, which does not equal the yield to maturity.

C

If a one-year zero-coupon bond has a face value of $100, is purchased for $94, and is held to maturity the: A. holding period return will exceed the yield to maturity. B. yield to maturity will exceed the holding period return. C. yield to maturity will be 6.38%. D. holding period return is 6.0%.

C

If bank with $100 million in assets and $10 million in equity increases its assets by adding $1 to capital for every $1 added to assets: A. the debt-to-equity ratio will increase. B. the debt-to-equity ratio will remain constant. C. the debt-to-equity ratio will decrease. D. the answer cannot be determined from the information in the question.

C

If the U.S. government's borrowing needs increase, all other factors constant the: A. demand for bonds will decrease. B. price of bonds will increase. C. supply of bonds will increase. D. yields on bonds will decrease.

C

If the annual interest rate is 5%(.05), the price of a one-year Treasury bill per $100 of face value would be: A. $95.00 B. $97.50 C. $95.24 D. $96.10

C

If the federal government were to offer larger tax breaks on the purchase of new equipment for businesses, all other factors constant, we would expect to see the: A. bond demand curve shift right. B. bond supply curve shift left. C. bond supply curve shift right. D. bond demand curve shift left.

C

If the purchase price of a bond exceeds the face value, the yield to maturity: A. is greater than the coupon rate because the capital gain is positive. B. will equal the current yield. C. will be less than the coupon rate because the capital gain will be negative. D. will be greater than the current yield.

C

If the risk on foreign government bonds increases relative to U.S. government bonds, the price of U.S. government bonds should: A. not change since U.S. government bonds are free of default risk. B. decrease since people will bail out of all government bonds. C. increase as the demand for these bonds increases. D. not be affected because the two types of bonds are traded in different markets.

C

In reading bond quotes: A. the bid price is usually above the asked price. B. the asked price is fixed over the life of the bond. C. the asked price is usually above the bid price. D. bid and asked prices must be equal as set forth by SEC regulations.

C

Loans made between borrowers and lenders are: A. usually not taxable at the federal level. B. legal only in the state of origination. C. assets of the lenders. D. assets of the borrowers.

C

Loans made in the federal funds market: A. are highly collateralized. B. are made by the Federal Reserve System to the bank within 24 hours. C. are unsecured loans. D. are insured by the FDIC.

C

Mergers resulting from the financial crisis of 2007-2009 have left what percentage of deposits in the hands of 4 banks? A. 10% B. 30% C. 40% D. 60%

C

Most of the buying and selling in primary markets: A. is in the public view. B. is highly transparent and closely monitored by the SEC. C. involve an investment bank. D. is done by the Federal Reserve.

C

Net interest income for a bank is: A. the difference between gross income and net income after taxes. B. the interest banks earn from uses of funds. C. the difference between interest income and interest expense. D. the difference between interest income and total expenses.

C

Of the more than 6,100 banks in the United States at the end of 2013, by far the greatest numbers of them were: A. regional banks. B. money center banks. C. community banks. D. savings banks.

C

Small savers would rather use financial institutions than lend directly to borrowers because: A. financial institutions will offer the savers higher interest rates than the savers could obtain directly from borrowers. B. lenders wouldn't want to deal with small savers. C. it allows them to diversify risk. D. the liquidity is lower with financial institutions but the return is higher.

C

Standardization of financial instruments has occurred as a result of: A. the rule of 70. B. the law of demand. C. economies of scale. D. the law of supply.

C

Suppose that the expected return on bonds falls relative to other assets. In the bond market this will result in: A. the bond supply curve shifting left. B. a movement down the bond demand curve. C. a shift to the left of the bond demand curve. D. an increase in the price of bonds.

C

Suppose that the return on assets other than bonds falls. In the bond market this will result in a(n): A. movement down the bond demand curve. B. shift to the left of the bond demand curve. C. increase in the price of bonds. D. shift to the left of the bond supply curve.

C

The bond supply curve slopes upward because: A. as bond prices rise people holding bonds are more tempted to hold them. B. as bond prices rise yields increase. C. for companies seeking financing, the higher the price of bonds the more attractive it is to sell bonds. D. as bond prices rise yields decrease.

C

The credit risk a bank faces is the risk resulting specifically from: A. the economy entering a recession. B. interest rates falling. C. some of the bank's loans not being repaid. D. the bank experiencing a decrease in deposits.

C

The current yield of a bond: A. is another term for the coupon rate. B. is another term for the yield to maturity. C. equals zero for a zero-coupon bond since these bonds have no coupon payments. D. is the difference between its future value and its present value.

C

The demand for U.S. government bonds is high relative to other bond issues because: A. liquidity of other bond issues is high relative to U.S. government bonds. B. U.S. bond market has low transaction spreads due to high illiquidity. C. market for U.S. government bonds is more liquid than most if not all other bond markets. D. U.S. government bonds have higher default.

C

The difference in the prices of a zero-coupon bond and a coupon bond with the same face value and maturity date is simply: A. zero, since they are the same. B. the present value of the final payment. C. the present value of the coupon payments. D. the future value of the coupon payments.

C

The federal funds market: A. is the term used for bank borrowing from the Federal Reserve System. B. is the lending to banks by the U.S. treasury when banks face liquidity emergencies. C. is the inter-bank market where excess reserves from one bank can be loaned to another bank. D. is the borrowing by American banks from foreign lenders.

C

The fundamental characteristics influencing the value of a financial instrument include each of the following except: A. the size of the payment promised. B. when the promised payment will be made. C. where the instrument is traded. D. the likelihood of payment.

C

The market for bonds is initially described by the supply of bonds - S0, and the demand for bonds - D0, with the equilibrium price and quantity being P0 and Q0. If the U.S. government's borrowing needs decrease, all other factors constant: A. Bond supply curve to shift to S1 B. Bond demand curve to shift to D1 C. Bond supply curve to shift to S2 D. Bond demand curve to shift to D2

C

The relationship between the price and the interest rate for a zero coupon bond is best described as: A. volatile. B. fluctuating. C. inverse. D. non-existent.

C

The size of the bond dealer's spread is mainly a function of the: A. purchase price of the bond. B. current yield. C. liquidity of the bond market. D. face value of the bond.

C

Which of the following is a problem of adverse selection?

The lender has a problem of distinguishing good risk from bad risk borrowers.

The tendency for large banks to have a higher return on equity than small banks suggests: A. small banks have better management than large banks. B. large banks can charge higher interest rates than small banks. C. there could be significant economies of scale in banking. D. larger banks are better able to escape the cost of regulation.

C

The total assets of commercial banks in 2013 amounted to: A. three times nominal GDP in the U.S. B. about one-half of nominal GDP in the U.S. C. about four-fifths of nominal GDP in the U.S. D. about one-tenth of nominal GDP in the U.S.

C

The ultimate role of the financial system of a country is to: A. provide a place for wealthy households to save. B. be a low-cost source of funds for government. C. facilitate production, employment, and consumption. D. provide jobs in the financial sector.

C

Today the primary distinction between direct and indirect finance is in: A. direct finance the asset holder has a claim on a financial institution while in indirect finance the asset holder has a direct claim on the borrower. B. indirect finance the lender has a direct claim on the borrower while in direct finance the lender has a claim on a financial institution. C. direct finance the asset holder has a direct claim on the borrower while in indirect finance the asset holder has a claim on a financial institution. D. indirect finance the asset holder has a claim on the government while in direct finance the asset holder has a direct claim on a private sector corporation.

C

Tom obtains a car loan from Old Town Bank. A. The car loan is Tom's asset and the bank's liability. B. The car loan is Tom's asset, but the liability belongs to the bank's depositors. C. The car loan is Tom's liability and an asset for Old Town Bank. D. The car loan is Tom's liability and a liability of the bank until Tom pays it off.

C

U.S. government bonds that provide for bondholders to receive a fixed rate of interest plus the change in the consumer price index were designed to remove: A. default risk. B. liquidity risk. C. inflation risk. D. interest-rate risk.

C

Well-run financial markets: A. keep transactions costs high to benefit brokers B. prevent the widespread pooling of information C. ensure that resources are allocated efficiently D. are usually the result of little or no government regulation

C

When the price of a bond equals the face value the: A. yield to maturity will be above the coupon rate. B. yield to maturity will be below the coupon rate. C. current yield is equal to the coupon rate. D. yield to maturity is greater than the current yield.

C

Which of the following best expresses the equation for holding period return? A. Current yield + coupon rate B. Yield to maturity - current yield C. Current yield + capital gain D. Coupon rate + capital gain

C

Which of the following is a bank asset? A. Demand deposits B. Borrowings from other banks C. Mortgage loans D. CDs

C

Which of the following is likely to be a primary financial market transaction? A. You cash the check your grandmother sent you for your birthday. B. You call a broker and purchase bonds for your retirement fund. C. A city issues bonds to finance new road construction. D. A supermarket needs to borrow the funds for a second location and takes out a loan from a commercial bank to pay for it.

C

Which of the following is not a financial intermediary? A. A bank B. An insurance company C. The New York Stock Exchange D. A mutual fund

C

Which of the following statements best completes this sentence: "On a bank's balance sheet..."? A. assets show the sources of funds and the net worth shows the uses of funds. B. net worth shows the sources of funds and liabilities show the uses of funds. C. assets show the uses of funds and liabilities show the sources of funds. D. net worth represents both a source and a use of funds.

C

Which of the following statements regarding checkable deposits is most accurate? A. Checkable deposits are a larger source of bank funds today than in 1970. B. Checkable deposits are no longer a source of bank funds. C. Checkable deposits are a less important source of bank funds today than in 1970. D. Checkable deposits continue to be the largest source of bank funds.

C

4) You are comparing two investment options that each pay 6 percent interest, compounded annually. Both options will provide you with $12,000 of income. Option A pays $2,000 the first year followed by two annual payments of $5,000 each. Option B pays three annual payments of $4,000 each. Which one of the following statements is correct given these two investment options? Assume a positive discount rate. (No calculations needed.) A) Both options are of equal value since they both provide $12,000 of income. B) Option A has the higher future value at the end of Year 3. C) Option B has a higher present value at Time 0. D) Option B is a perpetuity. E) Option A is an annuity.

C) Option B has a higher present value at Time 0.

The tendency for large banks to have a higher return on equity than small banks suggests:

There could be significant economies of scale in banking.

15) Which one of the following statements concerning interest rates is correct? A) Savers would prefer annual compounding over monthly compounding given the same annual percentage rate. B) The effective annual rate decreases as the number of compounding periods per year increases. C) The effective annual rate equals the annual percentage rate when interest is compounded annually. D) Borrowers would prefer monthly compounding over annual compounding given the same annual percentage rate. E) For any positive rate of interest, the annual percentage rate will always exceed the effective annual rate.

C) The effective annual rate equals the annual percentage rate when interest is compounded annually.

20) A(n) ________ loan has regular payments that include both principal and interest but these payments are insufficient to pay off the loan. A) perpetual B) continuing C) balloon D) pure discount E) interest-only

C) balloon

12) The actual interest rate on a loan that is compounded monthly but expressed as an annual rate is referred to as the ________ rate. A) stated B) discounted annual C) effective annual D) periodic monthly E) consolidated monthly

C) effective annual

8) An ordinary annuity is best defined as: A) increasing payments paid for a definitive period of time. B) increasing payments paid forever. C) equal payments paid at the end of regular intervals over a stated time period. D) equal payments paid at the beginning of regular intervals for a limited time period. E) equal payments that occur at set intervals for an unlimited period of time.

C) equal payments paid at the end of regular intervals over a stated time period.

22) With an interest-only loan the principal is: A) forgiven over the loan period; thus it does not have to be repaid. B) repaid in decreasing increments and included in each loan payment. C) repaid in one lump sum at the end of the loan period. D) repaid in equal annual payments. E) repaid in increasing increments through regular monthly payments.

C) repaid in one lump sum at the end of the loan period.

You have savings accounts at two separately FDIC insured banks. At one of the banks your account has a balance of $200,000. At the other bank the account balance is $60,000. If both banks fail, you will receive: A. $250,000. B. $60,000. C. $260,000. D. $200,000.

C. $260,000

assume we have a stock currently worth $100. We also assume the interest rate is zero, and we can buy option for this stock with a strike price of $100. if the stock can rise or fall by $20 with equal probability over the option period, and the option cannot be exercised until the expiration date, what is the time value of the option?

C. 10 20X.5

The need for a lender of last resort was identified as far back as: A. the start of the Great Depression in 1929. B. 1913, when the Federal Reserve was created. C. 1873, by British economist Walter Bagehot. D. 1776, by the first U.S. Secretary of the Treasury, Alexander Hamilton.

C. 1873, by British economist Walter Bagehot

You have a portfolio valued at $1000. over the next 12 months it loses 75% of its value. What return does the portfolio need to earn over the following 12 months to restore the portfolio to its original value?

C. 300% 1000 x .25 = 250 x 3.0 = 750 + 250 = 1000

Following the consolidation that resulted from the 2007-2009 financial crisis in the U.S., the 4 largest commercial banks share of total deposits was: A. 75%. B. 50%. C. 40%. D. 25%.

C. 40%

You start with a $1,000 portfolio; it loses 50% over the next year, the following year it gains 50% in value. At the end of two years your portfolio is worth:

C. 750 1000/.5= 500 x 1.5 = 750

If the DIJA is at 10,205 and it is up 4% from the previous day, what was the index at the close of the previous day?

C. 9,812.5 (10,205/1.04)

A bank run involves: A. illegal activities on the part of the bank's officers. B. a bank being forced into bankruptcy. C. a large number of depositors withdrawing their funds during a short time span. D. a bank's return on assets being below the acceptable level.

C. A large number of depositors withdrawing their funds during a short time span.

The financial system is inherently more unstable than most other industries due to the fact that: A. while in most other industries customers disappear at a faster rate, in banking they disappear slowly so the damage is done before the real problem is identified. B. banks deal in paper profits, not in real profits. C. a single firm failing in banking can bring down the entire system; this isn't true in most other industries. D. there is less competition than in other industries.

C. A single firm failing in banking can bring down the entire system; this isn't true in most other industries

The inter-bank loans that appear on banks' balance sheets represent what proportion of bank capital? A. Nearly ten percent B. Almost three-fourths C. About one-third D. Less than one percent

C. About one third

The financial crisis of 2007-2009 has made which of the following regulatory goals a top priority for government: A. disclosure of accounting information. B. minimum capital requirements. C. avoidance of systemic risk. D. promotion of competition.

C. Avoidance of systemic risk

Bank panics have often begun as a result of: A. rumors only. B. real economic events only. C. both rumors and real economic events. D. neither rumors nor economic events.

C. Both rumors and real economic events

the right to buy a given quantity of an underlying asset at a predetermined price on or before a specified date is called a(n):

C. Call option

You have savings accounts at two separately FDIC insured banks. At one of the banks your account has a balance of $200,000. At the other bank the account balance is $60,000. You find out the banks are going to merge. If you are concerned about the possibility of the new bank failing, you should: A. do nothing; you are still insured up $250,000 per account. B. consider moving $10,000 to another account at the same bank. C. consider moving $10,000 to another account at a different bank. D. do nothing; as an individual you are only insured up $250,000 no matter where the accounts are.

C. Consider moving $10,000 to another account at a different bank

If the lender of last resort function of the government is to be effective in working to minimize a crisis, it must be: A. reserved only for those banks that are most deserving. B. used on a limited basis. C. credible, with banks knowing they can get loans quickly. D. only available during economic downturns.

C. Credible, with banks knowing they can get loans quickly

One negative consequence of regulatory competition is: A. it is expensive. B. financial institutions are over regulated at a cost to customers. C. financial institutions often seek out the most lenient regulator. D. it minimizes competition.

C. Financial institutions often seek out the most lenient regulator

Under the purchase-and-assumption method, the FDIC usually finds it: A. can sell the failed bank for more than the bank is actually worth. B. can sell the bank at a price equaling the value of the failed banks assets. C. has to sell the bank at a negative price since the bank is insolvent. D. cannot sell the bank and almost always has to revert to the payoff method for dealing with a failed bank.

C. Has to sell the bank at a negative price since the bank is insolvent.

The federal funds market: a. Is the term used for bank borrowing from the Federal Reserve System? b. Is the lending to banks by the U. S. treasury when banks face liquidity emergencies? C. Is the inter-bank market where excess reserves from one bank can be loaned to another bank? d. Is the borrowing by American banks from foreign lenders?

C. Is the inter-bank market where excess reserves from one bank can be loaned to another bank?

The government regulates bank mergers, sometimes denying the proposed merger. Often the reason given for the denial is to protect small investors. What are small investors being protected from? A. with a larger bank the bank is likely to take greater risk and may fail. B. in order to pay for the merger, the bank may seek higher returns putting the depositors' funds at greater risk. C. mergers can increase the monopoly power of banks and the bank may seek to exploit this power by raising prices and earning unwarranted profits. D. bank runs hurt larger banks more than smaller banks.

C. Mergers can increase the monopoly power of banks and the bank may seek to exploit this power by raising prices and earning unwarranted profits

It is difficult for depositors to know the true health of banks because: A. regulations prohibit banks making their financial statements publicly available. B. the financial statements of banks are too difficult for most people to understand. C. most of the information on bank loans is private and based on sophisticated models. D. banking is competitive and financial records of banks are not divulged to prevent competitor banks from having an advantage.

C. Mot of the information on bank loans is private and based on sophisticated models.

The payoff method used by the FDIC to address the insolvency of a bank is when the FDIC: A. pays the owners of the bank for the losses they would otherwise face. B. pays off all depositors the balances in their accounts so no depositor suffers a loss, though the owners of the bank may suffer losses. C. pays off the depositors up to the current $250,000 limit, so it is possible that some depositors will suffer losses. D. takes all of the assets of the bank, sells them, pays off the liabilities of the bank, in full and then replenishes their fund with any remaining balance.

C. Pays off the depositors up to the current $250,000 limit, so it is possible that some depositors will suffer losses

Bank failures tend to occur most often during periods of: A. stock market run ups when, like many companies, banks tend to be overvalued. B. high inflation when the fixed rate loans of many banks cause their real returns to decrease. C. recessions when many borrowers have a difficult time repaying loans and lending activity slows. D. wars and other civil unrest.

C. Recessions when many borrowers have a difficult time repaying loans and lending activity slows

Implicit government support for "too big to fail" banks: A. increases the scrutiny of the bank's risk by large corporate depositors. B. reduces the risk faced by depositors with accounts less than $250,000. C. reduces the risk faced by depositors with accounts exceeding $250,000. D. reduces the moral hazard problem of insuring large banks.

C. Reduces the risk faces by depositors with accounts exceeding $250,000

The fact that banks can be either nationally or state chartered creates: A. situations where some banks go unregulated. B. situations where banks operating in more than one state can escape regulation. C. regulatory competition. D. banks being simultaneously regulated by more than one agency.

C. Regulatory competition

Deposit insurance only seems to be viable at the federal level. This is likely due to the fact that: A. state funds are less informed about the solvency of national banks. B. a run on the banks within a state will always spread countrywide. C. the U.S. Treasury backs the FDIC and can therefore withstand virtually any crisis. D. the cost of state insurance is prohibitively high.

C. The U.S. Treasury backs the FDIC and can therefore withstand virtually any crisis

The Dow Jones Industrial Average is:

C. The average price of stock in 30 of the largest companies in the U.S.

What matters most during a bank run in: A. the number of loans outstanding. B. the solvency of the bank. C. the liquidity of the bank. D. the size of the bank's assets.

C. The liquidity of the bank.

Which of the following regulates commercial banks as well as savings banks and savings and loans? A. The Federal Reserve System B. Securities and Exchange Commission C. The Office of the Comptroller of the Currency D. The Internal Revenue Service

C. The office of the Comptroller of the Currency

the value of a derivative is determined by:

C. The value of the underlying asset

Which of the following is not a goal of the Dodd-Frank Act of 2010? A. To anticipate and prevent financial crises by limiting systemic risk B. To end "too big to fail" C. To promote competition D. To reduce moral hazard

C. To promote competition

In a derivative transaction:

C. What one person gains is what the other person loses

An option's value will never be less than 0 because:

C. an option holder will never make an additional payment to exercise the option

The concept of limited liability says a stockholder of a corporation:

C. cannot lose more than his/her investment

People differ on the method by which stock should be valued. Some people are chartists, others behavioralists. The basic difference between these groups is:

C. chartists study charts of stock prices; behavioralists focus on investor psychology and behavior

with a put option, the option holder:

C. has the right to sell the asset

Emperical evidence points to the fact that financial crises: A. are newsworthy but have no impact on economic growth. B. have a negative impact on economic growth only for the year of the crisis. C. have a negative impact on economic growth for years. D. can have a positive impact on economic growth as weak borrowers are weeded out.

C. have a negative impact on economic growth for years

One key difference between options contracts and futures contracts is:

C. in an options contract both parties have equal rights

As the volatility of the stock price increases, the time value of the option:

C. increases

the intrinsic value of the call option:

C. is the greater of zero or the difference between the price of the underlying asset and the strike price

at expiration, the time value of an option:

C. is zero

An index number is valuable because:

C. it provides a meaningful measurement scale to calculate percentage changes

Sue buys a futures contract for U.S treasury bonds and on the settlement date the interest rate is higher than expected. Sue will have:

C. lost money on her long position

The Nasdaq Composite Index is:

C. made up of mainly newer firms, and heavily influenced by technology and internet companies

Banking regulations prevent banks from: A. holding more than 10 percent of their assets in common stock of companies. B. owning corporate jets. C. owning common stocks of corporations. D. building big office buildings.

C. owning common stocks of corporations.

In today's world, the goal of financial stability means: A. no institution should fail. B. competition should be eliminated. C. preventing large-scale financial catastrophes. D. creating one mega regulatory agency.

C. preventing large-scale financial catastrophes

Prior to the financial crisis of 2007-2009 banks did all but which of the following to bulk up their profit: A. bought or sponsored hedge funds. B. traded securities for customers. C. purchased equities for their own account. D. colluded to fix benchmark interest rates.

C. purchased equities for their own account.

A share of common stock represents a(n): a) claim from a lender against a borrower b) share in the company's debts c) share of ownership in the company d) unlimited liability to the owner of the stock

C. share of ownership in the company

The option holder is:

C. the buyer of the option

on the settlement date of a futures contract:

C. the future's price is equal to the price of the underlying asset

if we have a stock selling for $95 and a call option for this stock has a strike price of $82 and an option price of $13.6:

C. the intrinsic value of the option is 13 and the time value of the option is .60

The strike price of an option is:

C. the price at which the option holder has the right to buy or sell

The right to buy a given quantity of an underlying asset at a predetermined price on or before a specific date is called a(n):

Call option.

8. Identify four factors that will cause the value of call options to increase.

Call options will increase in value if there is a decrease in the strike price; if there is an increase in the market price of the underlying asset, if there is an increase in the time to expiration, or if there is an increase in the volatility in the price of the underlying asset.

The concept of limited liability says a stockholder of a corporation:

Cannot lose more than his/her investment.

Consider a $1,000 face value bond with a $55 coupon payment and 1 year to maturity. Calculate the current yield, coupon rate and the yield to maturity if the bond is purchased for $1,130.

Current yield = 4.87%, coupon rate = 5.50%, yield to maturity = -6.64%.

10. A cash-settled option is defined as an option which does which one of the following? A. requires a cash deposit upon purchase B. has a foreign currency as its underlying asset C. has the U.S. dollar at its underlying asset D. entails a cash payment to the holder upon exercise E. offers the option to either deliver the underlying asset or a cash payment

D

12. An option that would NOT yield a positive payoff if exercised today is referred to by which one of the following terms? A. hollow option B. zero option C. in-the-cellar option D. out-of-the-money option E. strike-out

D

12. Users of commodities are: a. usually not participants in futures contracts. b. speculators preferring to get the large returns which result from large risk. c. likely to take the short position in a futures contract. d. buyers of futures.

D

16. As the time of settlement gets closer: a. the price of the futures contract will diverge from the price of the underlying asset. b. the price of the futures contract will always be above the price of the underlying asset. c. the price of the underlying asset and the future's price will show no correlation at all. d. the price of the futures contract will move in lockstep with the price of the underlying asset.

D

2. Financial intermediation exists, in part, because: A. Financial markets work so well B. Direct finance through stocks and bonds is the dominant form of financing C. Transaction costs of financial intermediation is always higher than direct finance D. The transaction costs associated with direct finance can at times be prohibitive

D

2. The purpose of derivatives is to: a. increase the risk so the return is larger. b. eliminate risk for both parties in the transaction. c. postpone the risk for both parties in the transaction. d. transfer the risk from one person to another.

D

21. An option trading strategy that utilizes both put and call options is referred to as which one of the following? A. bull call spread B. butterfly spread C. split D. combination E. counteraction

D

22. A bank can usually offer a saver a higher return for the same risk for all of the following reasons except: A. The bank can usually purchase assets at a lower cost than any one saver B. The bank can pool the resources of small savers and purchase higher valued assets C. Economies of scale can also be applied by the bank in its purchase of assets D. Savers do not have good enough information to know if the return is sufficient

D

22. An arbitrageur is someone who: a. always takes the long position in a futures contract. b. always takes the short position in a futures contract. c. seeks the high returns that come from the high risk inherent in futures markets. d. simultaneously buys and sells financial instruments to benefit from temporary price differences.

D

26. Mutual funds offer investors: A. A greater return for greater risk than what an investor can earn on his own B. A lower return for more risk than what the investor could earn on his own C. A lower return for less risk than what the investor could earn on his own D. A way for individuals to eliminate the idiosyncratic risk associated with any single investment

D

27. You are buying the June call on General Electric stock at $0.19. What amount will you pay per share if you decide to exercise this option? A. $32.50 B. $32.69 C. $33.81 D. $34.00 E. $34.19

D

29. A lender usually knows less about the creditworthiness of a borrower than the borrower does. This is an example of: A. Opportunistic behavior B. Economies of scale C. Diminishing marginal returns D. Information asymmetry

D

3. A pure discount bond is also known as a: A. consol. B. fixed payment loan. C. coupon bond. D. zero-coupon bond.

D

31. Financial intermediaries reduce the problems in lending associated with information asymmetries by all of the following except: A. Collecting and processing standardized information B. Screening applicants to be sure they are creditworthy C. Monitoring loan recipients to be sure the funds are used properly D. Charging interest rates high enough to discourage undesirable borrowers

D

35. Which one of the following combinations creates an in-the-money option? A. underlying stock price is less than the strike price of a call B. underlying stock price is $18 and the put has an exercise price of $15 C. underlying stock price is $22 and the call has an exercise price of $25 D. put strike price exceeds the underlying stock price E. put price is equal to the call price

D

36. Which of the following statements is true? a. Call options can be sold prior to expiration but put options cannot. b. Put options can be sold prior to expiration but call options cannot. c. No option can be sold prior to expiration. d. Both American and European options can be sold prior to expiration.

D

38. Someone who purchases a call option is really buying insurance to protect against: a. the stock not being available when they want to purchase it. b. the price of the stock falling. c. a seller not being able to deliver the stock. d. the price of the stock rising.

D

40. In a financial market where information is symmetric: A. All information would be known by both parties in a transaction B. One party to a transaction knows information the other party does not C. The ability to obtain information is available to only one party D. There would be no adverse selection

D

42. The time value of the option can best be defined as a. the commission earned by a broker. b. the fee earned for the potential benefits from buying the option. c. the service fee charged by the SEC for regulating the option market. d. the fee paid for the potential benefits from buying an option (excluding its intrinsic value).

D

44. Assume we have a stock currently worth $100. We also assume the interest rate is zero, and we can buy options for this stock with a strike price of $100. If the stock can rise or fall by $5 with equal probability over the option period, and the option cannot be exercised until the expiration date, what is the time value of the option? a. $10 b. $5 c. $0 d. None of the answers is correct.

D

46. Mary Jones is the president of a local bank. She knows that half of the loan applicants in town she would classify as high risk and the other half as low risk. She observes that the other banks in town charge two different interest rates, a lower rate for low risk borrowers and the higher rate for high risk borrowers. She decides that to have an advantage over the other banks she will offer an average rate to everyone. The likely result will be: A. Mary's bank will be highly successful as this will provide the bank with a large competitive advantage B. Mary's bank is likely to see a dramatic increase in both types of borrowers C. Mary's bank will experience adverse selection and have a disproportionate number of low risk borrowers D. Mary's bank will experience adverse selection and have a disproportionate number of high risk borrowers

D

5. Which one of the following distinguishes an option as an American style option? A. option that grants its holder the right to purchase at the strike price B. option that grants its holder the right to sell at the strike price C. option that obligates its holder to sell at the strike price D. option that can be exercised at any time prior to expiration E. option that can only be exercised at expiration

D

51. Which one of the following is the upper price bound for the intrinsic value of a European call option on a stock? A. $0 B. strike price C. stock price D. Max (S - K, 0) E. Max (K - S, 0)

D

53. The principal in an interest rate swap is: a. always transferred from the originator to the counter party of the swap. b. is usually held by a clearinghouse to guarantee payment. c. usually borrowed from a third party. d. is not borrowed, lent, or exchanged. It just serves as the basis for the calculation of cash flows.

D

54. Adverse selection: A. Increases the efficiency of most markets B. Usually causes prices to adjust faster than they otherwise would C. Makes it easier for all customers to find what they want D. Results in fewer market transactions

D

55. Which one of the following represents an arbitrage opportunity? A. stock price of $18 and strike price of $20 B. call price of $0.40 and put price of $0.40 C. PCP-implied put price of $0.30 and call price of $0.28 D. PCP-implied put price of $0.30 and put market price of $0.31 E. PCP-implied call price of $0.20 and a put market price of $0.22

D

56. One key difference between swaps and option contracts is: a. swaps are derivative agreements and options are not. b. swaps do not involve any risk and options do. c. options transfer risk, swaps create risk. d. options trade on organized exchanges and swaps do not.

D

56. What is the total option premium you will receive if you sell 6 October $25 calls on Texas Instruments? ============== A. $80 B. $350 C. $695 D. $4,170 E. $4,375

D

65. You own one SPX call option with a strike of 1,400. What is the payoff at maturity for this option contract if the S&P 500 index is 1,414? A. $0 B. $14 C. $140 D. $1,400 E. $14,000

D

67. Which of the following could be the lemons problem, applied to financial markets? Explain A. Lenders seeing a disproportionate share of high quality loan applicants B. An average interest rate that is too high for the actual risk obtained C. Profits for many lenders increasing significantly D. High quality potential borrowers relying more on internally generated funds to finance investment

D

70. You purchased a call option with a $22.50 strike price and a call premium of $0.40. On the expiration date, the underlying stock was priced at $23.40 per share. What is the percentage return on your investment? A. -100 percent B. 0 percent C. 50 percent D. 125 percent E. 200 percent

D

76. Jeff paid a call premium of $0.60 when he purchased his call option with a strike price of $22.00. What is the break-even stock price? A. $0.00 B. $0.25 C. $22.25 D. $22.60 E. $22.75

D

77. Credit may dry up at the start of an economic downturn because of all of the following except: A. Lenders require information and accurate information is more difficult to obtain B. It becomes more difficult for lenders to determine the creditworthiness of borrowers C. Lenders see greater risk in making loans to borrowers D. The free-rider problem worsens during a downturn

D

77. Russ paid a total of $75 to purchase 5 call options with a strike price of $17.50. What is the break-even stock price? A. $0.15 B. $0.30 C. $17.35 D. $17.65 E. $32.50

D

79. The principal-agent problem is quite common in large public corporations due to: A. The fact that large corporations generate large sales volumes B. The fact that large companies employ many people C. Too little regulation by government D. The fact that the people making the operational decisions are usually not the owners

D

8. Which one of the following guarantees that the terms of an exchange-listed option contract are fulfilled when an option is exercised? A. Securities and Exchange Commission B. Federal Reserve C. New York Options Exchange D. Options Clearing Corporation E. Securities Investors Protection Corporation

D

81. Tom borrows $100,000 from his local bank to purchase inventory for his store for the upcoming holiday season. Tom's neighbor tells him about a get-rich-quick scheme that can take this $100,000 and triple it in a month. Tom decides to buy into this scheme figuring he can repay the bank and still have plenty left for inventory. This is an example of: A. Adverse selection B. Sound risk analysis on Tom's part C. Diversification D. Moral hazard

D

87. Each of the following is an example of a restrictive covenant on a mortgage loan, except: A. Net worth requirements B. Requiring that the borrower reside in a home for which he or she receives a mortgage C. Insisting the borrower carry physical damage insurance on the property securing the loan D. Requiring the borrower to obtain comprehensive health insurance

D

88. A European call has a strike price of $37.50. The underlying stock's price is $38.20. What is the lower price bound of this call? A. $0.00 B. $0.30 C. $0.50 D. $0.70 E. $1.00

D

93. A stock is currently selling for $40.85. A 3-month call option with a strike price of $40 has an option premium of $1.30. The risk-free rate is 2 percent and the market rate is 9.5 percent. What is the option premium on a 3-month put with a $40 strike price? Assume the options are European style. A. $0.00 B. $0.05 C. $0.15 D. $0.25 E. $0.35

D

95. A call option with 1 month to expiration currently sells for $0.70. A put option with the same expiration sells for $1.10. The options are European style. The risk-free rate is 3 percent and the strike price of both options is $18.00. What is the current stock price? A. $16.87 B. $17.06 C. $17.29 D. $17.56 E. $17.86

D

A $1000 face value bond, with one year to maturity that sells for $950 and has a $40 annual coupon has a: A. current yield and yield to maturity of 4.00%. B. yield to maturity that equals the current yield. C. coupon rate of 4.00% and a current yield that is below this. D. current yield of 4.21%.

D

A bank that specializes in granting loans to firms in a specific line of business: A. may decrease its operating cost and decrease its credit risk. B. may increase both its operating cost and its credit risk. C. may increase its operating cost and decrease its credit risk. D. may decrease its operating costs and increase its credit risk.

D

A collection of assets is known as a(n): A. asset-backed security. B. derivative. C. futures contract. D. portfolio.

D

A primary financial market is: A. a market just for corporate stocks. B. a market only for AAA rated Securities. C. the New York Stock Exchange. D. one in which newly issued securities are sold.

D

An increase in the nation's wealth, all other factors constant, would cause the: A. bond supply curve to shift left. B. bond demand curve to shift left. C. bond supply curve to shift right. D. bond demand curve to shift right.

D

Bank's hold marketable securities as part of their assets. For U.S. banks these marketable securities include: A. stocks and bonds. B. only the stocks of U.S. corporations. C. only the bonds of the U.S. treasury. D. only bonds.

D

Considering the balance sheet for all commercial banks in the U.S., the largest category of assets is: A. cash items. B. U.S. Government Securities. C. required reserves. D. loans.

D

Debt instruments that have maturities less than one year are traded in the: A. primary market exclusively. B. bond markets exclusively. C. bond market if they are already in existence. D. money market.

D

Financial instruments used primarily to transfer risk would include all of the following, except: A. an insurance contract. B. a futures contract. C. options. D. a bank loan.

D

Financial intermediaries pool funds of: A. many small savers and provide it to a few large borrowers. B. few large savers and provide it to many small borrowers. C. few large savers a few large borrowers. D. many small savers and provide it to many borrowers.

D

If a bank has $200 million in deposits, the required reserve rate is 10 percent and the bank has $23 million in reserves: A. the bank is short of required reserves. B. the bank has excess reserves of $21 million. C. the bank has excess reserves of $13 million. D. the bank has excess reserves of $3 million.

D

If financial markets didn't exist: A. required returns would be lower since fewer instruments would trade. B. liquidity would diminish and returns would be lower. C. more funds would flow directly between borrowers and savers. D. liquidity would diminish, reducing the flow of funds between borrowers and savers.

D

If interest rates are expected to fall, bond prices will: A. fall as the demand for bonds decreases. B. remain constant until interest rates actually change. C. fall as people fear capital losses in the future. D. increase due to the demand for bonds increasing.

D

If the U.S. government's borrowing needs decrease, all other factors constant the: A. supply of bonds will increase. B. demand for bonds will decrease. C. price of bonds will decrease. D. price of bonds will increase.

D

If the U.S. government's borrowing needs increase, in the bond market this would be seen as: A. the bond demand curve shifting right. B. a movement up the bond supply curve. C. the bond demand curve shifting left. D. the bond supply curve shifting right.

D

Interest-rate risk would not matter to which of the following bondholders? A. A holder of a U.S. government bond. B. A holder of a U.S. government bond indexed for inflation. C. A holder of a U.S. government bond who plans on selling it in one year. D. A holder of a U.S. government bond that plans on holding it until it matures.

D

Loans made between lenders and borrowers are: A. assets to the borrowers. B. liabilities of the lenders. C. not taxable in the state of origination. D. liabilities of the borrowers.

D

Many people believed that when the calendar changed from December 31, 1999 to January 1, 2000, many bank records were going to be wiped out, so many people planned on withdrawing their funds. If this were to happen, this would be an example of: A. credit risk. B. operational risk. C. interest rate risk. D. liquidity risk.

D

Money markets are where trades occur for: A. stocks. B. bonds of all maturities. C. derivatives. D. short-term bonds issued by both governments and private companies.

D

Most home mortgages are good examples of: A. consols. B. zero-coupon bonds. C. coupon bonds. D. fixed-payment loans.

D

Non-depository institutions would include all of the following except: A. finance companies. B. pension funds. C. insurance companies. D. credit unions.

D

Nondepository institutions: A. do not serve as intermediaries. B. only serve as brokers. C. only transform assets. D. do not accept deposits.

D

Once you buy a coupon bond, which of the following can change? A. Coupon rate B. Coupon payment C. Face value D. Yield to maturity

D

Over-the-counter (OTC) markets: A. employ specialists to minimize price volatility. B. are centralized exchanges but you must be a dealer to be part of an exchange. C. only deal in the stocks of companies with over $100 million in capital. D. are networks of security dealers linked electronically.

D

Reasons for the rapid structural change in financial markets in recent years include all of the following except: A. globalization. B. technological advances in computing. C. technological advances in communication. D. high real interest rates.

D

Savings and loans primarily provide: A. large commercial loans. B. unsecured credit card loans. C. student loans. D. home mortgages.

D

Suppose a particular depository institution that specializes in residential mortgages is owned by its depositors. The institution is probably a: A. regional or super-regional bank. B. money center bank. C. community bank. D. savings bank.

D

Suppose that a bank initially has a leverage ratio of 8 to 1. If this bank increases its capital by $1 million and its assets by $10 million, then the bank's: A. risk increases and its leverage decreases. B. liabilities decrease and its leverage increases. C. leverage decreases and its liabilities increase. D. leverage and risk increases.

D

Suppose that general business conditions improve, and at the same time, wealth increases. Based on this information, we know that: A. bond prices increase. B. yield to maturity decreases. C. the real interest rate increases. D. the quantity of bonds increases.

D

The U.S. Treasury issues bonds where the return is indexed to the consumer price index. We should expect that these bonds, relative to other U.S. Treasury bonds, will have: A. lower price and lower return due to the decreased risk. B. lower price and a lower fixed return since the demand for them should be higher. C. higher price and higher fixed return since we always seem to have some inflation. D. higher price and lower return due to the decreased risk from inflation in holding these bonds.

D

The better the information provided to financial markets the: A. less the amount of funds transferred between savers and borrowers. B. greater the amount of funds transferred between savers and borrowers though risk increases. C. higher the return required by lenders. D. greater will be the flow of funds in these markets.

D

The holding period return on a bond: A. can never be more than the yield to maturity. B. will equal the yield to maturity if the bond is purchased for face value and sold at a lower price. C. will be less than the yield to maturity if the bond is sold for more than face value. D. will be less than the yield to maturity if the bond is sold for less than face value.

D

The market for bonds is initially described by the supply of bonds - S0, and the demand for bonds - D0, with the equilibrium price and quantity being P0 and Q0. Suppose that the expected return on bonds falls relative to other assets. In the bond market this will result in: A. Bond supply curve to shift to S1 B. Bond demand curve to shift to D1 C. Bond supply curve to shift to S2 D. Bond demand curve to shift to D2

D

The owner of a small business applies for a bank loan and tells the loan officer that the funds will be used to expand inventory for the upcoming holiday season. The small business finds itself in need of additional funds to meet the monthly rent for the next quarter and the owner uses the loan proceeds to pay the rent. This is an example of: A. liquidity risk. B. default risk. C. a lack of diversification for the bank. D. information asymmetry.

D

The price (P) of a consol offering an annual coupon payment (C) is best expressed by: A. F/C B. C(1 + i) C. C/(1 + i) D. C/i

D

The price of a coupon bond can best be described as the: A. present value of the face value. B. future value of the coupon payments. C. future value of the coupon payments and the face value. D. present value of the face value plus the present value of the coupon payments.

D

Trading risk faced by U.S. banks results from: A. the free-rider problem. B. changes in regulations. C. adverse selection. D. moral hazard.

D

When expected inflation increases, for any given nominal interest rate the: A. cost of borrowing increases and the desire to borrow decreases. B. real interest rate increases. C. bond supply curve shifts to the left. D. cost of borrowing decreases and the desire to borrow increases.

D

When interest rates fall a bank's capital will usually: A. not change. B. decrease. C. turn negative. D. increase.

D

Which of the following correctly portrays a bank's balance sheet? A. Total Bank Liabilities = Total Bank Capital + Total Bank Assets B. Total Bank Assets = Total Bank Capital - Total Bank Liabilities C. Total Bank Assets = Total Bank Liabilities - Total Bank Capital D. Total Bank Assets = Total Bank Liabilities + Total Bank Capital

D

Which of the following is not a bank asset? A. Securities B. Mortgage loans C. Reserves D. Non-transaction deposits

D

Which of the following is not a reason why interbank lending dried up during the financial crisis of 2007-2009? A. Banks preferred to hold on to their liquid assets in case their own need for them increased. B. Banks grew increasingly concerned about the ability of their trading partners to repay the loans. C. The increased cost of loans. D. The Fed grew increasingly wary of making liquidity available to banks.

D

Which of the following is not a reason why the yield to maturity can differ from the current yield? A. Because the yield to maturity considers the capital gain/loss. B. Because the current yield focuses only on the coupon payment and the purchase price. C. Because most bonds are not purchased for face value. D. Because the current yield moves in the opposite direction from price.

D

Which of the following is true of interest-rate risk? A. It is the risk that the coupon rate for a bond will change, affecting current bondholders' coupon payments. B. It refers to the probability that a borrower will default on debt obligations. C. It is the risk that the face value of a bond will change before maturity. D. Individuals owning long-term bonds are exposed to greater interest-rate risk.

D

Which of the following statements about the result of a deterioration in business conditions that also causes a decrease in a nation's wealth is false? A. The impact on bond prices will be ambiguous since both the bond demand and supply curves shift left. B. The price of bonds will increase if bond supply decreases more than bond demand. C. Interest rates will increase if bond demand decreases more than bond supply. D. Neither bond demand nor bond supply will shift.

D

Which of the following statements best completes this sentence: "On a bank's balance sheet..."? A. liabilities show the uses of funds and assets show the sources of funds. B. assets show the sources of funds and the net worth shows the uses of funds. C. net worth shows the sources of funds and liabilities show the uses of funds. D. liabilities show the sources of funds and assets show the uses of funds.

D

Which of the following statements is most accurate? A. Yield to maturity is equal to the coupon rate if the bond is held to maturity. B. Yield to maturity is the same as the coupon rate. C. Yield to maturity will exceed the coupon rate if the bond is purchased for face value. D. Yield to maturity is the same as the coupon rate if the bond is purchased for face value and held to maturity.

D

Which of the following statements is most correct for U.S. commercial banks? A. Net interest margin is much larger than return on equity. B. Net interest margin is about equal to return on equity. C. Net interest margin averages about two times the return on equity. D. Net interest margin is closely related to the return on assets.

D

Which of the following would lead to an increase in bond supply? A. A decrease in government spending relative to revenue. B. An increase in corporate taxes. C. A decrease in expected inflation. D. An improvement in general business conditions.

D

Assume we have a stock currently worth 100. we also assume the interest rate is zero, and we can buy options for this stock with a strike price of $100. if the stock can rise or fall by $5 with equal probability over the option period, and the option cannot be exercised until the expiration date, what is the time value of this option?

D none of the answers is correct should be 5x.5=2.5

21) The entire repayment of a(n) ________ loan is computed simply by computing one single future value. A) interest-only B) balloon C) amortized D) pure discount E) bullet

D) pure discount

9) A perpetuity is defined as: A) a limited number of equal payments paid in even time increments. B) payments of equal amounts that are paid irregularly but indefinitely. C) varying amounts that are paid at even intervals forever. D) unending equal payments paid at equal time intervals. E) unending equal payments paid at either equal or unequal time intervals.

D) unending equal payments paid at equal time intervals.

On November 20, 1985, the Bank of New York needed to use the lender of last resort function due to: A. a run on the bank started by a rumor that the president of the bank embezzled tens of millions of dollars from the bank. B. a computer error caused the bank's records to wipe out the balances of all of its customers. C. a rumor that the bank was about to be taken over by FDIC due to insolvency. D. a computer error that made it impossible for the bank to keep track of its Treasury bond trades.

D. A computor error that made it impossible for the bank to keep track of its Treasury bond trades

The government's role of lender of last resort is directed to: A. large manufacturing firms that employ thousands of people. B. depositors; this is role the government plays when they insure depositors' balances in banks that fail. C. developing countries that are trying to build their financial systems. D. banks that experience sudden deposit outflows.

D. Banks that experience sudden deposit outflows

Banks can effectively choose their regulators by deciding whether to: A. be a private or public corporation. B. be a member of the Federal Reserve or not. C. purchase FDIC insurance or to forego the coverage. D. be chartered at the national or state level.

D. Be chartered at the national or state level

Users of commodities are:

D. Buyers of futures

When comparing stock indexes around the world we:

D. Can examine their respective movements if we look at them as percentage changes

Rumors of a bank failing, even if not true, can become a self-fulfilling prophecy because: A. customers will not want to obtain loans from this bank. B. equity investors will not be able to sell the bank's stock. C. regulators will scrutinize the bank heavily looking for something wrong. D. depositors will rush to the bank to withdraw their deposits and the bank under normal situations would not have sufficient liquid assets on hand.

D. Depositors will rush to the bank to withdraw their deposits and the bank under normal situations would not have sufficient liqued assets on hand.

Governments supervise banks mainly to do each of the following, except: A. reduce the potential cost to taxpayers of bank failures. B. be sure the banks are following the regulations set out by banking laws. C. reduce the moral hazard risk. D. eliminate all risk faced by investors.

D. Eliminate all risk faced by investors

The Dow Jones Industrial Average:

D. Gives greater weight to shares with higher prices

Since the 1920's, the ratio of assets to capital has almost tripled for commercial banks. Many economists believe this is the direct result of: A. lower quality management in banks. B. the increase in branch banking. C. allowing banks to offer non-bank services. D. government provided deposit insurance.

D. Government provided deposit insurance

Which of the following is not a positive effect of the Basel Accord? A. It forced regulators to change the way they thought about bank capital. B. It promoted a more uniform international system. C. It provided a framework that less developed countries could use to improve the regulation of their banks. D. It provided a system to differentiate between bonds based on their systemic risk.

D. If provided a system to differentiate between bonds based on their systemic risk

If the Dow Jones Industrial Average is currently at 10,000 and the price of one stock included in the index increases by $10, the DJIA will:

D. Increase by 0.1%

The government's providing of deposit insurance and functioning as the lender of last resort has significantly: A. decreased the incentive for bank managers to take on risk. B. increased the amount of regulation of banks required, but has had no effect on bank's incentive to take on risk. C. increased the incentive for banks to take on risk, but has had no effect on the amount of regulation of banks required. D. increased the amount of regulation of banks required and increased the incentive for banks to take on risk.

D. Increased the amount of regulation of banks required and increased the incentive for banks to take on risk

Which of the following is not an important addition made to the Basel Accords by Basel III in 2010? A. It supplements capital requirements based on risk-weighted assets with restrictions on leverage. B. It introduces three buffers over and above capital requirements itself. C. It adds a liquidity requirement that compels banks to hold a quantity of high-quality liquid assets. D. It ends the too-big-to-fail problem.

D. It ends the too-big-to-fail problem.

The S&P's 500 Index differs from the DJIA because:

D. It takes into account the prices of more stocks and it uses a different weighting scheme

Many states had their own insurance fund to protect depositors. The critical problem with these state funds is: A. they are monopolies in their own state and extract extremely high prices for the insurance they provide. B. they are highly inefficient they cannot achieve the economies of scale a federal fund can achieve. C. they do not have regulators as knowledgeable as the regulators at FDIC. D. no state fund is large enough to withstand a run on all of the banks it insures.

D. No state fund is large enough to withstand a run of all of the banks it insures

The CAMELS ratings are: A. made public monthly to the financial markets so people can judge the relative quality of banks. B. published once a quarter in banking journals issued by the Federal Reserve. C. included in the annual report of publicly owned banks. D. not made public.

D. Not made public

If a public corporation goes bankrupt and does not have enough assets to pay off all creditors:

D. The stockholders cannot lose more than their investment

The purpose of derivatives is to:

D. Transfer the risk from one person to another

which of the following statements is true: a. call options can be sold prior to expiration but put options cannot b. put options can be sold prior to expiration but call options cannot c. no option can be sold prior to expiration d. both american and european options can be sold prior to expiration

D. both American and European options can be sold prior to expiration

A baker of bread has a long-term fixed-price contract to supply bread. which of the following would NOT reduce her risk? a. raking the long position in wheat futures contract b. hedging this risk in the wheat futures market c. finding a wheat farmer who will take the short position in a wheat futures contract d. finding a wheat farmer who will take the long position in a wheat futures contract

D. finding a wheat farmer who will take the long position in a wheat futures contract

The stocks that make up the DJIA:

D. have changed as the structure of the economy has changed

at expiration, the value of an option:

D. is equal to intrinsic value

there's a call option written for 100 shares of GM stock for $85.00 a share, prior to the third friday of October 2018: the option writer:

D. is required to post margin

standardization of derivative contracts:

D. leads to greater liquidity and lower risk

one key difference between swaps and options contracts is:

D. options trade on organized exchanges and swaps do not

one argument why farmers in poor countries remain poor is:

D. poor farmers in many countries lack access to commodity futures markets

An arbitrageur is someone who:

D. simultaneously buys and sells financial instruments to benefit from temporary price differences

Which of the following statements is most correct? a. stockholders have limited liability and have no control over corporate leadership b. stockholders can dislodge the managers of the corporation but not the board of directors c. stockholders have unlimited liability and can dislodge members of the board of directors d. stockholders can dislodge members of the board and have limited liability

D. stockholders can dislodge members of the board and have limited liability

the time value of the option can best be defined as:

D. the fee paid for the potential benefits from buying an option (excluding its intrinsic value)

which of the following would tend to decrease the size of the time value of the option?

D. the time to expiration of the options contract is near

As inflation increases, for any fixed nominal interest rate, the real interest rate: a. Also increases b. Remains the same, that's why it is real c. Decreases d. Decreases by less than the increase in inflation

Decreases

The government provides deposit insurance; this insurance protects:

Depositors for up to $250,000 should a bank fail

Rumors of a bank failing, even if not true, can become a self-fulfilling prophecy because:

Depositors will rush to the bank to withdraw their deposits and the bank under normal situations would not have sufficient liquid assets on hand

Rumors of a bank failing, even if not true, can become a self-fulfilling prophecy because:

Depositors will rush to the bank to withdraw their deposits and the bank under normal situations would not have sufficient liquid assets on hand.

Rumors of a bank failing, even if not true, can become a self-fulfilling prophecy because:

Depositors will rush to the bank to withdraw their deposits and the bank under normal situations would not have the sufficient liquid assets on hand.

Kate buys a share of Google. Google uses the funds raised from selling its stock to expand its operations into Asia. This is an example of: a. Direct finance b. Indirect finance c. Use of a financial institution d. A loan

Direct finance

33. Which one of the following statements is correct? A. Reduced Value index options are equal in size to one percent of the standard index option. B. The holder of a stock index put option is betting that the underlying index will increase in value. C. Most index options are traded on the New York Options Exchange. D. The contract size for a call option on the S&P 500 is 10 times the index. E. Some stock index options close in the morning while others close at the end of the trading day.

E

36. Which one of the following options is out-of-the-money? A. call with a $20 strike and a stock price of $21 B. put with a $35 strike and a stock price of $33 C. call with a $45 strike and stock price of $46 D. put with a $75 strike and a stock price of $70 E. call with a $50 strike and a stock price of $49

E

38. A decrease in which one of the following will increase the intrinsic value of a put option? A. strike price B. exercise price C. option premium D. time value E. underlying stock price

E

44. You own 300 shares of ABC stock. Which one of the following would allow you to receive an option premium in exchange for selling your shares in ABC at the strike price? A. straddle B. long spread C. selling a put D. buying a call E. writing a covered call

E

52. Which one of the following is the upper price bound for the intrinsic value of a European put option on a stock? A. 0 B. strike price C. stock price D. Max (S - K, 0) E. Max (K - S, 0)

E

58. How much will it cost to purchase 5 May $27.50 calls on Texas Instruments? ============ A. $21 B. $1,215 C. $1,245 D. $1,720 E. $2,075

E

59. You bought a call option with a strike price of $35. What is your total payoff on this option contract if the underlying stock is selling for $36.70 on the option expiration date? A. $.00 B. $30.00 C. $33.00 D. $133.00 E. $170.00

E

61. Josh owns 2 call options on Foster Glass stock. The exercise price is $47.50 and the stock price at expiration is $49.01. What is the total payoff on the option contracts? A. -$0.00 B. -$3.02 C. $3.02 D. $30.20 E. $302.00

E

63. You purchased 7 put option contracts on Alto Industries. The strike price was $42.50 and the option premium was $1.30. On the expiration date, the stock was valued at $41.40 a share. What is the payoff on the option contracts? A. -$140 B. $0 C. $110 D. $360 E. $770

E

67. You purchased one SPX put option with a strike of 1,400. You wrote one SPX put option with the same maturity date and a strike of 1,300. At maturity, what is your total payoff if the S&P 500 index is 1,320? A. -$8,000 B. -$2,000 C. $2,000 D. $4,000 E. $8,000

E

73. Kim Lee purchased 6 put option contracts on Eastern Imports stock at a strike price of $47.50. The option premium was $0.65. At expiration, the stock was valued at $44.90 a share. What is her percentage return? A. -100 percent B. 0 percent C. 5.47 percent D. 32.82 percent E. 300 percent

E

74. You own 6 put option contracts on JL Industrial stock. You paid an option premium of $0.75 for a strike price of $42.50. On the option expiration date, the stock was selling for $41.00 a share. What is your percentage return? A. -100 percent B. -18.75 percent C. 51.26 percent D. 78.75 percent E. 100 percent

E

75. Jasmine purchased one call option with a strike price of $35 when the call premium was $1.10. What is the break-even stock price? A. $.00 B. $33.90 C. $34.45 D. $35.00 E. $36.10

E

82. You purchased a put with a strike price of $37.5 and an option premium of $0.45. You simultaneously bought the stock at a price of $36 a share. What is your profit per share on these transactions if the stock price at expiration is $33.50? A. -$1.15 B. -$0.15 C. $0.15 D. $0.75 E. $1.05

E

92. A stock is currently selling for $26.50. A 3-month put option with a strike price of $30 has an option premium of $4.05. The risk-free rate is 2.5 percent and the market rate is 9.75 percent. What is the option premium on a 2-month call with a $30 strike price? Assume the options are European style. A. $0.00 B. $0.33 C. $0.41 D. $0.67 E. $0.73

E

96. A call option has a premium of $2.80, a strike price of $55, and 3 months to expiration. The current stock price is $52.20. The stock will pay a $1.25 dividend in one month. The risk-free rate is 2.5 percent. What is the premium on a 3-month put with a strike price of $55? Assume the options are European style. A. $2.08 B. $2.15 C. $3.32 D. $4.12 E. $6.51

E

97. A call option has a premium of $0.60, a strike price of $40, and 3 months to expiration. The current stock price is $39.60. The stock will pay a $0.80 dividend two months from now. The risk-free rate is 3 percent. What is the premium on a 3-month put with a strike price of $40? Assume the options are European style. A. $0.25 B. $0.51 C. $0.78 D. $1.23 E. $1.50

E

7) You are comparing two annuities that offer regular payments of $2,500 for five years and pay .75 percent interest per month. You will purchase one of these today with a single lump sum payment. Annuity A will pay you monthly, starting today, while annuity B will pay monthly, starting one month from today. Which one of the following statements is correct concerning these two annuities? A) These annuities have equal present values but unequal future values. B) These two annuities have both equal present and equal future values. C) Annuity B is an annuity due. D) Annuity A has a smaller future value than annuity B. E) Annuity B has a smaller present value than annuity A.

E) Annuity B has a smaller present value than annuity A.

3) Project A has cash flows of $4,000, $3,000, $0, and $3,000 for Years 1 to 4, respectively. Project B has cash flows of $2,000, $3,000, $2,000, and $3,000 for Years 1 to 4, respectively. Which one of the following statements is correct assuming the discount rate is positive? (No calculations needed) A) The cash flows for Project B are an annuity, but those of Project A are not. B) Both sets of cash flows have equal present values as of Time 0. C) The present value at Time 0 of the final cash flow for Project A will be discounted using an exponent of three. D) Both projects have equal values at any point in time since they both pay the same total amount. E) Project B is worth less today than Project A.

E) Project B is worth less today than Project A.

2) Project X has cash flows of $8,500, $8,000, $7,500, and $7,000 for Years 1 to 4, respectively. Project Y has cash flows of $7,000, $7,500, $8,000, and $8,500 for Years 1 to 4, respectively. Which one of the following statements is true concerning these two projects given a positive discount rate? (No calculations needed) A) Both projects have the same future value at the end of Year 4. B) Both projects have the same value at Time 0. C) Both projects are ordinary annuities. D) Project Y has a higher present value than Project X. E) Project X has both a higher present and a higher future value than Project Y.

E) Project X has both a higher present and a higher future value than Project Y.

6) Which one of these statements related to growing annuities and perpetuities is correct? A) You can compute the present value of a growing annuity but not a growing perpetuity. B) In computing the present value of a growing annuity, you discount the cash flows using the growth rate as the discount rate. C) The future value of an annuity will decrease if the growth rate is increased. D) An increase in the rate of growth will decrease the present value of an annuity. E) The present value of a growing perpetuity will decrease if the discount rate is increased.

E) The present value of a growing perpetuity will decrease if the discount rate is increased.

Trading in electronic exchanges has grown tremendously in recent years, what are some of the disadvantages of trading in decentralized electronic exchanges?

Electronic operations have proven prone to errors that threaten the existence of brokers. In addition, amid the complex system of multiple, imperfectly linked exchanges, new trading patterns have arisen that render the entire system fragile, raising serious worries among investors about the liquidity and value of their stocks. Efforts to speed up electronic trading drain resources from more efficient uses. To see this point, imagine that an HFT firm relocates its computing facilities closer to an exchange so that it can cut the transmission time for orders by a few microseconds (millionths of a second). The goal of the move is to profit by trading an instant faster than competitors when new information becomes available, such as a stock issuer's quarterly profit statement or the nation's monthly employment report. Yet microsecond gains in trading speed likely diminish the willingness of market makers to provide liquidity because they don't wish to be "picked off" by well-equipped HFTs. Over time, market makers also may be forced to invest heavily to keep up with the faster turnover of stocks, with little gain to the broader economy. Finally, the advent of multiple electronic exchanges and ECNs has not resulted in a single, integrated, and transparent U.S. stock market.

Stock market bubbles impact consumers by:

Encouraging greater consumption of luxury goods and less saving.

Which of the following is an example of the economies of scope argument for increased profits for large financial holding companies? a. Financial holding companies offer a wide array of services under one name b. Financial holding companies need only one CEO, one Board of Directors, and one accounting system regardless of size c. Financial holding companies are well diversified so risk is reduced d. The profitability of financial holding companies does not rely on one particular line of business

Financial holding companies offer a wide array of services under one name

Compare and contrast financial institutions that act as brokers to those that transform assets. In what sense are both types of institutions financial intermediaries? Provide one example of each type and describe how each functions as a financial intermediary.

Financial institutions that act as brokers provide a way for lenders/savers to buy securities from borrowers/spenders. Such institutions make it easier for borrowers and savers direct access financial markets. Financial institutions that transform assets collect deposits (and payments for insurance policies) to raise funds that are then loaned to borrowers/spenders. These institutions allow borrowers and savers to interact indirectly. Depository institutions accept deposits from savers and issue loans to borrowers. Insurance companies accept premiums from policy holders (savers) and invest these funds in securities. When a policy holder makes a claim (borrower), he/she receives compensation in the even of a bad event (accident, illness, theft, etc.). Pension funds invest contributions from savers and provide payments to retirees (borrowers). Securities firms provide brokerage services, allowing investors (savers) the ability to buy securities (issued by borrowers) in financial markets. Investment banks serve as underwriters, easing access to markets by bringing securities issued by borrowers into secondary markets for purchase by savers. Mutual funds mainly transform assets, allowing savers to purchase a diverse group of securities (issued by borrowers) with a small initial investment. Finance companies raise funds from buying securities in financial markets and loan out funds to borrowers. Government-sponsored programs, such as Social Security, provide the same services that pension funds and insurance companies provide privately.

Explain the various ways that financial intermediaries increase the efficiency of an economy.

Financial intermediaries increase an economy's efficiency in a number of ways. First, they provide a means for savers to channel funds to borrowers (spenders). This puts otherwise idle resources to use, increasing an economy's output. While savers theoretically could lend directly to borrowers, the transaction costs as well as the risk would be significantly increased, to the point where these funds may not actually flow. Also, financial intermediaries lower the transaction costs of lending. This includes information gathering as well as monitoring costs. These lower transaction costs allow resources to be used to increase the output of goods and services in the economy.

How do financial markets pool and communicate the information regarding issuers of financial instruments in a convenient way?

Financial markets pool and communicate information about the issuers of financial instruments and summarize this information in the form of a price. For example, any information that says an issuer of a financial instrument is less likely to honor its payment would have the price of the instrument decrease or the required return increases. Any information that places the issuer in a more favorable light would have the opposite effect.

As the end of the year 1999 approached, many people worried that banks and more specifically the banks' computers would not be able to read the year 2000 correctly. This was commonly known as the Y2K problem. Many people were concerned that their bank would lose the record of their deposits etc., and made plans to take most of their funds out of the bank. Address the potential Y2K problem from the standpoint of bank risk. What two types of risk potentially could have been involved?

First, if the problem did arise and bank records would have been wiped out, this would have been an example of operational risk. A technological breakdown would have led to significant problems for many people and potentially a financial market panic. The risk would not have stopped there; as savers demanded their funds from banks, the banks would have faced liquidity risk. Most banks would not have been able to meet the demand for liquidity. This was a concern of the Federal Reserve who had significant amounts of currency on hand in the event that people did start to withdraw their savings from banks.

A $1000 face value bond, with an annual coupon of $40, one year to maturity and a purchase price of $980:

Has a current yield that equals 4.08% and a yield to maturity that equals 6.12%.

Cross-hedge

Hedging a particular spot position with futures contracts on a related, but not identical, commodity or financial instrument

The specific goals of central banks include each of the following except:

High levels of exports

Which of the following could the lemons problem, applied to financial markets, explain?

High quality potential borrowers relying more on internally generated funds to finance investment

The specific goals of central banks include all of the following except:

High stock prices

High oil prices tend to harm the auto industry and benefit oil companies; therefore, high oil prices are an example of: a. Systematic risk b. Idiosyncratic risk c. Neither systematic nor idiosyncratic risk d. Both systematic and idiosyncratic risk

Idiosyncratic risk

If a bank has a net worth that is negative, what do you know about the relationship between the amounts the bank has in assets and liabilities?

If a bank had a negative net worth, it means the bank's capital is less than zero. This can only be the case if the bank's liabilities exceed its assets.

Suppose that a bond is purchased at a discount (meaning that it is sold for less than face value). Could the yield to maturity ever be less than the coupon rate? Could the holding period return be less than the coupon rate? Explain.

If a bond is purchased for less than face value, the yield to maturity will always exceed the coupon rate. For the yield to maturity to be less than the coupon rate the price of the bond would have to exceed the face value. On the other hand, the holding period return could be less than the coupon rate. Even if a bond is purchased for less than face value, there is no guarantee it will sell before the maturity date for an amount that is at or above the face value; in fact it could sell for an amount well below the actual purchase price.

Many people are worried that, with the growing number of people that will retire in the U.S. over the next 40 years, the federal government will need to borrow large amounts of money to finance the Social Security System. If we assume that Social Security taxes and the current eligibility age remain constant, explain the likely impact this will have on bond markets.

If the Social Security Administration (SSA) finds that it will need to borrow to finance its obligations this will cause the bond supply to increase, a shift to the right of the bond supply curve. All other factors constant, this will cause bond prices to fall and yields to increase. The yields on all bonds will rise, however, since the U.S. government, and the SSA is a government agency, is usually viewed as the risk-free rate from a default standpoint. Since the yields on these bonds will likely increase, this will cause the yields on all bonds to rise since all other bonds have their respective risk premiums which are then added to the risk-free return associated with U.S. government bonds.

Bank managers seem to have to walk a tightrope between managing risk and earning a profit. Explain.

If we consider the various forms of bank risk and the possible ways to manage it, we can see the challenges posed for a bank seeking to make the highest possible profit (to satisfy its owners). Managing liquidity risk means having to structure securities holdings so that sales can be carried out if needed. This means holding shorter-term securities such as Treasury securities, but they tend to pay less of a return. Managing credit risk means having to diversify, which may mean higher costs to the bank as it gives up a competitive advantage in a narrow line of business? Managing interest-rate risk means restructuring assets to better match liabilities and, as with liquidity risk management, this also reduces potential profitability. Add to this the limitations imposed by regulators (such as not being able to own stock and monitoring of bank leverage) and one might wonder how banks ever manage to earn a profit.

Marking to market

In futures trading accounts, the process whereby gains and losses on outstanding futures positions are recognized on a daily basis

For fiscal policymakers, one of the results of an independent central bank is:

Increased government spending has to be financed with either higher taxes or increased government borrowing. (Ch. 15)

The government's providing of deposit insurance and functioning as the lenders of last resort has significantly:

Increased the amount of regulation banks required and increased the incentive for banks to take on risk.

There is a strong consensus among economists that monetary policy is more effective when it is formed:

Independently of political pressure

General agreement among economists finds that they believe monetary policy is more effective when it is formed:

Independently of political pressure (Ch. 15)

The notion that stock prices reflect all current available information:

Indicates that mutual fund managers will not, on average, outperform market averages.

Which of the following is true of interest-rate risk?

Individuals owning long-term bonds are exposed to greater interest-rate risk.

A lender usually knows less about the creditworthiness of a borrower than the borrower does. This is an example of:

Information asymmetry (Ch. 11)

Information asymmetry that exists in lending creates what type of risk for banks? Discuss the ways for a bank to handle or minimize this risk.

Information asymmetry leads to the problems of moral hazard and adverse selection. In banking this creates credit risk. The credit risk can be minimized by acquiring additional information. This additional information may be in the form of a loan application, a credit report, and credit scoring tools, charging different interest rates based on risk, and by monitoring the borrower after the loan is made.

When considering different investments, a risk-averse investor is most likely to focus on purchasing: a. Investments with the greatest spread in the expected rate of return b. Investments that offer the lowest standard deviation in the investments' expected rates of return for any given expected rate of return c. Only risk-free investments d. Investments with the lowest risk premium, regardless of the expected rate of return

Investments that offer the lowest standard deviation in the investments' expected rates of return for any given expected rate of return

Everything else equal, if the growth rate of a country exceeds its sustainable rate, the central bank:

Is likely to raise the interest rate to slow the rate of growth

The chairman of the Board of Governors:

Is selected from the Board of Governors, appointed by the U.S. President

The Standard & Poor's 500 Index differs from the Dow Jones Industrial Index because:

It takes into account the prices of more stocks and it uses a different weighting scheme.

If the government did not offer the too-big-to fall safety net:

Large banks would be more disciplined by the potential loss of large corporate accounts

Suppose that a bank initially has a leverage ratio of 7 to 1. If this bank increases its capital by $1 million and its assets by $8 million, then the bank's:

Leverage and risk increases (Ch. 12)

When the yield curve is upward sloping, people are expecting: a. An economic slowdown b. The U.S. Treasury may default on its obligations c. The Federal Reserve is going to ease monetary policy d. Long-term yields to be higher than short-term yields

Long-term yields to be higher than short-term yields

Sue buys a futures contract for U.S. Treasury bonds and on the settlement date the interest rate on U.S. Treasury bonds is higher than Sue expected. Sue will have:

Lost money on her long position.

Secondary reserves for banks are:

Mainly the bank's liquid securities

Secondary reserves for banks are:

Mainly the bank's liquid securities.

An economic rationale for government protection of small investors is that:

Many small investors cannot adequately judge the soundness of their bank (Ch. 14)

An economic rationale for government protection of small investors is that:

Many small investors cannot adequately judge the soundness of their bank.

Cash market (AKA: Spot market)

Market in which commodities or financial instruments are traded for immediate delivery

Mary Jones is the president of a local bank. She knows that half of the loan applicants in town she would classify as high risk and the other half as low risk. She observes that the other banks in town charge two different interest rates, a lower rate for low-risk borrowers, and the higher rate for high-risk borrowers. She decides that to have an advantage over the other banks she will offer an average rate to everyone. The likely result will be:

Mary's bank will experience adverse selection and have a disproportionate number of high risk borrowers.

The fact that many companies employ supervisors to oversee the actions of workers is a way to treat:

Moral hazard.

It is difficult for depositors to know the true health of banks because:

Most of the information on bank loans is private and based on sophisticated models.

A risk-averse investor versus a risk-neutral investor: a. Will never take a risk, while the risk neutral investor will b. Needs greater compensation for the same risk versus the risk neutral investor c. Will take the same risks as the risk neutral investor if the expected returns are equal d. Needs less compensation for the same risk versus the risk neutral investor

Needs greater compensation for the same risk versus the risk neutral investor

Which of the following statements about the result of a deterioration in business conditions that also causes a decrease in a nation's wealth is false?

Neither bond demand nor bond supply will shift.

Standard & Poor's sells information to investors; this is their primary business. Is this an example of a financial intermediary? Explain.

No. A financial intermediary is involved indirectly in a financial transaction. It matches up the ultimate lenders (savers) with the ultimate spenders (borrowers). The funds flow through the intermediary which is acting as a "middleman." That is not the case with Standard & Poor's.

Suppose that an internet-based program, Novus, wants to raise $10 million to expand its business operations. Describe how Novus can raise these funds directly through each of the follow options: issuing stock, issuing bonds, or obtaining a bank loan. Compare and contrast these three options.

Novus can issue new stock worth $10 million. Alternatively, it can issue $10 million in bonds. In either of these two cases, Novus will seek out an investment bank to serve as an underwriter (to bring the shares from the primary to the secondary market). If Novus issues stock, it is not obligated to pay dividends to its new stock holders, but if it doesn't it risks reducing the value of its stock. If Novus issues bonds, it must pay interest in regular payments. If Novus goes to a bank for a loan, it will make regular payments that include interest (and possibly parts of the principal amount owed), similar to a bond issue.

Explain why holding period return, as an economic measure, does not have the same significance as current yield or yield to maturity.

One of the things economists do is try to explain behavior, or decisions people make. Current yield and yield to maturity are a priori measures, meaning we can calculate these prior to actually making the purchase of the bond. The holding period return cannot be calculated a priori, it is only calculated after the bond is purchased; to a certain degree it represents a "sunk" cost.

. Bank's hold marketable securities as part of their assets. For U. S. banks these marketable securities include: a. Stocks and bonds b. Only the stocks of U. S. corporations c. Only the bonds of the U. S. treasury d. Only bonds

Only Bonds

The Federal Reserve's policy regarding announcing its policy decisions has:

Only recently gone to immediate announcement; until 1994 these policy decisions were secret

The use of money makes us more efficient because: a. We spend more time trading and more time producing b. People can specialize in what they do well c. With money we borrow less d. Money increases in value over time

People can specialize in what they do well

What evidence is there that the transaction costs involved with the buying and selling of stocks is low?

Probably the best evidence is the volume of trading that occurs on an average day. As an example, on an average day billions of shares of stock may trade in the U.S. alone, and while most of these trades are undertaken using brokers, the fee the broker requires is usually a very small percentage of the overall value of the instruments traded. The volume of trades and the low fees for these trades would not result if transaction costs were high.

Bank failures tend to occur most often during periods of:

Recessions when many borrowers have a difficult time repaying loans and lending activity slows down.

One of the cash items included on the asset side of banks' balance sheets is reserves. What makes up reserves and what is their purpose?

Reserves are made up specifically of two items: one is cash in the bank, also referred to as vault cash. The other is deposits the bank has at the Federal Reserve. The main purpose of reserves is to make sure banks have enough liquidity to meet the demands of customers for withdrawal requests.

During the financial crisis of 2007-2009 in the U.S. it was revealed that the function of a lender of last resort had not kept pace with the evolving financial system because:

Shadow banks lacked access to the financial resources available through the lender of last resort

.Identify which of the following is not one of the five core principles of money and banking. a. Risk requires compensation b. Time has value c. Information is the basis for decisions d. Stability creates risk

Stability creates risk

An argument that comes up from time to time is that credit unions have an advantage over other financial depository institutions in the sense that they are non-profit institutions and, therefore, are exempt from taxes on income that other private depository institutions pay. As a result, credit unions may be able to charge lower rates of interest to borrowers and pay a higher rate to depositors than these other institutions. What do you think of this argument?

The argument may have some validity as stated, however, whether credit unions are able to charge a lower rate of interest to borrowers or offer a higher rate to depositors is questionable. Most credit unions are smaller than most banks, partially due to their employer ties and/or other restrictions on membership, and as a result are not as able to exploit economies of scale and scope as larger banks. Also, credit unions may have better information to assess members' creditworthiness; on the other hand, they may make more questionable loans thinking that the borrower represents a better risk because of this membership. This is a form of adverse selection.

As we saw in the chapter, some financial instruments are used primarily to transfer risk. Explain how a bread maker can use a financial instrument to transfer the following risk: the bread maker has the opportunity to provide bread to a local army base. The base figures they will need 10,000 loaves of bread each week, or roughly 500,000 for a year. The problem is the baker must quote a price for the entire year. The baker would really like to have this contract but he realizes that fluctuating input prices (specifically wheat) could result in significant losses.

The baker could quote a price for bread based on today's price and then purchase wheat a futures contract for wheat at today's price, for delivery one year from now. If actual wheat prices do increase the baker will lose money on the actual baking operations but these losses will be offset by the profits he will earn on the wheat futures contract. If wheat prices end up decreasing, he will suffer losses on the futures contract but will offset these by having higher profits from baking. In this case the futures contract accomplishes exactly what it was supposed to do. It transferred the risk of volatile wheat prices from the baker, who otherwise wouldn't accept the opportunity to provide bread at a guaranteed price for a year, to someone who was more willing to accept this risk.

If a bank has $200 million in deposits, the required reserve rate is 10 percent and the bank has $23 million in reserves: a. The bank is short of required reserves b. The bank has excess reserves of $21 million c. The bank has excess reserves of $13 million D. The bank has excess reserves of $3 million

The bank has excess reserves of $3 million

If a bank has deposits of $250 million, reserves that total $30 million and has a required reserve ratio of 10 percent:

The bank has excess reserves of $5 million

A bank has the following assets: Reserves of $15 million; Loans of $150 million; and Securities of $50 million. Their liabilities include Deposits of $150 million; Borrowed funds of $35 million and Bank Capital of $30 million. If the required reserve rate is 10 percent, answer the following: What is the amount of excess reserves the bank is currently holding? What are the options available to the bank if customers decide to withdraw $10 million in deposits?

The bank is not holding any excess reserves. With deposits of $150 million and a required reserve rate of 10%, the reserves of $15 million are what the bank needs. If customers decide to withdraw $10 million from the bank the bank can meet this need either through asset adjustment or liability adjustment. First, from the asset side, the bank could sell securities for cash or decide not to renew some loans. Since the bank is interested in maintaining good customer relations, they would probably opt for the selling of securities. From the liability adjustment side, the bank could borrow the additional funds required or try to increase deposits, perhaps by offering an attractive interest rate on long-term CDs.

A bank develops specialized skills in analyzing companies from one specific industry. This contributes significantly to the bank achieving economies of scale because a large portion of its total loan portfolio is made up of companies in this industry. What are the long-run profit prospects for this bank? Explain.

The bank may have lowered its transaction costs due to its specialization of mainly lending to firms in one industry but it has considerably increased its idiosyncratic or credit risk. It would likely suit the bank better in the long run to diversify its portfolio so it is only exposed to systematic risk. Ideally if the bank could generate the loans and then securitize them and sell them off (like home mortgages) it may have the best possible outcome.

A bank's reserves include:

The bank's deposits at the Federal Reserve (Ch. 12)

A bank's Return on Assets (ROA) is calculated by dividing:

The bank's net profits after taxes by its assets.

Compute the change in the price of a five-year (until maturity) $1,000 face value zero-coupon bond that currently yields 7% when expected inflation increases from 3% to 4%.

The bond currently will sell for $712.99. Once the expected inflation increases by 1%, the bondholders would want to keep the same real return, which would drive the bond yield up to 8%. This increase in bond yield will drive the price down to $680.58, or a decrease of more than 4.8% of the bond's price.

How can a bond mutual fund report a return of over 13% when the coupon rate of the bonds they are holding are just 7% and interest rates are falling?

The bond mutual fund is advertising its holding period return. As illustrated in the text, if a 20-year, 7-percent coupon bond with a face value of $100 is sold before maturity when interest rates have fallen to 6.5%, the price of that bond will rise to $106.50. The $6.50 of capital gain plus the $7.00 coupon payment represent a one-year holding period return of 13.5%. As the text notes, this is why past performance cannot guarantee future returns; if rates rise instead of fall the holding period return will not be as favorable.

Which bond will have a higher yield to maturity, a $1,000 face value bond, with a 5.0% coupon rate that sells for $900; or a $1000 face value bond, with a $50 annual coupon that sells for $1,050? Explain your choice.

The bond that is selling for $900 will. Both bonds have the same coupon rate, 5%, and they have the same maturity, so the bondholder's returns from the coupons are equal. What differentiates the two is that the bondholder who purchases the bond for $900 will also receive a capital gain which increases his/her yield to maturity.

If a bond's rating improves it should cause: a. The bond's price and yield to increase, all other factors constant b. The bond's price and yield to decrease, all other factors constant c. The bond's price to increase and its yield to decrease, all other factors constant d. The bond's price to decrease and its yield to increase, all other factors constant

The bond's price to increase and its yield to decrease, all other factors constant

In the late 1990s, the U.S. government ran a surplus for the first time in decades. It instituted a buyback program, whereby the Treasury bought outstanding government bonds. How would this program affect the bond market price, yield, and quantity of bonds? How might it affect the liquidity of government bonds?

The buyback of government bonds would reduce bond supply, increasing the bond price, reducing the yield, and decreasing the quantity of bonds.

3. We have a futures contract for the purchase of 10,000 bushels of wheat at $3.00 per bushel. If the price of wheat were to increase to $3.50, explain what happens to the parties involved in the contract in terms of marking to market. Be sure to identify who is long and short and specifically how much is transferred.

The buyer of the contract, the long position, will pay $30,000 for 10,000 bushels of wheat. The seller of the contract, the short position, delivers 10,000 bushels of wheat and receives $30,000. If before expiration the market price of wheat increases to $3.50 the seller, (short) will have to give the buyer, (long) $5000 so that the buyer will still only have to pay $30,000 for the wheat. So the buyer's margin account will be marked to market (credited) with $5,000, which comes from the seller's margin account which is marked to market (debited) for the $5,000.

Net interest income for a bank is:

The difference between interest income and interest expense (Ch. 12)

Contagion is:

The failure of one bank spreading to other banks through depositors withdrawing of funds

9. If the current closing price in the stock of XYZ, Inc. is $87.50 and the July expiration put options with a strike price of $80 are selling for $1.05, what is the intrinsic value of the option? What is the option premium?

The intrinsic value of a put option is the strike price less the market price, which in this case is a negative $7.50. An option cannot have an intrinsic value less than zero, however, since the option does not have to be exercised. So in this case the intrinsic value is zero. The option premium is the option price less the intrinsic value, so the option premium is $1.05.

7. Suppose you purchase a call option to purchase General Motors common stock at $80 per share in March. The current price of GM stock is $83 and the time value of the option is $5. What is the intrinsic value of the option? As the expiration date approaches, what will happen to the size of the time value of the option?

The intrinsic value of an option is equal to the difference between the current market price and the strike price, which in this case is $83 - $80, or $3. The time value of the option at expiration is zero since the option value equals the intrinsic value.

Why might a life insurance company insist on an individual having a physical exam before agreeing to provide life insurance to the individual?

The life insurance policy is a contract that transfers risk from the buyer to the seller, in this case from the individual to the company. The price of the contract is based upon certain assumptions regarding the general health of the individual and specific information such as gender, age, etc. The company wants to make sure there is not any information hidden (information asymmetry) or other problem that would significantly alter its decision to provide the coverage or the price of the coverage.

Consider a typical individual who owns the following financial instruments: A life insurance policy for $250,000; a certificate of deposit for $10,000; homeowner's and auto insurance policies; $50,000 in a mutual fund, and $150,000 in her pension fund at work. Which of these are instruments used primarily as stores of value and which are being used to transfer risk?

The life insurance policy, the homeowner and auto insurance policies are instruments being used to primarily transfer risk. The cost of an untimely death or loss resulting from an auto accident or damage to her house is a risk the individual prefers to transfer to someone else. The certificate of deposit, the balances in her mutual fund and pension are instruments that are serving primarily as stores of value. In these instruments wealth is being accumulated and stored for use at a later time.

What matters most during a bank run is:

The liquidity of the bank

Is the obtaining of a car loan a primary or secondary market transaction?

The obtaining of a car loan is a primary market transaction since the loan represents a newly-issued instrument by the bank.

Suppose a family member approaches you to borrow $2,000 for the down payment on an automobile. You have the cash available in a savings account that currently earns 5% annual interest. You and the family member consider the following repayment options: (i) Borrower repays $259 each year over the next ten years. (ii) Borrower repays $300 each year over the next five years, plus a lump-sum payment of $895 in the fifth year. (iii) Borrower repays you $2,100 at the end of one year. For each of the options above, show that the present values of each option are approximately equal. Then, relate each of the options above to the four types of bonds, indicating which option is equivalent to which type of bond. Explain why.

The present values are calculated as follows: (i) Fixed-payment loan. The borrower repays a loan in fixed annual payments for a pre-determined period of time. Present value of fixed payment loan = = Fixed payment/(1 + i) + Fixed payment/(1 + i)2 +...+ Fixed payment/(1 + i)n = $259/(1 + 0.05) + $259/(1 + 0.05)2 +...+ $259/(1 + 0.05)10 = $2,000 (ii) Coupon bond. The borrower repays the loan in fixed annual payments ("coupons") and pays a one-time lump sum payment in the last year of the loan ("face value"). Present value of coupon bond = PCB PCB = Coupon payment/(1 + i) + Coupon payment/(1 + i)2 +...+ Coupon payment/(1 + i)n + Face value/(1 + i)n = $300/(1 + 0.05) + $300/(1 + 0.05)2 +...+ $300/(1 + 0.05)5 + $895/(1 + 0.05)5 = $2,000 (iii) Zero-coupon bond. The borrower repays the principal plus interest in one annual payment, with no intermediate payments. Price of zero coupon bond = Payment/(1 + i) = $2100/(1 + 0.05) = $2,000

The strike price of an option is:

The price at which the option holder has the right to buy or sell.

In Chapter 11 we discussed the principal-agent problem as a form of moral hazard. Discuss the unique problems a bank manager faces in terms of trying to please the owners of the bank and at the same time trying to appease regulators.

The principal-agent problem arises from the fact that often the manager of a company is not acting in the best interest of the owners. In the case of a bank, the owners would certainly want a high profit with a reasonable amount of risk. On the other hand the regulators of a bank prefer the bank minimize risk. For example, we saw that increasing the leverage (higher debt-to-equity ratio) can increase the return on equity, something the owners of the bank would prefer. The higher leverage on the other hand also increases the risk, something the regulators will frown upon. So the bank manager is in the unenviable position of having to balance the desires of the owners, the regulators, and to a large degree the depositors. This is not an easy task.

Requiring that borrowers put up collateral to obtain a loan is a tool designed to treat:

The problem of moral hazard.

A high school basketball player decides to bypass college and go right into the NBA, (the National Basketball Association). Describe the risk the individual is taking and a contract that might transfer the risk

The risks the individual is taking are numerous; one, he may not be as talented as he thinks and does not perform as well as he thinks he will and his value decreases. Perhaps more important, he could suffer a career-ending injury. In either case by bypassing college he has left himself with fewer options than he might otherwise have. These risks can be transferred through a few different types of contracts. First, he can negotiate a guaranteed contract that will pay him even if he is injured and can't play. The team would likely go along with this if the annual compensation is reduced. The individual could ask for the majority of his first contract to be in a guaranteed upfront payment which can then be used to purchase an annuity to provide income for the rest of his life. The individual could also purchase a disability insurance policy to provide a specified income in the event that he is injured and cannot do his job.

There is a futures contract for the purchase of 100 bushels of wheat at $2.50 per bushel. If the market price of wheat increases to $3.00 per bushel:

The seller (short position) needs to transfer $50 to the buyer (long position).

The practice of "redlining" in bank lending is clearly an example of discrimination in lending. Agree or disagree? Why?

The term "redlining" alludes to a practice in which bank officers would not lend in certain areas marked off by redlines drawn on a map. From a lender's perspective, such a practice is a way to manage credit risk. If default rates are higher in one area than another, banks will have an incentive to funnel loan funds to the lower-risk area. Unfortunately, given the racial composition of many high-risk neighborhoods, the policy looked discriminatory even though it may have in fact been colorblind. It should also be noted that studies conducted after Congress took steps to remedy the perceived problem do not add further evidence as to whether or not discrimination had been at the root of redlining.

If a one year bond has a face value of $100 and is purchased for $94, and is held to maturity:

The yield to maturity will exceed the holding period return.

Central banks often find:

There are tradeoffs that make pursuing all their goals simultaneously impossible.

The text identified the various sources of risk for bonds. Are U.S. Treasury TIPS bonds free from risk? Explain.

These bonds are free from two of the main sources of risk that make holding bonds risky. U.S. Treasury bonds are free from default risk, and the TIPS bonds also remove the inflation risk. However, the risk still present with these bonds is the interest-rate risk. If the bondholder does not plan on holding these bonds until maturity, changes in the interest rate (specifically increases) can result in capital losses or returns less than expected.

Checkable deposits have decreased since the 1970's mainly because:

These deposit accounts offer little or no interest so depositors find them to be expensive.

The existence of a lender of last resort creates moral hazard for bank managers because:

They have an incentive to take too much risk in their operations

Which of the following is NOT a reason that firms in the shadow banking system were more vulnerable than commercial banks during the financial crisis of 2007-2009?

They were more heavily regulated than commercial banks, making them less able to adjust to changing market conditions.

Often a bank will require a loan officer to make personal visits on customers with loans outstanding. This is encouraged because:

This is an effective monitoring technique and should reduce moral hazard.

The reasons for government to get involved in the financial system include each of the following except:

To protect the bank's monopoly position

The primary use of derivative contracts is: a. For IRA and other pension plans since they only have value well into the future b. To shift risk among investors c. For investors seeking a greater return by taking greater risk d. To add to the profits an investor obtains through information asymmetry

To shift risk among investors

The most common form of zero-coupon bonds found in the United States is:

U.S. Treasury bills.

. A bank's reserves include: a. Vault cash b. U. S. Treasury Securities c. The bank's loan portfolio d. U. S. Treasury bills and vault cash

Vault cash

Consider a $1,000.00 face value bond with a $55 annual coupon and 10 years until maturity. Calculate the current yield; the coupon rate and the yield to maturity under each of the following: a) The bond is purchased for $940.00 b) The bond is purchased for $1,130.00 c) The bond is purchased for $1,000.00

We can use a financial calculator to solve for each of these. The easiest answer is to realize the coupon rate will not change; it is $55/$1000 or 5.50% (.055). The current yield, which is the coupon divided by the purchase price will vary for each: for a), the current yield is 5.85%; for b) it is 4.87% and for c) it is 5.50%. The yield to maturity will also vary: for a) it is 6.33% for b) it is 3.90% and for c) it is 5.50%.

Calculate the price of a zero coupon bond that has an interest rate of 6.65% (.0665), a face value of $100.00 and six-months to maturity.

We can use the formula from the text where Price (P) = Face value/(1 + i)n. In this case, P = $100/(1 + .0665)0.5, which equals $96.83.

Credit cards usually charge higher rates of interest than most other forms of lending. In terms of information, collateral and monitoring, how might these higher rates be explained?

When providing credit cards to customers, banks have the ability to obtain information at the time of application and based on this information they decide to issue or not issue the card. Once issued however, the ability of the bank to obtain further information and monitor the behavior of the individual is limited and before the card issuer can respond the cardholder can incur significant debt. Also, these are basically unsecured loans, meaning there is no collateral for the lender to seize if payment is not made. All of these facts and more make credit card loans risky and demanding of the higher rate.

Briefly explain one function of financial instruments that can make them very different from money.

While financial instruments can function as a means of payment and a store of value, similar to money, one function that can make them very different from money is their ability to transfer risk between buyer and seller. A good example of this is the use of a futures contract that guarantees to the seller of the contract a price well into the future. Another common example is an insurance policy that transfers risk from the insured (a homeowner) to the insurer (the insurance company).

Notice the following model of a bond market. In each situation given, explain what happens to the bond price and yield and why. a) Expected inflation increases b) The return on bonds rises relative to other assets c) The federal government deficit increases

a) If expected inflation increases the demand for bonds will decrease and the supply will increase. Both of these will reinforce each other, causing the bond prices to fall and interest rates to increase. b) If the return on bonds rises relative to other assets, the bond demand curve will shift to the right, causing bond prices to increase and interest rates to decrease. c) If the federal budget deficit increases, the bond supply curve will shift to the right, causing the bond prices to fall and interest rates to increase.

Everything else equal, if the ratio of bank assets to bank capital decreases, the bank's return on equity should: a. Decrease b. Remain constant c. Increase d. Cannot be determined from the information provided

a. Decrease

Which of the following is an accurate statement about universal banks? a. In Germany universal banks do everything under one roof, including direct investment in the shares of nonfinancial firms b. In Germany the provision of insurance, banking, and securities must be done by separate corporations c. As in Germany, universal banks in the United States do everything under one roof, including direct investment in the shares of nonfinancial firms d. Universal banks in the United States account for the largest share of financial intermediary assets

a. In Germany universal banks do everything under one roof, including direct investment in the shares of nonfinancial firms

The bond demand curve slopes downward because:

at lower prices the reward for holding the bond increases.

Which of the following is an example of the economies of scale argument for increased profits for large financial holding companies? a. Financial holding companies offer a wide array of services under one name b. Financial holding companies need only one CEO, one Board of Directors, and one accounting system regardless of size c. Financial holding companies are well diversified so risk is reduced d. The profitability of financial holding companies does not rely on one particular line of business

b. Financial holding companies need only one CEO, one Board of Directors, and one accounting system regardless of size

If a bank increases its assets by adding $1 to capital for every $1 added to assets: a. Leverage increases b. Leverage decreases c. Leverage stays constant d. The answer cannot be determined from the information in the question

b. Leverage decreases

The interest rate at which banks lend each other Eurodollars is known as: a. The international federal funds rate b. The London Interbank Offered Rate c. The discount rate d. The International Prime Rate

b. The London Interbank Offered Rate

As general business conditions improve, we would witness the following in the bond market:

bond prices decreasing

A decrease in expected inflation for any given nominal interest rate will cause:

bond prices to increase and interest rates to decrease

An increase in the nation's wealth, all other factors constant, would cause:

bond prices to rise and yields to decrease.

If the federal government were to offer larger tax breaks on the purchase of new equipment for businesses, all other factors constant, we would expect to see the:

bond supply curve shift right.

If the U.S. government's borrowing needs increase, in the bond market this would be seen as the:

bond supply curve shifting right

When expected inflation increases, for any given nominal interest rate the:

bond supply curve shifts right.

The holding period return has relevance because:

bonds are frequently traded.

. If Bank A sells some its loans to Bank B for cash, everything else equal: a. Bank A's assets decrease and Bank B's assets increase b. Bank A becomes less liquid while Bank B becomes more liquid c. Banks A's total assets do not change, but Bank A is more liquid d. Bank A's liabilities decrease by the amount of the loans that are sold

c. Banks A's total assets do not change, but Bank A is more liquid

Net interest income for a bank is: a. The difference between gross income and net income after taxes b. The interest banks earn from uses of funds c. The difference between interest income and interest expense d. The difference between interest income and total expenses

c. The difference between interest income and interest expense

A 10-year Treasury note as a face value of $1,000, price of $1,200, and a 7.5% coupon rate. Based on this information, we know the:

coupon payment on this bond is equal to $75.

A $1,000 face value bond purchased for $965.00, with an annual coupon of $60, and 20 years to maturity has a:

current yield equal to 6.22% and a coupon rate below this.

A $1000 face value bond, with one year to maturity that sells for $950 and has a $40 annual coupon has a:

current yield of 4.21%.

A bank that cannot meet its loan commitments is experiencing the results of: a. Interest rate risk b. Credit risk c. Trading risk d. Liquidity risk

d. Liquidity risk

The money aggregate M2 includes: a. Large denomination time deposits b. Stock and bond mutual fund shares c. Savings deposits but not money market deposit accounts d. M1

d. M1

. If a one-year bond currently yields 4% and is expected to yield 6% next year, the Liquidity Premium Theory suggests the yield today on a two-year bond will be: a. More than 4% but less than 5% b. 5% c. 4% d. More than 5%

d. More than 5%

The risks that banks are exposed to include each of the following, except: a. Interest rate risk b. Credit risk c. Operational risk d. Regulatory risk

d. Regulatory risk

According to the liquidity premium theory of the term structure, a flat yield curve indicates that short-term interest rates are expected to

decline moderately in the future.

According to the liquidity premium theory of the term structure, a downward sloping yield curve indicates that short-term interest rates are expected to

decline sharply in the future.

Suppose there is a decrease in the price at which a bondholder sells her bond. In this case, the holding period return will:

decrease, since this lowers the capital gain

Risk premiums on corporate bonds tend to ________ during business cycle expansions and ________ during recessions, everything else held constant.

decrease; increase

The Glass-Steagall Act, before its repeal in 1999, prohibited commercial banks from A) issuing equity to finance bank expansion. B) engaging in underwriting and dealing of corporate securities. C) selling new issues of government securities. D) purchasing any debt securities.

engaging in underwriting and dealing of corporate securities.

The U-shaped yield curve in the figure above indicates that short-term interest rates are expected to

fall sharply in the near-term and rise later on.

If interest rates are expected to rise, the bond prices will:

fall, due to the demand for bonds decreasing

The U.S. Treasury issues bonds where the return is indexed to the consumer price index. We should expect that these bonds, relative to other U.S. Treasury bonds, will have:

higher price and lower return due to the decreased risk from inflation in holding these bonds.

Foreign exchange rate stability is important because a decline in the value of the domestic currency will ________ the inflation rate, and an increase in the value of the domestic currency makes domestic industries ________ competitive with competing foreign industries. A) increase; more B) increase; less C) decrease; more D) decrease; less

increase; less

An increase in the riskiness of corporate bonds will ________ the yield on corporate bonds and ________ the yield on Treasury securities, everything else held constant.

increase; reduce

Consider a zero-coupon bond with a $1,100 payment in one year. Suppose the interest rate decreases from 10% to 8%. The price of this bond:

increases from $1,000 to $1,018.

U.S. government bonds that provide for bondholders to receive a fixed rate of interest plus the change in the consumer price index were designed to remove:

inflation risk.

The Federal Reserve System was created to A) make it easier to finance budget deficits. B) promote financial market stability. C) lower the unemployment rate. D) promote rapid economic growth.

promote financial market stability.

The Federal Reserve Banks are ________ institutions since they are owned by the ________. A) quasi-public; private commercial banks in the district where the Reserve Bank is located B) public; private commercial banks in the district where the Reserve Bank is located C) quasi-public; Board of Governors D) public; Board of Governors

quasi-public; private commercial banks in the district where the Reserve Bank is located

If the U.S. government's borrowing needs increase, all other factors constant the:

supply of bonds will increase.

The additional incentive that the purchaser of a Treasury security requires to buy a long-term security rather than a short-term security is called the

term premium

In reading bond quotes:

the asked price is usually above the bid price.

The bond dealer's spread is:

the asking price less the bid price

A decrease in the nation's wealth, all other factors constant, would cause:

the bond demand curve to shift left.

Default risk is the risk associated with:

the bond issuer not being able to make the promised payments.

According to the liquidity premium theory of the term structure

the interest rate on long-term bonds will equal an average of short-term interest rates that people expect to occur over the life of the long-term bonds plus a term premium.

Everything else held constant, if the tax-exempt status of municipal bonds were eliminated, then

the interest rate on municipal bonds would exceed the rate on Treasury bonds.

According to the expectations theory of the term structure

yield curves should be equally likely to slope downward as slope upward.

As general business conditions improve, all other factors constant the:

yield on bonds will increase.

the two parts that make up an option's price are:

C. the intrinsic value and the time value of the option

The term structure of interest rates is

the relationship among interest rates on bonds with different maturities.

Consider the factors that affect bond demand and bond supply. Describe how the following are likely to change during a period of robust economic growth: wealth, default risk, and general business conditions. For each, state how the factor is likely to change, and discuss the implications for bond demand/supply, bond price, and yield. Bond prices tend to decrease during periods of high economic growth. What does this reveal about which of these factors is important?

(i) Wealth affects bond demand and is likely to increase during an economic expansion. Households experience an increase in the value of their total assets (including stocks, for example). This leads to an increase in bond demand, increase in bond price, and decrease in yield. (ii) Default risk affects bond demand. The risk of default is likely to decrease during an economic expansion because borrowers are more likely to honor their debts. This leads to an increase in bond demand, increase in bond price, and decrease in yield. (iii) General business conditions affect bond supply. These improve during economic booms. This leads to an increase in bond supply, decrease in bond price, and increase in yield. If bond prices tend to decrease during periods of robust growth, this tells us that the improvement in general business conditions has a larger effect on the bond market than changes in wealth or default risk.

A 30-year Treasury bond as a face value of $1,000, price of $1,200 with a $50 coupon payment. Assume the price of this bond decreases to $1,100 over the next year. The one-year holding period return is equal to:

-4.17%

A 30-year Treasury bond as a face value of $1,000, price of $1,200 with a $50 coupon payment. Assume the price of this bond decreases to $1,100 over the next year. The one-year holding period return is equal to:

-4.17%.

Interest-rate swaps are:

Agreements between two parties to exchange periodic interest-rate payments over some future period.

A moral hazard situation arises in the lender of last resort function because:

A central bank finds it difficult to distinguish illiquid from insolvent banks.

Four features to hedge a stock portfolio:

1.) The current value of the stock portfolio 2.) Beta of stock portfolio 3.) Contract value of the index futures contract 4.) Beta of the futures contract

Which of the following are depository institutions? A. Credit unions B. Mutual funds C. Pension funds D. Insurance companies

A

An investor puts $2,000 into an investment that will pay $2,500 one-fourth of the time; $2,000 one-half of the time, and $1,750 the rest of the time. What is the investor's expected return? a. 12.5% b. $250.00 c. 6.25% d. 3.125%

3.125%

The principal-agent problem is:

A form of Moral Hazard

If a consol is offering an annual coupon of $50 and the annual interest rate is 6%, the price of the consol is:

50/.06=$833.33

If the expected path of one-year interest rates over the next five years is 4 percent, 5 percent, 7 percent, 8 percent, and 6 percent, then the expectations theory predicts that today's interest rate on the five-year bond is

6 percent

A monthly growth rate of 0.5% is an annual growth rate of: a. 6.00% b. 5.00% c. 6.17% d. 6.50%

6.17%

Consider a two-year, 4.5% coupon bond with a $500 face value that is originally purchased for $475. Calculate the holding period return of this bond if it is sold after one year at a price of $485

6.84%

How many members are on the Board of Governors of the Federal Reserve system?

7 (Ch. 16)

The Federal Reserve's Open Market Committee currently meets:

8 times a year (Ch. 16)

If a bank has $100 million in assets and a capital of $10 million, its debt-to-equity ratio is:

9 to 1

If a bank has $100 million in assets and a net worth of $10 million, its debt-to-equity ratio is:

9 to 1

Calculate the holding period return for a $1,000 face value bond with a $60 annual coupon purchased for $970.00 and sold three years later for $1,060.00.

9.02%. Here we have to consider the present value of the three coupon payments as well as the present value of the capital gain that results from purchasing the bond for $970 and selling it for $1,060.

Which of the following best expresses the formula for determining the price of a U.S. Treasury bill that matures n periods from now per $100 of face value when the interest rate is i? A. $100/(1 + i)n B. $100(1 + i) C. $100/(1 + i) D. 1 + $100/(1 + i)n

A

Which of the following statements is not true? A. The largest source of funds for banks to lend comes from the owner's capital. B. Transaction deposits make up less than 10 percent of banks sources of funds. C. The largest sources of funds for banks are non-transactions accounts. D. Borrowing is a larger source of funds for banks than transaction deposits.

A

Which of the following would lead to a decrease in bond demand? A. An increase in expected inflation. B. An increase in wealth. C. A decrease in risk. D. A decrease in liquidity.

A

10. What is a credit-default swap?

A CDS is a credit derivative that allows lenders to insure themselves against the risk that a borrower will default. The buyer of a CDS makes payments to the seller, and the seller agrees to pay the buyer if an underlying loan or security defaults.

Explain why a bank with a high debt-to-equity ratio may be more profitable than a bank with a lower ratio but would also have a higher level of risk.

A bank with a high debt-to-equity ratio is financing the acquisition of assets with borrowed funds. The return (profit) earned on these assets belong to the owners (the providers of the bank's capital or equity). With a high debt-to-equity ratio, the owners of the bank stand to earn a higher return on equity than the owners of a bank with a lower debt-to-equity ratio, assuming the same return on assets. The problem or the risk comes from the fact that with the high leverage anything that reduces the value of assets can potentially wipe out the capital of the bank, leaving the bank insolvent since the capital cushion is inadequate to meet the drop in asset value.

A moral hazard situation arises in the lender of last resort function because:

A central bank finds it difficult to distinguish illiquid from insolvent banks

. Comparing checks and currency, we can say: a. Both are money but only currency is legal tender b. Only checks are both money and legal tender c. A check isn't money but currency is d. Both are money and legal tender

A check isn't money but currency is

Use the example of a consol to show how bond prices and yields are inversely related.

A consol is a bond that pays a fixed payment forever but does not mature. In calculating the price of a consol we use the formula Price = Coupon/interest rate. This simple formula shows the inverse relationship between price and interest rate since price is on the left and interest rate is in the denominator on the right.

What is the relationship between financial market development and economic growth?

A country's economic growth is linked to financial market development. As the text points out, a country's financial system has to grow as its level of economic activity rises, or the country will stagnate. Economic research has shown that there is strong positive correlation between financial market development and economic growth across countries.

A $1000 face value bond purchased for $965.00, with an annual coupon of $60, and 20 years to maturity has:

A current yield equal to 6.22%.

A $1000 face value bond, with one year to maturity that sells for $950 and has a $40 annual coupon has: a. A current yield and yield to maturity of 4.00% b. A yield to maturity that equals the current yield c. A coupon rate of 4.00% and a current yield that is below this d. A current yield of 4.21%

A current yield of 4.21%

Holding risk constant, an investor earning 6% from a tax-exempt bond who is in a 25% tax bracket would be indifferent between that bond and: a. A taxable bond with a 8% yield b. A taxable bond with a 4.5% yield c. A taxable bond with a 6.25% yield d. A taxable bond with a 7.5% yield

A taxable bond with a 8% yield

11) The interest rate that is most commonly quoted by a lender is referred to as the: A) annual percentage rate. B) compound rate. C) effective annual rate. D) simple rate. E) common rate.

A) annual percentage rate.

A problem with the too-big-to-fail policy is that it ________ the incentives for ________ by big banks. A) increases; moral hazard B) decreases; moral hazard C) decreases; adverse selection D) increases; adverse selection

A) increases; moral hazard

assume we have a stock currently worth $50. We also assume the interest rate is zero, and we can buy options for this stock with a strike price of $50. if the stock can rise or fall by $10 with equal probability over the option period, and the option cannot be exercised until the expiration date, what's the time value of this option?

A. 5 10 x .5

if a futures contract for U.S Treasury bonds decreases by "17" in the financial page listings, the price of the contract decreased by:

A. 531.25 17 x 31.25

Deflation can cause widespread bank crises for all of the following reasons except: A. a decline in the value of borrowers' net worth but not their liabilities. B. borrowers' default rates increase. C. bank balance sheets deteriorate as the level of economic activity decreases. D. information asymmetry problems decrease during deflationary periods.

A. A decline in the value of borrowers' net worth but not their liabilities.

A bank supervisor examines the bank's portfolio of loans to see if the loans are being repaid in a timely manner. In terms of the acronym CAMELS, this would be part of rating the bank's: A. asset quality. B. losses. C. management. D. earnings.

A. Asset quality

What do bondholders and stockholders have in common?

A. Both are claimants

The S&P's 500 index:

A. Gives more weight to large companies than small companies

In the ten years after the FDIC limit was increased to $100,000: A. more than four times the number of banks and savings and loans failed than did during the first 46 years of FDIC's existence. B. less than one-fourth the number of banks and savings and loans failed than during the first 46 years of FDIC's existence. C. the cost to taxpayers of failed institutions in that period was negligible because FDIC was in place. D. increasing the deposit insurance limit to $250,000 provided complete coverage for all deposits except those of large corporations.

A. more than four times the number of banks and savings and loans failed than did during the first 46 years of FDIC's existence

if the price of the underlying asset has a standard deviation of zero:

A. options for this asset would likely not exist - no volatility

An individual who neither uses nor produces a commodity but sells a futures contract for the asset is:

A. speculating that the price of the commodity is going to fall

Forward contract

Agreement between a buyer and a seller, who both commit to a transaction at a future date at a price set by the negotiation today

Credit default swaps are credit derivatives that:

Allow lenders to insure themselves against the risk that a borrower will default.

Financial markets enable the transfer of risk by: a. Requiring that risk-averse investors have access to U.S. Treasury bond markets b. Allowing individuals and firms less willing to bear risk to transfer risk to other individuals and firms more willing to bear risk c. Making sure that higher default risk is offset by greater liquidity d. Enabling even unsophisticated investors to purchase highly complex financial instruments

Allowing individuals and firms less willing to bear risk to transfer risk to other individuals and firms more willing to bear risk

You win your state lottery. The lottery officials offer you the option of taking your winnings in one lump-sum payment, or fixed annual payments for the next 20 years. The sum of the 20 annual payments is larger than the lump-sum payment. Before deciding, what are the key factors you will want to consider that could influence your decision?

Although this is certainly a pleasant decision to think about, the options presented do require serious thought. You should start by finding the interest rate that equates the present value of the 20 payments with the lump sum. Next, compare this rate to the interest rate you think you could safely earn on the lump-sum if you invested it. If the market offers a higher interest rate, then this is a reason you may wish to take the lump sum. Although morbid, you should also consider your own life expectancy. If you do not think you are going to live another 20 years, you would certainly want to take your winnings early. Lifestyle is also important as is the degree of risk aversion you exhibit. One advantage to taking the payments over 20 years is that it is a form of expenditure discipline that may prevent you from going through the funds quickly (though it is highly likely that you will find plenty of sources that will loan you funds for the assignment of the winnings to them). If you are a person that benefits from external discipline, the 20-year payout may be more attractive. While this certainly is not an exhaustive list, it does show that many of the factors discussed in the chapter come into play in making a decision such as this on needs external discipline, the 20-year payout may be more attractive. While this certainly is not an exhaustive list, it does show that many of the factors discussed in the chapter come into play in making a decision such as this one.

6. Explain the difference between American and European options.

American options can be exercised on any date from the time they are written until the date they expire. As a result, the holder of an American option has three choices; (1) continue to hold the option; (2) sell the option to someone else; (3) exercise the option immediately. The holder of a European option has only two options on a date prior to expiration, hold or sell.

An annuity is a contract that makes monthly payments as long as someone lives. Explain why an individual would want to purchase such a contract. What risk is being transferred?

An annuity transfers the risk that the buyer will live longer than expected. If an individual had certainty regarding his/her life expectancy he/she could plan accordingly and set up a budget that would exhaust his/her wealth at the time of death. We do not usually have such certainty and the risk is that we may live longer than we expect and could run out of funds before we die. With an annuity the individual transfers this risk to a company (for a fee) who is pooling many of these individuals and with the "law of large numbers" is better equipped to take this risk.

Which of the following would lead to an increase in bond supply?

An improvement in general business conditions.

Which of the following would lead to a decrease in bond demand?

An increase in expected inflation.

Suppose that the return on assets other than bonds falls. In the bond market this will result in: a. A movement down the bond demand curve b. A shift to the left of the bond demand curve c. An increase in the price of bonds d. A shift to the left of the bond supply curve

An increase in the price of bonds

Which of the following is a problem of moral hazard?

An individual who purchases auto insurance an begins to leave his or her keys in the car while running into a store.

Which of the following is a problem of moral hazard?

An individual who purchases auto insurance and begins to leave his or her keys in the car while running into a store.

Which of the following is a problem of moral hazard?

An individual who purchases full auto insurance begins to leave his or her keys in the car while running into a store

A $5 million deposit outflow from a bank has the immediate effect of ________. A) reducing deposits and reserves by $5 million B) reducing deposits and loans by $5 million C) reducing deposits and securities by $5 million D) reducing deposits and capital by $5 million

Answer: A

A bank failure occurs whenever ________. A) a bank cannot satisfy its obligations to pay its depositors and have enough reserves to meet its reserve requirements B) a bank suffers a large deposit outflow C) a bank has to call in a large volume of loans D) a bank is not allowed to borrow from the Bank of Canada

Answer: A

Which of the following are reported as assets on a bank's balance sheet? A) Borrowings B) Reserves C) Notice deposits D) Bank capital

Answer: B

All of the following are examples of off-balance sheet activities that generate fee income for banks except ________. A) foreign exchange trades B) guaranteeing debt securities C) back-up lines of credit D) selling negotiable CDs

Answer: D

Explain why two countries with the same average rate of inflation may not present the same inflation risk for holders of those countries' bonds?

As the text points out, while the expected (or average) inflation can be the same, the standard deviation around this expected rate presents different amounts of risk. The higher the standard deviation, the greater the risk. For countries where inflation is volatile, the standard deviation around their average expected rate will be greater, and therefore their bonds will present greater inflation risk.

Which of the following is not a form of short-term loan in the shadow banking system?

Bank Deposits

The main reason why banks are the leading sources of external finance for businesses is:

Banks have an information-cost advantage in reducing adverse selection.

The government's role of lender of last resort is directed to:

Banks that experience sudden deposit outflows

Which of the following is not a reason why the yield to maturity can differ from the current yield?

Because the current yield moves in the opposite direction from price.

A decrease in expected inflation for any given nominal interest rate will cause: a. Bond prices to increase and interest rates to decrease b. Bond prices to decrease and interest rates to increase c. The bond demand curve to shift to the left d. The bond supply curve to shift to the left

Bond prices to increase and interest rates to decrease

Bank's hold marketable securities as a part of their assets. For the U.S. banks these marketable securities include:

Bonds only

Suppose a bank experiences a sudden and large deposit withdrawal. As a part of its liquidity management this bank should:

Borrow from other banks (Ch. 12)

The moral hazard that can result from debt financing is mainly due to the:

Borrower taking greater risk in hopes of obtaining a larger return (Ch. 11)

The moral hazard that can result from debt financing is mainly due to the:

Borrower taking greater risk in hopes of obtaining a larger return.

Moral hazard is not eliminated in debt financing because:

Borrowers have an incentive to assume greater risk than is in the interest of the lender.

1. The value of an option is dependent upon the value of the underlying security. This relationship defines an option as which one of the following? A. equity security B. fixed income security C. derivative security D. transfer security E. dependent security

C

10. Financial intermediaries: A. Increase the cost of financial transactions but offset these higher costs by providing safekeeping of customer funds B. Provide handling of payments but usually less efficiently than other firms C. Reduce the cost of financial transactions D. Provide safety of resources, but only for the large borrowing customers who can afford it

C

11. A pension fund manager who plans on purchasing bonds in the future: a. wants to insure against the price of bonds falling. b. can offset the risk of bond prices rising by selling a futures contract. c. will take the long position in a futures contract. d. will take the short position in a futures contract.

C

14. The fact that financial intermediaries employ experts to carry out particular activities and so lower transactions costs is usually associated with the following economic concept: A. The law of demand B. Economies of scale C. Comparative advantage D. Information costs

C

15. Which of the following is not a financial instrument? A. A share of Microsoft stock B. A U.S. Treasury Bond C. An electric bill D. A life insurance policy

C

16. Examples of economies of scale are: A. The additional fees financial intermediaries charge on small accounts B. The decrease in overall transaction costs that occur as volume increases C. The reduction in the cost per transaction that occurs as the number of transactions increase D. The decrease in overall information costs that occurs as more transactions are handled

C

17. One reason financial intermediaries earn profits is because: A. Individuals are not aware of the true cost of using an intermediary B. Financial intermediaries are charging for services people do not value C. Individuals are willing to pay for the reduction in transaction costs financial intermediaries provide D. They raise the cost of transactions and pass these higher costs on to customers

C

2. A call option grants its owner which one of the following? A. right to buy B. obligation to buy C. right to sell D. obligation to sell E. choice to either buy or sell

C

The difference between a bank's reserves and its required reserves is: A. profits. B. net interest income. C. excess reserves. D. vault cash.

C

Program trading

Computer-assisted monitoring of relative prices of financial assets. Used to exploit perceived arbitrage opportunities

What are some of the advantages of trading in decentralized electronic exchanges?

Customers can see the orders, the orders are executed quickly, trading occurs 24 hours a day, and costs are low.

Futures contract

Contract between a seller and a buyer specifying a commodity or financial instrument to be delivered and price paid at contract maturity.

Sue uses a credit card to purchase a new pair of jeans. Sue is: a. Using money to buy her jeans since credit cards is money b. Creating a liability that she will ultimately have to pay with money c. Using an electronic payment form of money d. Using a form of money included in M2

Creating a liability that she will ultimately have to pay with money

Consider a $1,000 face value bond with a $55 coupon payment and 1 year to maturity. Calculate the current yield, coupon rate and the yield to maturity if the bond is purchased for $940.

Current yield = 5.85%, coupon rate = 5.50%, yield to maturity = 12.23%.

54. Which one of the following values is discounted in the put-call parity formula? A. call price B. put price C. stock price D. strike price E. option premium

D

An over-the-counter (OTC) market is: A. made up of dealers who only sell government bonds. B. an example of a centralized market. C. made up of dealer who buy and sell only for their own accounts. D. made up of dealers who buy and sell for their customers and for their own accounts.

D

Which of the following statements is NOT true? a. a value-weighted index is a better index to use to reflect changes in the economy's overall wealth b. a price-weighted index is a better index to use to reflect the average change in the price of a typical share of stock c. the DJIA is a price-weighted index D. The S&P 500 is a price-weighted index

D. the S&P 500 is a price weighted index -> FALSE the S&P 500 gives more weight to larger companies

as the time of settlement gets closer:

D. the price of the futures contract will move in lockstep with the price of the underlying asset

someone who purchases a call option is really buying insurance to protect against:

D. the price of the stock rising

13. Which one of the following terms is defined as the payoff that would be received if an option were expiring immediately? A. parity price B. market price C. time value D. underlying value E. intrinsic value

E

1) Which one of the following statements correctly defines a time value of money relationship? A) Time and future values are inversely related, all else held constant. B) Interest rates and time are positively related, all else held constant. C) An increase in a positive discount rate increases the present value. D) An increase in time increases the future value given a zero rate of interest. E) Time and present value are inversely related, all else held constant.

E) Time and present value are inversely related, all else held constant.

18) A loan that calls for periodic interest payments and a lump sum principal payment is referred to as a(n) ________ loan. A) amortized B) modified C) balloon D) pure discount E) interest-only

E) interest-only

10) A Canadian consol is best categorized as a(n): A) ordinary annuity. B) amortized cash flow. C) annuity due. D) discounted loan. E) perpetuity.

E) perpetuity.

If you were going to issue bonds, would you prefer to be in a country where the average inflation rate is 3% inflation but fluctuates wildly, or in a country with a higher, 4% expected inflation rate that is stable (meaning it's always 4%). Explain.

Even though the 4% expected rate is higher, it is stable. As we saw, the inflation risk isn't really the risk from inflation; it is the risk that results from unexpected changes in inflation which then can significantly alter the real interest rate, and therefore the real returns bondholders receive. Because bondholders tend to be risk-averse, they would want to be compensated for the inflation risk, and since the inflation risk results from the fluctuations in the rate of inflation, the returns required by bondholders in the country where the average expected rate is 3% but volatile are likely to be higher than the required returns on the bonds in the higher but stable inflation country. This explains, at least partially, why the central banks in many developed countries strive for inflation stability. Stable prices will lead to lower inflation risk and a more efficient bond market.

Under the purchase-and-assumption method of dealing with a failed bank, the FDIC:

Finds another bank to take over the insolvent bank

The problem for a central bank setting a zero inflation policy would be:

Firms would have to cut the nominal wage to reduce the real wage

Which of the following is not a role of a financial institution acting as a financial intermediary?

Formulating oversight regulations

Use our model of the bond market (supply and demand) to explain what happens if the U.S. economy continues to grow at robust rates.

Growth in the economy should result in greater supply of bonds, the bond supply curve shifting right, as more firms seek resources to finance expansion and inventories. The increase in supply by itself would result in lower bond prices and higher interest rates. From the demand side, the robust economy should also cause an increase in wealth. The increases in wealth would cause bond demand to increase (the curve shifts right), which would drive up bond prices and decrease interest rates. The net effect will be determined by which shift is larger. If supply shifts by an amount greater than demand, the bond prices will fall and yields will rise. On the other hand, if demand increases by more than supply, the bond prices will rise and yields will fall.

Direct finance:

Has recently become a much more important source of finance.

Which of the following could the lemons problem, applied to financial markets, explain?

High quality potential borrowers relying more on internally generated funds to finance investment.

Bank mergers require government approval because banking officials want to make sure that:

If a merger has a small community bank taken over by a larger regional bank, that the customers of the small town will still be well-served

If the Federal Reserve were to do away with the required reserve regulation, do you think banks would stop holding reserves? Explain.

If the Federal Reserve did away with the required reserve regulation, the likely outcome is there wouldn't be much of a change in the reserve holdings of most depository institutions. The reason for this is that these institutions are very sensitive to managing liquidity risk and as a result would want to have liquid assets on hand to meet the liquidity needs of depositors.

In the U.S., one problem with central bank independence is:

In a representative democracy, monetary policymakers must be held accountable to the public.

Calculate the monthly payment for a 30-year mortgage, where the amount borrowed is $100,000 and the annual interest rate is 6.0%.

If we use 360 months for the 30 years and convert the 6.0% annual rate to a monthly rate of 0.48676%, [this is found by solving (1 + im) = (1.06)1/12 where im = 0.0048676]. Using a financial calculator, we find the monthly payment equals $589.37.

If you focus on interest-rate risk, can you explain why banks offer higher interest rates on longer-term CDs than they do on short-term CDs?

In looking at bank balance sheets one important difference is that bank assets tend to be long term and its liabilities tend to be short term. This mismatch between maturities creates interest-rate risk. One way to reduce the risk for banks is to try to move more liabilities from short to long term. By offering higher rates on longer-term CDs banks may be able to lure savers from shorter-term to longer-term CDs thus increasing the average maturity of the banks' liabilities.

4. Consider a call option; in terms of the option writer and option holder, who is the buyer? Who is the seller? Finally, who has the option? Explain.

In the case of a call option, the option writer is the seller. Here the option writer is stating the underlying asset, strike price, and expiration or delivery date. The option holder is the buyer of the option. The option holder buys the right to have the option of actually purchasing the underlying asset on or before the expiration date for the strike price. The option holder has the option, because she could let the option expire and not "call away" the underlying asset, just foregoing the price paid for the option.

11. How did CDS' contribute to the financial crisis of 2007-2009?

In three important ways: (1) fostering uncertainty about who bears the credit risk on a given loan or security, (2) making the leading CDS sellers mutually vulnerable, and (3) making it easier for sellers of insurance to assume and conceal risk.

A bank's off-balance-sheet activities usually:

Increase its net income but do not change its assets or liabilities.

Financial intermediaries, through their ability to lower transaction costs:

Increase the amount of trading that occurs in an economy.

Consider a zero-coupon bond with a $1,100 payment in one year. Suppose the interest rate decreases from 10% to 8%. The price of this bond:

Increases from $1,000 to $1,018.

The fact that a bank's assets tend to be long-term while its liabilities are short-term creates:

Interest-rate risk.

Stocks appear to present risk, yet many people have substantial parts of their wealth invested in them. This behavior could be explained by the fact that:

Investing in stocks over the long run is not as risky as short-term holdings.

The moral hazard problem cause by government safety nets:

Is greater for larger banks

The federal funds market:

Is the inter-bank market where excess reserves from one bank can be loaned to another bank (Ch. 12)

Why are U.S. banks prohibited from owning stocks?

It probably has a lot to do with two main issues. One is the liquidity of these securities. The markets for these securities at times can be less liquid than say for U.S. treasury securities, and that can have an adverse impact on a bank that needs to get liquidity. Another issue can stem from fluctuating market values for these securities. As their market values fluctuates so will the value of the banks' assets. Especially in those instances when market values may drop significantly (stock market crashes, etc.), banks may find that the value of their assets falls so much their net worth disappears and they may become insolvent. Compounding this will be the potential runs on banks if it is rumored that a particular bank owned a lot of stock of a company that is failing.

12. Explain why for speculation, the purchase of an option may be more attractive than a futures contract or the outright purchase of the underlying asset.

Let's say an investor believes that interest rates are going to fall over the next few months. There are three ways to bet on this possibility. One is to purchase a bond and if the investor guesses correctly, the bond price will rise as the interest rate falls. This is expensive since it requires the purchase of the bond. Another strategy is to purchase a futures contract, meaning take the long position. If the market price of the bond increases with falling interest rates, the investor will reap the profits. This approach requires a small investment, but this approach is also very risky since the investment is highly leveraged since the market price can move against the investor. A third strategy involves the use of an option. The investor could purchase a call option on a Treasury bond. If he or she is right and interest rates fall, the value of the call option will rise, which is the upside. On the other hand, if the investor bets wrong and interest rates rise, the option will expire worthless and the investor just loses the fee paid for the option. The bet is both highly leveraged and limited in its potential losses.

A late-night news report says the president of a local bank is about to be arrested for embezzling money from the bank at which he works. This causes most of the depositors to line up in front of the bank the next morning wanting to withdraw their deposits. This is an example of:

Liquidity risk.

An economic rationale for government protection of small investors is that:

Many small investors cannot adequately judge the soundness of their bank

Which of the following is a bank asset?

Mortgage loans.

Explain why most financial instruments are fairly complex, while at the same time quite standardized.

Most financial instruments are complex in the sense that many possible contingencies are identified and both buyer and seller want to avoid problems that can arise from unforeseen events. To write a complete contract, however, would be very time consuming and expensive. As a result, most financial instruments are standardized because over time many common problems have been identified and worked into the contract, and standardization makes it easier to compare contracts and makes the instruments more liquid.

Considering the value of a financial instrument, the circumstances under which the payment is to be made influence the value because: a. We like uncertain payoffs because this adds to the return b. Payments that are made when we need them the most are more valuable c. The sooner the payment is to be made the better d. We know when certain events are going to occur and that is when we want the payment

Payments that are made when we need them the most are more valuable

The payoff method used by the FDIC to address the insolvency of a bank is when the FDIC:

Pays off the depositors up to the current $250,000 limit, so it is possible that some depositors will suffer losses

What should be the impact on a bank's return on assets and return on equity from increased use of off-balance-sheet activities?

Since off-balance-sheet activities generate income but do not add to assets or liabilities, the impact should be to increase the return on assets. Since ROA is calculated by dividing net income after taxes by total assets, only the numerator is increasing, not the denominator, so ROA should increase. The same thing could be said about return on equity (ROE); the net return after taxes increases, but the bank's capital does not, so ROE increases. This is the main reason banks are increasing their use of off-balance-sheet activities.

Capital is the cushion banks have against:

Sudden drops in the value of their assets.

Explain why the bid-ask spread on most municipal bonds would be greater than the spread on U.S. Treasury bonds.

Spreads are the difference between the dealer's bid and asked prices. Since dealers are ready to buy or sell the bond, they must carry an inventory, which means they accept risk just like any other bondholder would. One of these risks is liquidity risk, which is the risk of not being able to sell the bond when you would like. Since the market for U.S. Treasury bonds is far more liquid than would be the market for any single municipal bond, the dealer of the municipal bond would face greater liquidity risk and require a larger spread.

One reason it took so long to have a central bank in the United States is that:

States feared centralization of power (Ch. 16)

Index Arbitrage

Strategy of monitoring the futures price on a stock index and the level of the underlying index to exploit deviations from parity

Why didn't the over-the-counter (OTC) exchanges suffer the disruption of service that the New York Stock Exchange did after the terrorist attacks of September 11, 2001?

The New York Stock Exchange is a centralized exchange, meaning it is one physical location. Since it was located in New York near the World Trade Center it had to close as it was impossible for people to get into the area. The OTC exchange on the other hand is electronic networks where each dealer is linked electronically to other dealers. As a result, the bombing in New York certainly disrupted the ability of some New York dealers to trade, but the remainder of the exchange continued to function.

A home buyer is presented with two options for financing the purchase of a home: a 20 year fixed rate mortgage or a 20 year adjustable-rate mortgage, where the rate adjusts once a year. Which mortgage would you expect to start at the lowest interest rate and why?

The adjustable-rate mortgage should start at a lower interest rate. In the case of an adjustable-rate mortgage, the lender is able to transfer a significant amount of the interest-rate risk to the borrower. But as we saw in a previous chapter, most people are risk averse and in order to take on increased risk the individual requires compensation. Here the borrower is enticed to take on the interest-rate risk with the compensation in the form of a lower starting interest rate on the mortgage.

A bank has $100 million in assets and 50 percent of its assets are interest sensitive. The bank has $75 million in liabilities, 50 percent of which are interest sensitive. What is the bank's gap between interest-sensitive assets and liabilities?

The amount of assets that are interest sensitive is 50 percent of $100 million or $50 million. The amount of liabilities that are interest sensitive is 50 percent of $75 million which is $37.5 million. Subtracting the interest-sensitive liabilities from the interest-sensitive assets leaves a gap of +$12.5 million.

Can a financial instrument be bought and sold in both a primary and secondary financial market? Explain.

The answer is yes and highly likely. When a financial instrument is new, say a newly issued U.S. Treasury bond, it is initially sold in a primary financial market. Perhaps the bond is purchased directly by the Federal Reserve. At some later time, however, the Federal Reserve may decide to sell the bond and this transaction would be a secondary market transaction since the instrument already exists.

Describe what is likely to happen to the average price of a share of stock if the stock markets decide to close every Friday and Monday to provide workers at the exchanges with longer weekends.

The average price of stocks would decrease. The fact that the markets are open less decreases the liquidity of stocks and, as a result, their prices would have to be lower in order to entice savers to hold these instruments.

You are provided with the following information: a bank has a net income after taxes of $3.5 million; it has assets of $150 million; and bank capital of $12.5 million. What is the bank's return on assets; its return on equity, and its debt-to-equity ratio?

The bank's return on assets is 2.33%; this is obtained by dividing the $3.5 million in net income after taxes by the asset amount of $150 million and multiplying by 100 to convert the decimal into a percentage. The return on equity is 28.0%. We divide the $3.5 million in this case by the equity of $12.5 million and multiply by 100 to obtain our answer. The debt-to-equity ratio requires we first obtain the amount in liabilities. This is done by realizing that bank capital (equity) and total liabilities must equal total assets. So taking the amount in assets, $150 million, and subtracting the amount in capital, $12.5 million, leaves us with liabilities of $137.5 million. Dividing the $137.5 million by $12.5 million produces a debt-to-equity of 11 to 1.

If a bank's return on equity remains constant, but the ratio of bank assets to bank capital increases:

The bank's return on assets must have decreased

Economies of scale associated with financial intermediaries means:

The cost per transaction falls as a larger volume of similar transactions are handled.

Explain the relationship between coupon rate (or coupon yield) and current yield.

The coupon rate is simply the annual coupon divided by the face value. The current yield is the annual coupon divided by the price of the bond. The only time these should equal each other is when the price of the bond equals the face value. If the price is greater than the face value the current yield should be less than the coupon rate. If the price of the bond is less than the face value, the current yield should be greater than the coupon rate.

Which of the following statements are TRUE?

The expected return on corporate bonds decreases as default risk increases.

Suppose that the interest rate on a conventional 30-year mortgage is currently 8%. You receive a call from a mortgage broker who offers you a 30-year adjustable rate mortgage at 2% that is adjusted once each year. Evaluate each mortgage in terms of the following: risk that the monthly payment will change over the next 30 years and interest-rate risk.

The fixed-rate mortgage protects the borrower and the lender from any risk that the monthly payment will change. Whether the interest rate increases (reducing the present value to the lender) or decreases (increasing the present value to the lender), the payment is always the same. The risk that the present value of the mortgage will change because of a change in interest rates is the interest-rate risk. The adjustable rate mortgage has no interest-rate risk (because the present value is adjusted), but there is considerable risk that the monthly payment will change.

Identify the four broad categories that make up the asset side of the balance sheet for banks and which category is usually the largest.

The four categories that make up the asset side of the balance sheet for banks include: cash items, securities, loans and all other assets. The largest category is usually loans.

A bank that meets deposit withdrawal by borrowing additional funds will alter:

The liabilities side of the balance sheet

Explain why a bank manager and a bank regulator would likely view the timing at which a loan should be charged to the loan loss reserve differently.

The loan loss reserve is part of a bank's capital. A loan is part of a bank's assets. To charge a loan against this reserve will reduce both the bank's assets and its capital. A manager will try to avoid this scenario. Regulators, on the other hand, are more concerned with the bank's ability to meet its obligations, including the liquidity needs of its depositors. Non-performing loans, while they may appear as an asset are really not. The regulators would rather charge these off to the loan loss reserve and obtain a clearer view of the bank's actual capital position and its ability to meet its obligations and also to withstand a shock.

Banks do not hold many of their assets in the form of cash mainly because of:

The opportunity cost of holding cash; cash does not earn interest.

Why has the pace of structural change in financial markets accelerated in recent years?

The pace of structural change has accelerated dramatically in the past few years, driven by (1) ongoing technological advances in computing and communications and (2) increasing globalization. The former dramatically lowered the importance of a physical location of an exchange—as new technology allowed the rapid low-cost transmission of orders across long distances—while the latter encouraged unprecedented cross-border mergers of exchanges, integrating larger pools of providers and users of funds.

What is meant by the "paradox of leverage?"

The paradox of leverage occurs when all financial institutions try to deleverage at once. This will prove counterproductive as falling asset prices will mean more losses, diminishing their net worth still more, raising leverage and making the assets they hold seem riskier, compelling further sales, and so on.

As the time of settlement gets closer:

The price of the futures contract will move in lockstep with the price of the underlying asset.

The theory of efficient markets assumes that:

The prices of all financial instruments reflect all available information.

One reason lenders usually require a lot of information form loan applicants is to avoid:

The problem of adverse selection

One reason lenders usually require a lot of information from loan applicants is to avoid:

The problems of moral hazard (Ch. 11)

Examples of economies of scale are:

The reduction in cost per transaction that occurs as the number of transactions increase

Spot-futures parity

The relationship between spot and futures prices that holds in the absence of arbitrage opportunities

When you deposit $50 into your account and First Nation Bank and a $100 check you wrote on this account is cashed at a Chemical Bank, then:

The reserves at First National Bank fall by $50

Considering that, on average, the return on assets is the same for small and large banks, and the return on equity is higher for large banks than small banks, what can be one of the explanations for the trend toward bank mergers?

The return on assets being the same suggests that large banks may not have any advantage in managing assets than small banks. The fact that large banks have a higher return on equity means that they must have higher leverage. This suggests that there are economies of scale in banking.

Consider two investors: one is risk-neutral and the other is risk-averse. How do they each assess a risk premium?

The risk-neutral investor seeks a risk premium that has the price of the bond (and its subsequent yield) such that the price equals the expected value (the sum of the payoffs times the probabilities). The risk-averse investor would offer a price less than this (and therefore seek a higher yield) since he/she requires additional compensation for risk. Therefore, the risk-averse investor's risk premium would be greater.

At the time the government of Bulgrovia issued new bonds, they issued them at a price that reflected the risk-free rate because investors had no concerns regarding default risk, so did not require a risk premium. That risk-free rate was 4%. These bonds currently have one year to maturity and you notice the yield is 20%. Can you calculate the probability that the Bulgrovian government will default?

The simple answer is no. A risk-premium is a measure of the premium required by investors to accept risk; it is not a direct measure of the risk of default. We could only determine this if investors are risk-averse. If that were true then, we can calculate the probability fairly easily by realizing the probability the bond will not default can be expressed by 1.04/1.20, which equals 0.867. If we subtract this from 1.0 we obtain the probability of default which is 0.133.

In mid-2004 there was speculation that the Federal Reserve would be raising interest rates before the end of the year. How would this news affect the bond market and why?

The speculated increase in interest rates by the Federal Reserve would cause the value of bonds to decrease. As we saw in the text, an increase in the expected future interest rate makes bonds less attractive. This will lower the demand for bonds, causing bond prices to decrease and yields to increase. Moreover, the change in bond prices and yields will occur before the actual interest rate changes since existing and prospective bondholders will act on their expectations.

A put option described as in-the-money would find:

The strike price is above the market price of the stock.

A student receives a five-year loan to pay for a $2,000 used car. The lender and the student agree to an 8% interest rate on a fixed-rate loan. Expected inflation was estimated to equal 2.5%, but unexpectedly decreases to 2%. Which of the following is true?

The student is made worse off because her real cost of borrowing is higher.

If the U.S. government's borrowing needs increase, all other factors constant:

The supply of bonds will increase.

Financial intermediaries exist, in part, because:

The transaction and informational costs associated with direct finance can, at times, be prohibitive. (Ch. 11)

Financial intermediaries exist, in part, because:

The transaction cost associated with direct finance can at times be prohibitive.

One of the unique problems that banks face is:

They hold liquid assets to meet liquid liabilities

Why are stock market bubbles costly for the economy?

They lead to a misallocation of resources in both the short-term and long-term.

Considering a bank's balance sheet, which of the following statements is false?

Total Bank Assets = Total Bank Liabilities + Total Bank Capital

A bank has a need for cash for a short period of time to meet its liquidity needs. The bank has significant holding of U.S. treasury securities. The bank really does not want to sell the securities, realizing the liquidity need is a temporary problem. A pension fund has significant cash holdings and would like to earn some return on part of these holdings. The problem is the fund will need this cash in a few days to honor a purchase agreement it made for municipal bonds being issued. Is there any way these two organizations can work together to solve each other's problem?

This is a situation that is ideal for a repurchase agreement. Here the bank would agree to sell the Treasury securities (or some portion of them) to the pension fund in return for cash, and at the same time the bank would agree to buy the securities back at a future date (usually a day or two later) at a price equal to what the bank sold them for plus an additional amount reflecting the interest payment on the loan. In this way the bank gets the cash it needs and the securities back in a few days, and the pension fund earns income from the short-term loan and retains its cash position to make the securities purchase it planned.

Could the holding period return ever be less than the yield to maturity? Explain.

This is possible under the condition that the bond is sold before it matures for an amount less than the face value. If this happened then the holding period return would be less than the yield to maturity.

Hedger

Trader who seeks to transfer price risk by taking a futures position opposite to an existing position in the underlying asset

The purpose of derivatives is to:

Transfer the risk from one person to another.

Which of the following bonds are considered to be default-risk free?

U.S. Treasury Bonds

Why are options referred to as derivative instruments?

Unlike underlying instruments, such as stocks and bonds, derivatives are instruments where the value and the payoff of the instrument are derived from the behavior of the underlying asset. As an example, suppose Tom has a contract allowing him to purchase 100 shares of stock in ABC company at a price of $10 per share six months from now. The value of his option contract will increase as the actual price of the ABC stock (the underlying instrument) rises and exceeds $10 per share.

Calculate the price of a $1,000 face value bond that offers a $45 annual coupon, and has six years to maturity, when the interest rate is 6.0% (0.060).

Using a financial calculator the price of the bond is $926.24. We insert $1000 for the face (future) value; $45 for the annual payment, 6.0 for the annual interest rate, 6 for the N (or years) and solve for P (or PV on most calculators).

Which fact about the term structure is the Expectations Theory unable to explain? a. Why interest rates on bonds with different terms to maturity tend to move together over time b. Why yields on short-term bonds are more volatile than yields on long-term bonds c. Why longer-term yields tend to be higher than shorter-term yields d. Why yields on short-term bonds are more volatile than yields on long-term bonds and why longer-term yields tend to be higher than shorter-term yields

Why longer-term yields tend to be higher than shorter-term yields

Once you buy a coupon bond, which of the following can change? a. Coupon rate b. Coupon payment c. Face value d. Yield to maturity

Yield to maturity

Which of the following statements is most accurate?

Yield to maturity is the same as the coupon rate if the bond is purchased for face value and held to maturity.

Reverse trade

a trade that closes out a previously established futures position by taking the opposite position

A plot of the interest rates on default-free government bonds with different terms to maturity is called

a yield curve

Which of the following statements best describes financial instruments? a. All financial instruments are a means of payment b. Financial instruments can transfer resources between people but not risk c. Financial instruments can transfer resources and risk between people d. Financial instruments can transfer risk but not resources between people

c. Financial instruments can transfer resources and risk between people

If a bank has more interest-rate sensitive liabilities than interest-rate sensitive assets, an increase in the interest rate will cause profits to: a. Increase b. Decrease c. Remain constant d. Be negative, meaning there will not be profits, only losses

b. Decrease

. Suppose the economy has an inverted yield curve. According to the Liquidity Premium Theory, which of the following interpretations could be used to explain this? a. Interest rates are expected to rise in the future b. Investors expect an economic slowdown c. Investors are indifferent between bonds with different time horizons d. The term spread has increased

b. Investors expect an economic slowdown

A $1,000 face value bond, with an annual coupon of $40, one year to maturity and a purchase price of $980 has a:

current yield that equals 4.08% and a yield to maturity that equals 6.12%.

When expected inflation decreases for any given nominal interest rate, all of the following occur except the:

cost of borrowing increases and the desire to borrow decreases.

In calculating the current yield for a bond the:

coupon payment and purchase price is all that is needed.

If a bond's purchase price equals the face value the:

coupon rate equals the yield to maturity, which equals the current yield

The procedure that estimates the interest-rate sensitivity of a bank's assets and liabilities is called: a. Managing credit risk b. Estimating operating risk differential c. Trading risk minimization d. Gap analysis

d. Gap analysis

Competition between banks A) encourages greater risk taking. B) encourages conservative bank management. C) increases bank profitability. D) eliminates the need for government regulation.

encourages greater risk taking.

The current yield of a bond:

equals zero for a zero-coupon bond since these bonds have no coupon payments.

If bonds with different maturities are perfect substitutes, then the ________ on these bonds must be 18) equal.

expected return

Most home mortgages are good examples of:

fixed-payment loans.

The bond supply curve slopes upward because:

for companies seeking financing, the higher the price of bonds the more attractive it is to sell bonds.

The typical shape for a yield curve is

gently upward sloping

The demand for U.S. government bonds is high relative to other bond issues because:

market for U.S. government bonds is more liquid than most if not all other bond markets.

The risk premium on corporate bonds reflects the fact that corporate bonds have a higher default risk and are ________ U.S. Treasury bonds.

less liquid than

The size of the bond dealer's spread is mainly a function of the:

liquidity of the bond market

A particularly attractive feature of the ________ is that it tells you what the market is predicting about future short-term interest rates by just looking at the slope of the yield curve.

liquidity premium theory

Three factors explain the risk structure of interest rates

liquidity, default risk, and the income tax treatment of a security.

In considering the holding period return, the longer the term of the bond the:

more important is the capital gain.

Bond prices and yields:

move together inversely.

An increase in expected inflation for any given nominal interest rate will cause the:

price of bonds to decrease.

If the U.S. government's borrowing needs decrease, all other factors constant the:

price of bonds will increase

The price of a coupon bond can best be described as the:

present value of the face value plus the present value of the coupon payments.

When a loan is amortized, it means the:

principal and interest are paid off by the borrower over the life of the loan.

A zero-coupon bond refers to a bond which:

promises a single future payment.

When the current yield and the coupon rate are equal, the bond is:

purchased at a price that equals the face value.

If the U.S. government's borrowing needs increase, in the bond market this would be seen as:

the bond supply curve shifting right.

An increase in expected inflation for any given nominal interest rate will cause:

the real return to bondholders to decrease

The risk structure of interest rates is

the relationship among interest rates of different bonds with the same maturity.

a put option that is described as in the money would find:

the strike price is above the market price of the stock

If the purchase price of a bond exceeds the face value, the yield to maturity:

will be less than the coupon rate because the capital gain will be negative.

The holding period return on a bond:

will be less than the yield to maturity if the bond is sold for less than face value.

An inverted yield curve predicts that short-term interest rates

will fall in the future.

The impact of a decrease in expected inflation in the bond market will have a relatively large effect on the prices of bonds prices because the bond demand curve:

will shift right but the bond supply curve shifts left.


Ensembles d'études connexes

Ch. 43 Hematologic and Immunologic Dysfunction

View Set

Economics Guided Reading 2.2-2.3

View Set

Mitosis/Meiosis Lecture Quiz Ch 6+7

View Set